Nguyên lý cực hạn thường được áp dụng một cách hiệu quả trong các bất đẳng thức có tính tổ hợp, dạng chứng minh tồn tại k số từ n số thỏa mãn một điều kiện này đó.. Ví dụ 5.[r]
(1)Mục lục
Lời nói đầu Trần Nam Dũng
Nguyên lý cực hạn Trịnh Đào Chiến, Lê Tiến Dũng
Một số dạng tổng quát phương trình hàm Pexider áp dụng 16 Lê Sáng
Xây dựng lớp phương trình hàm nhờ đẳng thức lượng giác 24 Lê Thị Anh Đoan
Tính ổn định nghiệm số phương trình hàm Cauchy 35 Trần Viết Tường
Một số lớp phương trình hàm đa ẩn sinh phi đẳng thức 47 Lê Sáng, Nguyễn Đinh Huy
Từ công thức Euler đến toán số phức 57 Nguyễn Thị Tình
Một số ứng dụng phương trình Pell 67 Huỳnh Bá Lộc
Phép lượng giác cơng cụ giải tốn thi chọn học sinh giỏi 79 Nguyễn Trung Hưng
Sử dụng vành số nguyên để giải số toán số học 89 Phạm Thị Thúy Hồng
Nội suy theo yếu tố hình học đồ thị 96 Lê Sáng, Vũ Đức Thạch Sơn
(2)Lê Thị Thanh Hằng
Một số dạng tốn liên quan đến dãy số có quy luật 120 Trương Văn Điềm
Vận dụng tính đơn điệu tốn tìm giới hạn dãy số giải phương trình, bất phương trình, hệ phương trình 134 Huỳnh Tấn Châu
Ứng dụng số định lý giải tích 155 Lê Văn Thẩn
Một số phương pháp giải hệ phương trình 166 Huỳnh Kim Linh, Tô Hùng Khanh
Một số tốn đa thức kì thi học sinh giỏi 179 Nguyễn Văn Ngọc
Một số toán chia hết đa thức đối xứng 187 Huỳnh Duy Thủy
Nét đẹp hàm số tiềm ẩn toán bất đẳng thức, tốn tìm giá trị lớn giá trị nhỏ 195 Nguyễn Tài Chung
Thêm phương pháp để chứng minh bất đẳng thức 204 Tố Nguyên
Một số vấn đề phép nghịch đảo mặt phẳng ứng dụng 213 Trần Văn Trung
Sử dụng số tính chất ánh xạ để giải tốn phương trình hàm số 235 Nguyễn Hữu Tâm - Hoàng Tố Qun
(3)Lời nói đầu
Hịa nhịp với tuổi trẻ nước hoạt động sôi kỉ niệm ngày thành lập Đoàn niên Cộng sản Hồ Chí Minh thi đua lập thành tích chào mừng ngày sinh Bác Hồ kính yêu, tiến tới kỉ niệm 37 năm ngày giải phóng Nha Trang thực chương trình đổi giáo dục phổ thơng, Sở Giáo Dục Đào tạo Khánh Hịa phối hợp với Hội Toán học Hà Nội đồng tổ chức Hội thảo khoa học Các chuyên đề Toán học bồi dưỡng học sinh giỏi THPT khu vực Duyên hải Nam Trung Tây nguyên
Đây hội thảo lần thứ hai theo tinh thần cam kết tỉnh duyên hải Nam Trung Tây Nguyên việc hợp tác để phát triển kinh tế - văn hóa xã hội Sở Giáo dục Đào tạo Phú Yên tiến hành tổ chức Hội thảo lần thứ vào ngày 18-19/4/2011 thành phố Tuy Hòa liên kết bồi dưỡng học sinh giỏi bồi dưỡng học sinh giỏi mơn tốn trường Trung học phổ thông Chuyên tỉnh duyên hải Nam Trung Bộ Tây Nguyên Tại Hội thảo lần thứ thống giao cho Sở Giáo dục đào tạo Khánh Hòa tổ chức Hội thảo lần thứ hai Đây nét sinh hoạt truyền thống sinh hoạt chuyên môn, giao lưu hợp tác giáo dục, đào tạo sinh hoạt học thuật khác Và thực tế, đây, vùng duyên hải Nam Trung Tây Nguyên xuất ngày nhiều nét thành tích bật, có học sinh đạt giải toán Olympic quốc tế Năm nay, nhiều đội tuyển đạt giải cao kỳ thi học sinh giỏi quốc gia Các tỉnh Đắk Lắc, Phú Yên mạnh dạn cử đội tuyển tham dự kỳ thi Olympic Hà Nội mở rộng tiếng Anh đạt giải cao
Khu vực Duyên hải Nam Trung Tây nguyên thực khởi sắc, tạo tiền đề để vươn lên tầm cao mới, chủ động hội nhập, sánh vai ngang với khu vực khác nước
(4)Ban tổ chức nhận 30 báo cáo toàn văn gửi tới hội thảo Song khuôn khổ hạn hẹp thời gian, khâu chế thời lượng kỷ yếu, chúng tơi đưa vào kỷ yếu 22 bài, lại chế để gửi quý đại biểu thực chương trình báo cáo chun đề thức hội thảo
Nội dung kỷ yếu lần phong phú, bao gồm hầu hết chuyên đề phục vụ việc bồi dưỡng học sinh giỏi toán từ đại số, giải tích, hình học, số học đến dạng tốn liên quan khác Bạn đọc tìm thấy nhiều dạng toán từ kỳ olympic nước quốc tế, số dạng toán hàm số, lý thuyết nội suy, cực trị,
Ban tổ chức xin chân thành cảm ơn hợp tác giúp đỡ quý báu quý thầy giáo, giáo đặc biệt tồn thể tổ tốn trường THPT chun Lê Q Đơn Nha Trang, Khánh Hịa để có kỷ yếu với nội dung thiết thực phong phú
Vì thời gian chuẩn bị gấp gáp, nên khâu hiệu đính chế kỷ yếu chưa đầy đủ, chi tiết, chắn chứa nhiều khiếm khuyết Rất mong cảm thông chia sẻ quý đại biểu Những ý kiến đóng góp liên quan đến kỷ yếu xin gửi địa chỉ: Trường THPT Chuyên Lê Quý Đôn, số 67 Yersin, Nha Trang, Khánh Hòa Email: c3lqdon@khanhhoa.edu.vn
Xin trân trọng cảm ơn
(5)NGUYÊN LÝ CỰC HẠN
Trần Nam Dũng, Trường Đại học KHTN Tp HCM
Bài viết phát triển từ viết “Các phương pháp kỹ thuật chứng minh” mà chúng tơi trình bày Hội nghị “Các chun đề Olympic Tốn chọn lọc” Ba Vì, Hà Nội, tháng 5-2010 giảng dạy cho đội tuyển Olympic Việt Nam dự IMO 2010 Trong này, tập trung chi tiết vào ứng dụng Nguyên lý cực hạn giải toán
Một tập hợp hữu hạn số thực ln có phần tử lớn phần tử nhỏ Một tập N ln có phần tử nhỏ Ngun lý đơn giản nhiều trường hợp có ích cho việc chứng minh Hãy xét trường hợp biên! Đó nguyên lý
1 Một số ví dụ mở đầu
Ta xem xét số ví dụ sử dụng nguyên lý cực hạn
Ví dụ Có trường học, trường có n học sinh Mỗi học sinh quen với n+
học sinh từ hai trường khác Chứng minh người ta chọn từ trường bạn cho ba học sinh chọn đôi quen
Lời giải Gọi A học sinh có nhiều bạn trường khác Gọi số bạn nhiều làk Giả sử A trường thứ tập bạn quenA làM ={B1, B2, , Bk} trường thứ Cũng theo giả thiết, có học sinh C trường thứ quen với A Vì C quen khơng q k học sinh trường thứ nên theo giả thiết C quen với n+ 1−k học sinh trường thứ hai, đặtN ={D1, D2, , Dm}là người quen C trường thứ hai
m≤n+ 1−k.VìM, N thuộc tập hợp gồm nhọc sinh |M|+|N| ≥k+n+ 1−k =n+
nên ta có M∩N 6=∅ ChọnB thuộc M ∩N ta có A, B, C đơi quen Ví dụ Chứng minh khơng tồn số n lẻ, n >1 cho 15n+ chia hết cho n
Lời giải Giả sử tồn số nguyên >1sao cho15n+ 1chia hết cho n Gọiplà ước số nguyên tố nhỏ n, p lẻ Giả sử k số nguyên dương nhỏ cho 15k−1
chia hết chop (sốk gọi bậc 15 theo modulo p)
Vì152n−1 = (15n−1)(15n+ 1)chia hết chop.Mặt khác, theo định lý nhỏ Fermat 15p−1−1
chia hết cho p Theo định nghĩa k, suy k ước số số p−1 2n Suy
k|(p−1,2n).Doplà ước số nguyên tố nhỏ củannên (n, p−1) = 1.Suy (p−1,2n) =
Vậy k|2 Từ k = k = Cả hai trường hợp dẫn tới p= Nhưng điều mâu thuẫn 15n+ ln đồng dư 2mod7
Trong hai ví dụ trên, rõ ràng việc xét trường hợp biên đem đến cho thơng tin bổ sung quan trọng Trong ví dụ thứ nhất, việc chọnA học sinh có số người quen nhiều trường khác cho ta thông tin số người quen củaC trường thứ hai n+ 1−k Trong ví dụ thứ hai, dop ước số nguyên tố nhỏ nên p−1nguyên tố với n bội số p
(6)1 Chon điểm xanh vàn điểm đỏ mặt phẳng, khơng có điểm thẳng hàng Chứng minh ta nối2n điểm n đoạn thẳng có đầu mút khác màu cho chúng đôi không giao
2 Trên đường thẳng có 2n+ đoạn thẳng Mỗi đoạn thẳng giao với n đoạn thẳng khác Chứng minh tồn đoạn thẳng giao với tất đoạn thẳng lại
3 Trong mặt phẳng cho n >1 điểm Hai người chơi nối cặp điểm chưa nối véc-tơ với hai chiều Nếu sau nước người tổng véc tơ vẽ người thứ hai thắng; khơng cịn vẽ véc tơ mà tổng chưa có lúc người thứ thắng Hỏi người thắng chơi đúng?
4 Giả sử n số nguyên dương cho 2n+ chia hết chon
a) Chứng minh n >1thì n chia hết cho 3; b) Chứng minh n >3thì n chia hết cho 9;
c) Chứng minh n >9thì n chia hết cho 27 19;
d) Chứng minh n chia hết cho số nguyên tố p6= p≥19;
e)* Chứng minh n chia hết cho số nguyên tố p, p6= vàp6= 19 p≥163
2 Phương pháp phản ví dụ nhỏ nhất
Trong việc chứng minh số tính chất phương pháp phản chứng, ta có thêm số thơng tin bổ sung quan trọng sử dụng phản ví dụ nhỏ Ý tưởng để chứng minh tính chất A cho cấu hình P, ta xét đặc trưng f(P) P hàm có giá trị nguyên dương Bây giả sử tồn cấu hình P khơng có tính chất A, tồn cấu hình P0 khơng có tính chất A với f(P0) nhỏ Ta tìm cách suy điều
mâu thuẫn Lúc này, việc có cấu hình P0 khơng có tính chất A, ta cịn có
cấu hình P với f(P)< f(P0)đều có tính chất A
Ví dụ Cho ngũ giác lồi ABCDE mặt phẳng toạ độ có toạ độ đỉnh nguyên a) Chứng minh tồn điểm nằm nằm cạnh ngũ giác (khác với A, B, C, D, E) có toạ độ nguyên
b) Chứng minh tồn điểm nằm ngũ giác có toạ độ nguyên
c) Các đường chéo ngũ giác lồi cắt tạo ngũ giác lồi nhỏA1B1C1D1E1 bên
Chứng minh tồn điểm nằm biên ngũ giác lồi A1B1C1D1E1
Câu a) giải dễ dàng nhờ ngun lý Dirichlet: Vì có điểm nên tồn điểm X, Y mà cặp toạ độ(x, y)của chúng có tính chẵn lẻ (ta có trường hợp (chẵn, chẵn), (chẵn, lẻ), (lẻ, chẵn) (lẻ, lẻ)) Trung điểm Z XY điểm cần tìm
Sang câu b) lý luận chưa đủ, XY khơng phải đường chéo mà cạnh Z nằm biên Ta xử lý tình sau Để ý XY cạnh, chẳng hạn cạnh AB ZBCDE ngũ giác lồi có đỉnh có toạ độ nguyên ta lặp lại lý luận nêu ngũ giác ZBCDE, Ta dùngđơn biến để chứng minh trình khơng thể kéo dài mãi, đến lúc có ngũ giác có điểm nguyên nằm
(7)các ngũ giác vậy, chọn ngũ giác ABCDE có diện tích nhỏ (phản ví dụ nhỏ nhất) Nếu có nhiều ngũ giác ta chọn số chúng Theo lý luận trình bày câu a), tồn hai đỉnh X, Y có cặp toạ độ tính chẵn lẻ Trung điểm Z XY có toạ độ ngun Vì bên ngũ giác ABCDE khơng có điểm nguyên nên XY phải cạnh Khơng tính tổng qt, giả sử AB Khi ngũ giác ZBCDE có toạ độ đỉnh ngun có diện tích nhỏ diện tích ngũ giác ABCDE Do tính nhỏ ABCDE (phản ví dụ nhỏ phát huy tác dụng!) nên bên ngũ giác ZBCDE có điểm nguyên T Điều mâu thuẫn T nằm ngũ giác ABCDE
Phản ví dụ nhỏ cách tốt để trình bày chứng minh quy nạp (ở thường quy nạp mạnh), để tránh lý luận dài dòng thiếu chặt chẽ
Ví dụ Chứng minh a, b số nguyên dương nguyên tố tồn số nguyênx, y cho ax+by =
Lời giải Giả sử khẳng định đề không đúng, tức tồn hai số nguyên dươnga, bnguyên tố cho không tồn x, y nguyên cho ax+by = Gọi a0, b0 cặp số
như với a0+b0 nhỏ (phản ví dụ nhỏ nhất)
Vì (a0, b0) = (a0, b0) 6= (1,1) (do 1.0 + 1.1 = 1) nên a0 6=b0 Khơng tính tổng quát,
có thể giả sử a0 > b0 Dễ thấy (a0−b0, b0) = (a0, b0) = Do a0˘b0+b0 = a0 < a0+b0 nên
do tính nhỏ phản ví dụ, ta suy ra(a0−b0, b0) khơng phản ví dụ, tức tồn tạix, y
sao cho(a0−b0)x+b0y= Nhưng từ thìa0x+b0(y−x) = Mâu thuẫn điều giả
sử Vậy điều giả sử sai toán chứng minh Bài tập
5 Giải phần c) ví dụ
6 Trên mặt phẳng đánh dấu số điểm Biết điểm chúng đỉnh tứ giác lồi Chứng minh tất điểm đánh dấu đỉnh đa giác lồi
3 Nguyên lý cực hạn bất đẳng thức
Nguyên lý cực hạn thường áp dụng cách hiệu bất đẳng thức có tính tổ hợp, dạng chứng minh tồn k số từ n số thỏa mãn điều kiện
Ví dụ (Moscow MO 1984) Trên vịng trịn người ta xếp số thực khơng âm có tổng Chứng minh tổng tất tích cặp số kề khơng lớn 14
Lời giải Ta cần chứng minh với n≥4 số thực khơng âm a1, , an, có tổng 1, ta có bất đẳng thức
a1a2+a2a3+ +an−1an +ana1≤
1
Với n chẵn (n = 2m) điều chứng minh dễ dàng: đặt a1+a3+ +a2m−1 =a;
đó, rõ ràng,
a1a2+a2a3+ +an−1an+ana1≤(a1+a3+ +a2m−1)×(a2+a4+ +a2m) =a(1−a)≤
(8)Giả sửnlẻ vàak số nhỏ số cho (Để thuận tiện, ta giả sử1< k < n−1- điều khơng làm tính tổng qt khin ≥ 4.) Đặt bi =ai, với i= 1, , k−1, bk =ak+ak+1
vàbi =ai+1với i=k+ 1, , n−1.Áp dụng bất đẳng thức cho số b1, , bn−1,
ta được:
a1a2+ +ak−2ak−1+ (ak−1+ak+2)bk+ak+2ak+3+ +an−1an+ana1≤
1
Cuối cùng, ta sử dụng bất đẳng thức
ak−1ak+akak+1+ak+1ak+2≤ak−1ak +ak−1ak+1+ak+1ak+2 ≤(ak−1+ak+2)bk, để suy điều phải chứng minh
Đánh giá tốt nhất; dấu xảy trongn số 12, cịn số cịn lại
Ví dụ Cho n ≥4 số thực phân biệt a1, a2, , an thoả mãn điều kiện n
X
i=1
ai= 0, n
X
i=1
a2i =
Chứng minh tồn số a, b, c, d thuộc{a1, a2, , an} cho
a+b+c+nabc≤ n
X
i=1
a3i ≤a+b+d+nabd
Lời giải Nếu a ≤ b ≤ c ba số nhỏ với i = 1,2, , n ta có bất đẳng thức
(ai−a)(ai−b)(ai−c)≥0 Suy
a3i ≥(a+b+c)a2i −(ab+bc+ca)ai+abc với i= 1,2, , n Cộng tất bất đẳng thức này, với ý
n
P
i=1
ai = 0, n
P
i=1
a2i = ta n
X
i=1
a3i ≥a+b+c+nabc
Bây chọnd số lớn ta có
(ai−a)(ai−b)(ai−d)≤0
với i= 1,2, , n Và thực tương tự trên, ta suy bất đẳng thức vế phải bất đẳng thức kép cần chứng minh
(9)Lời giải Giả sử 100 số làC1 ≥C2 ≥ ≥C100>0.Nếu nhưC1 ≥100,thì C1+C2+C3 >
100 Do ta giả sử C1 < 100 Khi 100−C1 > 0,100−C2 > 0, C1−C2 ≥
0, C1−C3 ≥0,
100(C1+C2+C3)≥100(C1+C2+C3)−(100−C1)(C1−C3)−(100−C2)(C2−C3)
=C12+C22+C3(300−C1−C2)
> C12+C22+C3(C3+C4+ .+C100)
≥C12+C22+C32+ .+C1002 )>10000
Suy ra, C1+C2+C3>100
Bài tập
7 Trong bảng 2×n ta viết số thực dương cho tổng số cột Chứng minh ta xố cột số cho hàng, tổng số cịn lại khơng vượt q n+14
8 40 tên trộm chia 4000 euro Một nhóm gồm tên trộm gọi nghèo tổng số tiền mà chúng chia không 500 euro Hỏi số nhỏ nhóm trộm nghèo tổng số tất nhóm tên trộm bao nhiêu?
4 Nguyên lý cực hạn phương trình Diophant
Trong phần này, ta trình bày chi tiết ba ví dụ áp dụng nguyên lý cực hạn phương trình Fermat, phương trình Pell phương trình dạng Markov
Ví dụ Chứng minh phương trình x4+y4=z2 (1) khơng có nghiệm ngun dương
Lời giải Giả sử ngược lại, phương trình (1) có nghiệm nguyên dương, (x, y, z) nghiệm (1) vớiz nhỏ
(1) Dễ thấy x2, y2, z đôi nguyên tố
(2) Từ nghiệm phương trình Pythagore, ta có tồn tạip, q cho
x2 = 2pq y2 =p2−q2
z =p2+q2
(3) Từ đây, ta lại có ba Pythagore khác, vìy2+q2 =p2
(4) Như vậy, tồn tạia, bsao cho
q = 2ab y=a2−b2 p=a2+b2 a, bnguyên tố
(5) Kết hợp phương trình này, ta được:
(10)(6) Vì abvà a2+b2 nguyên tố nhau, ta suy chúng số phương (7) Như vậya2+b2 =P2 vàa =u2, b=v2.Suy P2 =u4+v4
(8) Nhưng ta thu điều mâu thuẫn với tính nhỏ z vì:
P2 =a2+b2=p < p2+q2=z < z2
(9) Như điều giả sử ban đầu sai, suy điều phải chứng minh
Phương pháp trình bày gọi phương pháp xuống thang Đây có lẽ phương pháp mà Fermat nghĩ tới viết lề sách Diophant dòng chữ mà sau gọi định lý lớn Fermat làm điên đầu hệ nhà tốn học
Ví dụ Tìm tất cặp đa thức P(x), Q(x) thỏa mãn phương trình
P2(x) = (x2−1)Q2(x) + 1(1)
Lời giải Khơng tính tổng qt, ta cần tìm nghiệm tập đa thức có hệ số khởi đầu dương
Nếu(x+√x2−1)n =P n(x)+
√
x2−1Q
n(x)(2)thì(x− √
x2−1)n =P n(x)−
√
x2−1Q
n(x) (3) Nhân (2) (3) vế theo vế, ta
1 = (x+px2−1)n(x−px2−1)n = (P
n(x) +
p
x2−1Q
n(x))(Pn(x)−
p
x2−1Q
n(x))
=Pn2(x)−(x2−1)Q2n(x)
Suy cặp đa thứcPn(x), Qn(x)xác định (2) (3) nghiệm (1) Ta chứng minh tất nghiệm (1) Thật vậy, giả sử ngược lại, tồn cặp đa thứcP(x), Q(x) khơng có dạngPn(x), Qn(x) thỏa mãn (1) Ta xét cặp đa thức(P, Q)như vớidegQnhỏ Đặt
(P(x) +px2−1Q(x))(x−px2−1) =P∗(x) +px2−1Q∗(x) (4)
Thì rõ ràng
(P(x)−px2−1Q(x))(x+px2−1) =P∗(x)−px2−1Q∗(x)
Suy (P∗, Q∗) nghiệm (1)
Khai triển (4), ta thu P∗(x) = xP(x)−(x2−1)Q(x), Q∗(x) = xQ(x)−P(x) Chú ý từ (1) ta suy (P(x)−xQ(x))(P(x) +xQ(x)) = −Q2(x) + Vì P(x) Q(x) có hệ số khởi đầu > degP = degQ+ nên ta có deg(P(x) +xQ(x)) = degQ+ Từ đây,
deg(−Q2(x) + 1)≤2deg(Q) nên ta suy radeg(Q∗(x))≤deg(Q)−1< degQ
Như vậy, theo cách chọn cặp(P, Q)thì tồn n cho (P∗, Q∗) = (Pn, Qn) Nhưng từ (4) suy
P(x) +px2−1Q(x) = (P∗(x) +px2−1Q∗(x))(x+px2−1)
= (x+px2−1)n(x+px2−1)
= (x+px2−1)n+1
Suy (P, Q) = (Pn+1, Qn+ 1), mâu thuẫn
(11)Ví dụ 10 Tìm tất giá trị k cho phương trình(x+y+z)2=kxyz có nghiệm ngun dương
Lời giải Giả sử k giá trị cần tìm Gọi x0, y0, z0 nghiệm nguyên dương phương
trình
(x+y+z)2=kxyz(1)
cóx0+y0+z0 nhỏ Khơng tính tổng qt, giả sửx0 ≥y0 ≥z0
Viết lại (1) dạng x2−(kyz−2y−2z)x+ (y+z)2 = 0,
ta suy rax0 nghiệm phương trình bậc hai
x2−(ky0z0−2y0−2z0)x+ (y0+z0)2 = 0(2)
Theo định lý Viet x1 = ky0z0 −2y0 −2z0 −x0 = (y0+z0)
2
x0 nghiệm (2) Từ
(x1, y0, z0) nghiệm (1) Cũng từ công thức trên, ta suy x1 nguyên dương Tức
(x1, y0, z0)là nghiệm nguyên dương (1) Từ tính nhỏ x0+y0+z0 tax1 ≥x0 Từ
đây ta có
ky0z0−2y0−2z0−x0≥x0
(y0+x0)2
x0
≥x0
Từ bất đẳng thức thứ hai ta suy ray0+z0 ≥x0 Từ đó, áp dụng vào bất đẳng thức thứ nhất, ta đượcky0z0≥4x0
Cuối cùng, chia hai vế đẳng thứcx20+y02+z02+ 2x0y0+ 2y0z0+ 2z0x0 =kx0y0z0 chox0y0z0,
ta
x0 y0z0
+ y0
x0z0
+ z0
x0y0
+
z0
+
x0
+
y0
=k
Từ suy k4 + + + + + 2≥k, tức k ≤ 323 Suy k≤10
Chú ý x0 = y0 = z0 = suy k = Nếu k 6= x0 ≥ đánh giá trở
thành k4 + + 12 + + + 2≥k suy k ≤ 263, suy k ≤8
Vậy giá trịk = 10 bị loại
Với k= phương trình có nghiệm, chẳng hạn (9, 9, 9) Với k= phương trình có nghiệm, chẳng hạn (4, 4, 8) Với k= phương trình có nghiệm, chẳng hạn (3, 3, 3) Với k= phương trình có nghiệm, chẳng hạn (2, 2, 4) Với k= phương trình có nghiệm, chẳng hạn (1, 4, 5) Với k= phương trình có nghiệm, chẳng hạn (1, 2,3) Với k= phương trình có nghiệm, chẳng hạn (1, 1, 2) Với k= phương trình có nghiệm, chẳng hạn (1, 1, 1)
Ngoài ra, ta chứng minh trường hợp k = phương trình khơng có nghiệm ngun dương (xin dành cho bạn đọc)
Vậy giá trị k cần tìm k = 1,2,3,4,5,6,8,9
Ví dụ 11 (CRUX, Problem 1420) Nếu a, b, c số nguyên dương cho
0< a2+b2−abc≤c
(12)Lời giải Giả sử ngược lại tồn số nguyên dươnga, b, csao cho 0< a2+b2−abc≤c
vàk =a2+b2−abc (1) khơng phải số phương
Bây ta cố định k c xét tập hợp tất cặp số nguyên dương (a, b) thỏa mãn phương trình (1), tức ta xét
S(c, k) = {(a, b)∈(N∗)2 :a2+b2−abc=k}
Giả sử (a, b) cặp số thuộc S(c, k) có a+b nhỏ Khơng tính tổng qt giả sửa≥b Ta xét phương trình
x2−bcx+b2−k =
Ta biết rằngx=alà nghiệm phương trình Gọia1 nghiệm cịn lại phương trình
này thìa1 =bc−a = (b
2−
k)
a
Ta chứng minh (bạn đọc tự chứng minh!) a1 nguyên dương Suy (a1, b)
cũng thuộc S(c, k)
Tiếp theo ta có a1 = (b2−k)/a < a2/a = a, suy a1+b < a+b Điều mâu thuẫn với
cách chọn (a, b) Bài tập
9 Chứng minh phương trình x3+ 3y3 = 9z3 khơng có nghiệm ngun dương 10 Chứng minh phương trình x2+y2+z2 = 2xyz khơng có nghiệm nguyên dương 11 (IMO 88) Nếua, b, q = (a2+b2)/(ab+ 1) số nguyên dương q số phương 12 (PTNK 03) Tìm tất số nguyên dương k cho phương trìnhx2−(k2−4)y2 =−24
có nghiệm nguyên dương
13 (Mathlinks) Cho A tập hợp hữu hạn số nguyên dương Chứng minh tồn tập hợp hữu hạn số nguyên dương B choA⊂B, Q
x∈B
x= Q
x∈B
x2
14.* (AMM 1995) Cho x, y số nguyên dương cho xy+x xy+y số phương Chứng minh có hai số x, y số phương
15 (IMO 2007) Cho a, b số nguyên dương cho 4ab−1 chia hết (4a2−1)2 Chứng minh rằnga =b
16 (VMO 2012) Xét số tự nhiên lẻ a, b mà a ước số củab2+ 2và b ước số a2+
Chứng minh a b số hạng dãy số tự nhiên(vn) xác định
v1 =v2 = = 4vn−1−vn−2 với n≥2
5 Nguyên lý cực hạn tổ hợp
Trên xem xét ví dụ áp dụng nguyên lý cực hạn mảnh đất màu mỡ dành cho nguyên lý cực hạn Nguyên lý cực hạn ứng dụng để chứng minh q trình dừng (trong tốn liên quan đến biến đổi trạng thái) toán đồ thị, hay tình tổ hợp đa dạng khác Các đối tượng thường đem để xét cực hạn thường là: đoạn thẳng ngắn nhất, tam giác có diện tích lớn nhất, góc lớn nhất, đỉnh có bậc lớn nhất, chu trình có độ dài ngắn
Dưới ta xem xét số ví dụ:
(13)Kết luận định lý nghe hiển nhiên chứng minh khơng đơn giản Chứng minh Kelly tham khảo từ Wikipedia
Giả sử phản chứng tồn tập hợp S gồm hữu hạn điểm không thẳng hàng đường thẳng qua hai điểm S chứa ba điểm Một đường thẳng gọi đường nối qua hai điểm S Giả sử (P,l) cặp điểm đường nối có khoảng cách dương nhỏ cặp điểm-đường nối
Theo giả thiết, l qua ba điểm S, nên hạ đường cao từ P xuống l tồn
Hình 1:
tại hai điểm nằm phía đường cao (một điểm nằm chân đường cao) Trong hai điểm này, gọi điểm gần chân đường cao B, điểm C
Xét đường thẳngmnối P vàC.Khoảng cách từ B tới m nhỏ khoảng cách từ P tới l,mâu thuẫn với giả thiết vềP vàl.Một cách để thấy điều tam giác vuông với cạnh huyền BC
đồng dạng nằm bên tam giác vuông với cạnh huyền P C
Do đó, khơng thể tồn khoảng cách dương nhỏ cặp điểm-đường nối Nói cách khác, điểm nằm đường thẳng đường nối chứa ba điểm
Ví dụ 13 Ví dụ 13 (Trận đấu tốn học Nga 2010) Một quốc gia có 210 thành phố Ban đầu thành phố chưa có đường Người ta muốn xây dựng số đường chiều nối thành phố cho: Nếu có đường từ A đến B từ B đến C khơng có đường từ A đến C.Hỏi xây dựng nhiều đường?
Lời giải Gọi A thành phố có nhiều đường (gồm đường xuất phát từ A đường đến A) Ta chia thành phố lại thành loại Loại I - Có đường xuất phát từ A Loại II - Có đường đến A Loại III: Khơng có đường đến A xuất phát từ A Đặt
m=|I|, n=|II|, p=|III|.Ta có m+n+p= 209
(14)Số đường liên quan đến thành phố loại không vượt p(m+n) (Do bậc
A=m+n lớn nhất) Tổng số đường bao gồm:
+ Các đường liên quan đến A:m+n
+ Các đường liên quan đến III : ≤p(m+n)
+ Các đường I II:≤mn
Suy tổng số đường nhỏ mn+ (p+ 1)m+ (p+ 1)n≤(m+n+p+ 1)2/3 = 2102/3 Dấu xảy với đồ thị phe, phe có 70 thành phố, thành phố phe có đường đến thành phố phe 2, thành phố phe có đường đến thành phố phe 3, thành phố phe có đường đến thành phố phe
Ví dụ 14 Trong quốc hội Mỹ, nghị sĩ có khơng q kẻ thù Chứng minh chia quốc hội thành viện cho viện, nghị sĩ có khơng q kẻ thù
Đây ví dụ mà tơi thích Có nhiều cách giải khác trình bày cách giải sử dụng nguyên lý cực hạn Ý tưởng đơn giản có nhiều ứng dụng (trong nhiều tốn phức tạp hơn)
Ta chia quốc hội thành viện A, B cách Với viện A, B, ta gọis(A), s(B) tổng tổng số kẻ thù thành viên tính viện Vì số cách chia hữu hạn nên phải tồn cách chia (A0, B0) cho s(A0) +s(B0) nhỏ Ta chứng minh cách chia
này thỏa mãn yêu cầu toán
Giả sử cách chia chưa thoả mãn yêu cầu, tức có nghị sĩ có nhiều kẻ thù viện Khơng tính tổng quát, giả sử nghị sĩ x thuộcA0
có kẻ thù A0 Khi ta thực phép biến đổi sau: chuyển x từ A0 sang B0
để cách chia làA0 =A0 {x} B0 =B0∪ {x} Vì x có kẻ thù trongA0 A0 khơng cịn chứa x nên ta có
s(A0)≤s(A0)−4(trong tổng s(x) kẻ thù x A0)
Vìx có khơng q kẻ thù có kẻ thù A0 nên x có nhiều kẻ thù
B0 (hay B0),
s(B0)≤s(B0) +
Từ đós(A0) +s(B0)≤s(A0) +s(B0)−2 Mâu thuẫn với tính nhỏ củas(A0) +s(B0).Vậy
điều giả sử sai, tức cách chia(A0, B0)thỏa mãn yêu cầu toán (đpcm) Bài tập
17 Cho 2n điểm mặt phẳng, khơng có điểm thẳng hàng Chứng minh điểm phân thành n cặp cho đoạn thẳng nối chúng không cắt 18 Trong mặt phẳng cho 100 điểm, khơng có ba điểm thẳng hàng Biết ba điểm chúng tạo thành tam giác có diện tích khơng lớn Chứng minh ta phủ tất điểm cho tam giác có diện tích
19 Trên mặt phẳng cho2n+ 3điểm, khơng có ba điểm thẳng hàng khơng có điểm nằm đường trịn Chứng minh ta chọn từ điểm điểm, cho điểm cịn lại có n điểm nằm đường trịn n điểm nằm ngồi đường trịn
20 Trong mặt phẳng cho n điểm ta đánh dấu tất điểm trung điểm đoạn thẳng có đầu mút điểm cho Chứng minh có nhất2n−3 điểm phân biệt đánh dấu
(15)rằng từ thành phố bay đến thành phố khác (có thể nối chuyến) Chứng minh thăm tất thành phố quốc gia sử dụng không a) 198 chuyến bay b) 196 chuyến bay
22* Trong nhóm 12 người từ người ln tìm người đôi quen Chứng minh tìm người đơi quen nhóm
Tài liệu tham khảo
[1] Nguyễn Văn Mậu,Các chun đề Olympic Tốn chọn lọc,Ba Vì , 5-2010
[2] Đoàn Quỳnh chủ biên, Tài liệu giáo khoa chuyên toán - Đại số 10, NXB GD, 2010 [3] http://fermatslasttheorem.blogspot.com/2005/05/fermats-last-theorem-n-4.html [4] vi.wikipedia.org/wiki/Định lý Sylvester-Gallai
(16)MỘT SỐ DẠNG TỔNG QUÁT CỦA PHƯƠNG TRÌNH HÀM PEXIDER VÀ ÁP DỤNG
Trịnh Đào Chiến, Trường Cao Đẳng Sư Phạm Gia Lai Lê Tiến Dũng, Trường THPT Pleiku, Gia Lai
Phương trình hàm Pexider (PTHP) phương trình hàm tổng quát trực tiếp Phương trình hàm Cauchy quen thuộc Bài viết đề cập đến số dạng tổng quát PTHP vài áp dụng chương trình Tốn phổ thơng
1 Phương trình hàm Pexider
PTHP gồm bốn dạng (lời giải xem [1] [2])
Bài tốn 1.1.Tìm tất hàm số f, g, h xác định liên tục R thỏa mãn điều kiện
f(x+y) =g(x) +h(y), ∀x, y ∈R (1)
Giải Nghiệm phương trình (1)
f(t) = ct+a+b, g(t) = ct+a, h(t) =ct+b; a, b, c∈R
Bài tốn 1.2.Tìm tất hàm số f, g, h xác định liên tục R thỏa mãn điều kiện
f(x+y) =g(x)h(y), ∀x, y ∈R (2)
Giải Nghiệm phương trình (2)
f(t) = abect, g(t) = aect, h(t) = bec; a, b, c∈R
hoặc
f ≡0, g ≡0, h∈CR, CR tập hợp hàm số liên tục R,
hoặc
f ≡0, h≡0, g ∈CR
Bài tốn 1.3.Tìm tất hàm số f, g, h xác định liên tục R+ thỏa mãn điều kiện
f(xy) =g(x) +h(y), ∀x, y ∈R+ (3)
Giải Nghiệm phương trình (3)
f(t) = m.lnt+a+b, g(t) = m.lnt+a, h(t) =m.lnt+b; m, a, b, c∈R
Bài tốn 1.4.Tìm tất hàm số f, g, h xác định liên tục R+ thỏa mãn điều kiện
f(xy) = g(x)h(y), ∀x, y ∈R+ (4)
Giải Nghiệm phương trình (4)
(17)2 Một số dạng tổng quát Phương trình hàm Pexider
Tùy theo mức độ kiến thức, PTHP có nhiều dạng tổng quát khác Dưới số dạng tổng quát phương trình (1) gần gũi với chương trình hệ phổ thơng chun Tốn Bài tốn 2.1 Tìm tất hàm số f, fi (i = 1,2, , n) xác định liên tục R thỏa mãn điều kiện
f
n
X
i=1 xi
!
=
n
X
i=1
fi(xi), ∀x, xi∈R (5)
Giải Đây dạng quy nạp cách tự nhiên Bài tốn 1.1 Nghiệm phương trình (5)
f(t) = at+
n
X
i=1
ai, fi(t) =at+ai; a, ∈R
Tương tự Bài tốn 2.1, ta giải phương trình hàm dạng
f
n
X
i=1 aixi
!
=
n
X
i=1
aifi(xi), ∀x, xi ∈R, ∈R
Bài toán sau dạng tổng quát bản, mà phương pháp quy nạp áp dụng lời giải Một số phần chứng minh có sử dụng số kiến thức bản, khơng q khó, Đại số tuyến tính Phương trình vi phân, thuộc chương trình sở Tốn cao cấp
Bài tốn 2.2 Tìm tất hàm số f, fi, gi (i = 1,2, , n) xác định tồn đạo hàm (theo biến số độc lập x, y) R thỏa mãn điều kiện
f(x+y) =
n
X
k=1
fk(x)gk(y), ∀x, y ∈R, n≥2 (6)
Giải Ta giải toán trường hợp n = Trường hợp n ≥3 giải tương tự Xét phương trình hàm
f(x+y) =f1(x)g1(y) +f2(x)g2(y), ∀x, y ∈R, (7)
trong hàmf, f1,f2,g1, g2 xác định tồn đạo hàm (theo biến số độc lập x, y)
trên R
Khơng tính tổng qt, ta ln giả thiết hệ hàm {f1(x), f2(x)}
{g1(x), g2(x)} độc lập tuyến tính Ta có
fx0(x+y) =f10(x)g1(y) +f20(x)g2(y), fy0(x+y) =f1(x)g10 (y) +f2(x)g20 (y)
Vì fx0 (x+y) =fy0(x+y), nên
(18)Ngồi ra, vì{g1(x), g2(x)}là độc lập tuyến tính, nên tồn số y1 , y2 cho
g1(y1) g1(y2) g2(y1) g2(y2)
6
=
Thay y1, y2 vào (8), ta
f10(x)g1(y1) +f20(x)g2(y1) = f1(x)g10 (y1) +f2(x)g02(y1), f10(x)g1(y2) +f20(x)g2(y2) = f1(x)g10 (y2) +f2(x)g02(y2)
Vì định thức nêu khác 0, nên hệ phương trình có nghiệm f10(x), f20(x) Do đó, ta biểu diễnf10(x) f20(x) quaf1(x) f2(x) dạng
f10(x) =a11f1(x) +a12f2(x), f20(x) = a21f1(x) +a22f2(x) (9)
Mặt khác, thay y= vào (7), ta có
f(x) =c1f1(x) +c2f2(x) (10)
Suy
f0(x) = c11f1(x) +c12f2(x), f00(x) = c21f1(x) +c22f2(x) (11)
- Nếu
c11 c12 c21 c22
= 0, từ (10) (11), ta thu phương trình vi phân tuyến tính
a1f0(x) +a2f00(x) = 0,
trong a1 a2 khơng đồng thời Giải phương trình vi phân này, ta tìm f(x)
Tất hàm f(x)này thỏa mãn (7) nên nghiệm phương trình - Nếu
c11 c12 c21 c22
6
= 0, từ (10) (11), ta biểu diễnf1(x)vàf2(x)bởi tổ hợp
tuyến tính f0(x) f00(x) Thay biểu diễn vào (5), ta thu phương trình dạng
f(x) +a1f0(x) +a2f00(x) =
Giải phương trình vi phân này, ta tìm f(x)
Tất hàm f(x) thỏa mãn (7) nên nghiệm phương trình Bài tốn giải
Dưới số trường hợp đặc biệt mà phương trình (7) trở thành số phương trình hàm Những phương trình tiếng có lời giải hồn tồn sơ cấp (có thể xem [1] [2])
- Vớif1(x) = f(x),g1(y)≡1,f2(x)≡1,g2(y) =f(y), phương trình (7) trở thành Phương
trình hàm Cauchy
f(x+y) =f(x) +f(y), ∀x, y ∈R
- Vớif1(x) =g(x),g1(y)≡1,f2(x)≡1,g2(y) = h(y), phương trình (7) trở thành Phương
trình hàm Pexider
(19)- Với f2(x)≡1, phương trình (7) trở thành Phương trình hàm Vincze f(x+y) =f1(x)g1(y) +g2(y), ∀x, y ∈R,
- Với f1(x) = f(x), g1(y) = g(y), f2(x) = g(x), g2(y) = f(y), phương trình (7) trở
thành phương trình hàm dạng lượng giác (vì nghiệm phương trình làf(t) =sint,
g(t) =cost)
f(x+y) = f(x)g(y) +g(x)f(y), ∀x, y ∈R
3 Áp dụng
PTHP tổng quát có nhiều áp dụng việc nghiên cứu số vấn đề liên quan Tốn phổ thơng Sau áp dụng liên quan đến phép chuyển đổi bảo toàn yếu tố góc tam giác
Bài tốn 3.1 Tìm tất hàm số f, g, h xác định liên tục R thỏa mãn điều kiện sau: “NếuA, B, C ∈R, A+B+C =π, thìA1+B1+C1 =π”, đóA1=f(A), B1 =f(B), C1 =f(C)
Giải Giả sử hàm sốf, g,h xác định liên tục R thỏa mãn điều kiện Ta có
A1+B1+C1 =π ⇒f(A) +f(B) +f(C) = π⇒
f(π−B−C) =π−f(B)−f(C) (12) Đặt F (x) =f(π−x), G(x) = π
2 −g(x), H(x) =
π
2 −h(x) Khi đó, phương trình (12) có
dạng
F(B +C) =G(B) +H(C) (13)
Phương trình (13) Phương trình Pexider biết Nghiệm liên tục tổng quát phương trình
F (x) =ax+c1+c2, G(x) = ax+c1, H(x) = ax+c2,
trong a, c1, c2 ∈R
Do
f(x) = a(π−x) +c1+c2, g(x) = π
2 −ax−c1, h(x) =
π
2 −ax−c2 (14)
Đặt a=−k, c1+c2+aπ=λπ, π
2 −c1=µπ,
π
2 −c2 =γπ Thế k+λ+µ+γ = Khi
đó, (14), ta thu
f(x) = kx+λπ, g(x) =kx+µπ, h(x) = kx+γπ,
trong k+λ+µ+γ =
Rõ ràng hàm số f, g,h nêu thỏa mãn điều kiện toán
(20)A1=f(A), B1 =f(B), C1 =f(C)
Giải Tương tự cách giải trên, ta tìm
f(x) = kx+λπ, g(x) =kx+µπ, h(x) = kx+γπ,
trong λ≥0, µ≥0, k+λ≥0, k+µ≥0, λ+µ≤1, k+λ+µ≤1
Kết Bài tốn 3.2 có nhiều áp dụng phép chuyển đổi bảo tồn yếu tố góc tam giác, chẳng hạn Hệ sau mà phần chứng minh dành cho bạn đọc
Hệ 3.1 NếuA, B, C ba góc tam giác, A1, B1, C1 xác định sau A1= B+C
2 , B1 =
C+A
2 , C1 =
A+B
2
cũng ba góc tam giác
Hệ 3.2 Nếu A, B, C ba góc tam giác thỏa mãn max{A, B, C} < π
2, tức
tam giác ABC nhọn, A1, B1, C1 xác định sau
A1 =π−2A, B1 =π−2B, C1 =π−2C
cũng ba góc tam giác
Hệ 3.3 NếuA, B, C ba góc tam giác, A2, B2, C2 xác định sau
A2= A
2, B2=
B
2, C2 =
π+C
2
cũng ba góc tam giác, C2 góc tù
Hệ 3.4 Nếu A, B, C ba góc tam giác, C góc tù, A2, B2, C2
xác định sau
A2= 2A, B2 = 2B, C2= 2π−C
cũng ba góc tam giác
Hệ 3.5 Nếu tam giác ABC có ba góc nhọn (hoặc vng C), A3, B3, C3 xác định
như sau
A3 = π
2 −A, B3=
π
2 −B, C3=π−C,
cũng ba góc tam giác tù (hoặc vng C3)
Hệ 3.6 Nếu tam giác ABC có góc C tù (hoặc vng), A3, B3, C3 xác định sau A3 =
π
2 −A, B3=
π
2 −B, C3=π−C,
cũng ba góc tam giác nhọn (hoặc vng C3)
Bây giờ, mở rộng cách tự nhiên tốn trên, ta có tốn sau
Bài tốn 3.3 Tìm tất hàm số f, g, h xác định liên tục R thỏa mãn điều kiện sau: “Nếu Ai∈R,
n
P
i=1
Ai = (n−2)π, n
P
i=1
A0i = (n−2)π”, A0i=f(Ai)
Giải Tương tự cách giải Bài toán 3.1, phương trình hàm cảm sinh Phương trình hàm Perxider tổng quát Các hàm số tìm
(21)trong n
P
j=0
kj =
Tương tự, mở rộng Bài toán 3.2, ta thu
Bài toán 3.4 Tìm tất hàm số fi (i= 1, , n, n ≥3) xác định liên tục R thỏa mãn điều kiện sau: “Nếu0≤Ai ≤2π,
n
P
i=1
Ai= (n−2)π, 0≤A0i ≤2π, n
P
i=1
A0i = (n−2)π”, A0i =f(Ai)
Giải Tương tự cách giải Bài toán 3.2, phương trình hàm cảm sinh Phương trình hàm Perxider tổng quát Các hàm số tìm
fi(x) = k0x+ki(n−2)π (i= 1, , n, n≥3), 0≤ki(n−2)≤2,0≤2k0+ki(n−2)≤2
Thu hẹp giả thiết Bài toán 3.4, ta thu
Bài toán 3.5 Tìm tất hàm số fi (i= 1, , n, n ≥3) xác định liên tục R thỏa mãn điều kiện sau: “Nếu ≤ Ai ≤ π,
n
P
i=1
Ai = (n−2)π, ≤ A0i ≤ π, n
P
i=1
A0i = (n−2)π”, A0i =f(Ai)
Giải Tương tự cách giải Bài toán 3.4, phương trình hàm cảm sinh Phương trình hàm Perxider tổng quát Các hàm số tìm
fi(x) = k0x+ki(n−2)π (i= 1, , n, n≥3), 0≤ki(n−2)≤1,0≤k0+ki(n−2)≤1
Từ kết ta thấy rằng, với ba góc tam giác cho trước, tạo ba góc tam giác suy nhiều hệ thức lượng giác liên quan đến góc tam giác Hơn nữa, cách phối hợp phương pháp khác nhau, ta cịn tạo nhiều đẳng thức bất đẳng thức lượng giác khác, vô phong phú Sau vài ví dụ
Giả sử rằng, ta chứng minh hệ thức sau xem chúng hệ thức "gốc" ban đầu
sinA+ sinB+ sinC ≤ √
3
2 , (15)
cosAcosBcosC ≤
8 (16)
0<sinAsinBsinC ≤ √
3
8 , (17)
sin 2A+ sin 2B+ sin 2C = sinAsinBsinC (18) Áp dụng Hệ 3.1 vào (15), ta có
sin
π−A
2
+ sin
π−B
2
+ sin
π−C
2
≤ √
3
Như vậy, ta tạo bất đẳng thức sau Bất đẳng thức cosA
2 + cos
B
2 + cos
C
2 ≤ 3√3
(22)Áp dụng Hệ 3.1 vào (16), ta có
cos
π−A
2
cos
π−B
2
cos
π−C
2
≤
8
Như vậy, ta tạo bất đẳng thức sau Bất đẳng thức sinA
2 sin B sin C ≤
Áp dụng Hệ 3.1 vào (18), ta có
sin
π−A
2
+ sin
π−B
2
+ sin
π−C
2
= sinπ−A sin
π−B
2 sin
π−C
2
hay
sin (π−A) + sin (π−B) + sin (π−C) = sinπ−A sin
π−B
2 sin
π−C
2
Như vậy, ta tạo đẳng thức sau
Đẳng thức sinA+ sinB+ sinC = 4cosA 2cos
B
2cos
C
2
Bây giờ, để sáng tác thêm hệ thức đa dạng hơn, ta tiếp tục khai thác kết trên, chẳng hạn từ Bất đẳng thức ta có
8 sin A sin
B
2 sin
C
2 ≤1⇔32 sin
A
2 sin
B
2 sin
C
2.cos
A
2cos
B
2cos
C
2 ≤4cos
A 2cos B 2cos C ⇔4
2 sinA 2cos
A
2 sin
B
2cos
B
2 sin
C
2cos
C
2
≤4cosA 2cos
B
2cos
C
2
⇔4 sinAsinBsinC ≤4cosA 2cos
B
2cos
C
2 (19)
Như vậy, ta tạo bất đẳng thức sau Bất đẳng thức sinAsinBsinC ≤cosA
2cos
B
2cos
C
2
Bởi (18) Đẳng thức 1, từ (19), ta có bất đẳng thức sau
Bất đẳng thức sin 2A+ sin 2B+ sin 2C ≤sinA+ sinB+ sinC
Ta tiếp tục khai thác Bất đẳng thức Nhận xét rằng, tam giácABC tam giác nhọn thì, áp dụng Hệ 3.2 vào Bất đẳng thức 4, ta có
sin (π−2A) + sin (π−2B) + sin (π−2C)
≤sin (π−2A) + sin (π−2B) + sin (π−2C)
⇔ −sin 4A−sin 4B−sin 4C ≤sin 2A+ sin 2B+ sin 2C
Như vậy, ta tiếp tục tạo bất đẳng thức sau
Bất đẳng thức sin 2A+ sin 2B+ sin 2C+ sin 4A+ sin 4B+ sin 4C ≥0 Bây giờ, áp dụng Hệ 3.3 vào Bất đẳng thức 4, ta có
sin
2.A
2
+ sin
2.B
2
+ sin
2.π+C
2
≤sinA + sin
B
2 + sin
π+C
(23)Ta tạo bất đẳng thức sau
Bất đẳng thức sinA+ sinB−sinC ≤sinA + sin
B
2 + cos
C
2
Bây giờ, giả sử tam giác ABC có gócC tù Áp dụng Hệ 3.4 vào Bất đẳng thức 1, ta có
cos2A + cos
2B
2 + cos
2C−π
2 ≤ 3√3
2
Ta tạo bất đẳng thức sau
Bất đẳng thức cosA+ cosB+ sinC ≤ √
3
C > π
2
Tiếp theo, giả sử tam giác ABC nhọn (hoặc vuông C) Áp dụng Hệ 3.5 vào (17), ta có
0<sinπ −A
sinπ −B
sin (π−C)≤ √
3
Ta bất đẳng thức sau
Bất đẳng thức 0<cosAcosBsinC≤ √
3
C≤ π
2
Bây giờ, giả sử tam giác ABC có góc C tù (hoặc vuông) Áp dụng Hệ 3.6 vào (15), ta có
0<sin
π
2 −A
+ sin
π
2 −B
+ sin (π−C)≤ √
3
Ta bất đẳng thức sau
Bất đẳng thức 0<cosA+ cosB + sinC ≤ √
3
C ≥ π
2
TÀI LIỆU THAM KHẢO
[1] J Aczél (1966), Lectures on Functional equations and their applications, Chapter 3, pp 141-145, Chapter 4, pp 197-199
[2] Nguyễn Văn Mậu, Một số lớp phương trình hàm đa ẩn hàm dạng bản, Kỷ yếu Hội thảo khoa học "Các chuyên đề chuyên Toán bồi dưỡng học sinh giỏi Trung học phổ thông", Hà Nội, 2011
(24)XÂY DỰNG MỘT LỚP PHƯƠNG TRÌNH HÀM NHỜ CÁC HẰNG ĐẲNG THỨC LƯỢNG GIÁC
Lê Sáng, Trường THPT Chun Lê Q Đơn - Khánh Hịa
Trong kì thi Đại học câu hỏi phương trình, bất phương trình thường ý,thì câu hỏi đề thi chọn học sinh giỏi quốc gia hay quốc tế tốn phương trình hàm chiếm phần trọng tâm Trong viết thử liên hệ kiến thức lượng giác học chương trình phổ thơng để đưa đến số tốn có nghiệm hàm số lượng giác
1 Các hàm số lượng giác chương trình tốn vài
tính chất
sin(x±y) = sinxcosy±sinycosx, ∀x, y ∈R(1)
sin(x+y) sin(x−y) = sin2xcos2y−sin2ycos2x, ∀x, y ∈R(2) cos(x±y) = cosxcosy∓sinxsiny, ∀x, y ∈R(3)
Từ (2) đưa đến cơng thức phương trình hàm ẩn hàm sin
g(x+y)g(x−y) = g2(x)−g2(y) với mọix, y ∈R
Từ (3) ta đạt công thức hàm cosin (phương trình hàm d’Alembert )
f(x+y) +f(x−y) = 2f(x)f(y)với x, y ∈R
Ngoài từ số công thức lượng giác mà ta đoán nghiệm
f(2x) = 2f2(x)−1, f(3x) = 4f3(x)−3f(x),∀x∈R
Quy ước: fn(x) = [f(x)]n
Bốn Phương trình hàm : Trong toán sau phần nhiều trước đến kết thường phải qua trung gian phương trình hàm sau
Các phương trình Cauchy
A(x+y) = A(x) +A(y) (I)
E(x+y) = E(x).E(y) (II)
L(xy) = L(x) +L(y) với x >0 (III)
F(xy) = F(x).F(y) với x >0 (IV)
Ta có nghiệm A(x)=ax ,với a=f(1) giải A.L.cauchy 1821
E(x) =exp(ax) hay E(x) =
L(x) =alnx hay L(x) =
(25)2 Phương trình hàm d’Alembert – Hàm cosin
Bài tốn Tìm hàm f(x) xác định liên tục thỏa mãn điều kiện
f(x+y) +f(x−y) = 2f(x)f(y), ∀x, y ∈R
f(0) = 1,∃x0∈R:|f(x0)|<1
Lời giải Vì f(0) = 1và f(x)liên tục R nên ∃ε >0sao cho f(x)>0, ∀x∈(−ε, ε)
Khi theo (2) với n0 ∈N đủ lớn
f(x0
2n0)>0⇒f(
x0
2n0)<1 (do phản chứng )
Vậy tồn tạix1 6= 0, x1 = 2xn00 cho
0< f(x1)<1.f(x)>0, ∀x∈(− |x1|,|x1|), f(x1) = cosα,0< α < π
2
Từ (1) suy
f(2x1) = 2f2(x1)−1 = 2cos2α−1 = cos 2α
Giả sửf(kx1) = coskα, ∀k= 1,2, , n∈N+ Khi
f((n+ 1)x1) = f(nx1+x1)
= 2f(nx1)f(x1)−f((n−1)x1) = cosnαcosα−cos (n−1)α
= cos (n+ 1)α
Từ suy raf(mx1) = cosmα, ∀m∈N+ f(x) hàm chẵn R
f(mx1= cosmα,∀m∈Z(3)
Do tính trù mật R ,f(x)và cosx hàm liên tục R nên f(x) = cosax, a ∈R∗
Thử lại ta thấyf(x) = cosax(a6= 0) thỏa mãn điều kiện toán
Nhận xét Thay giả thiết |f(x0)| <1 toán bởi|f(x0)| >1 ta có nghiệm tốn f(x) = cosh(x), hàm cosin hyprebol mà ta không khảo sát chương trình học phổ thơng
Nhận xét Khi f hàm khả vi, lấy đạo hàm theo y hai lần , ta f0(0) = 0, f00(x) =
k.f(x), k Nếuk = f(x) = ax+b;
Nếuk >0 f(x) =csinbx+dcosbx, c, d
Từ f(0) = 1, f00(0) = suy d= 1, bc= 0, b= f hằng; c= f(x) = cosx
Nếuk <0 f(x) =csinhbx+dcoshbx với b2=k,điều kiện f(0) = 1, f00(0) =
Vậy nghiệm f(x) = cosbx, bsố thực
Định lý (Định lý nghiệm Phương trình d’Alembert) Cho f :R→R hàm liên tục thỏa mãn điều kiện
(C)f(x+y) +f(x−y) = 2f(x)f(y), với x, y
(26)Bài toán (IMO1972) Tìm f :R→R liên tục thỏa mãn điều kiện
f(x+y) +f(x−y) = 2f(x)f(y), với x, y
Chứng minh f(x)6= 0,|f(x)| ≤1, ∀x∈R |g(y)| ≤1, ∀y∈R
Lời giải Do f bị chặn
f(x)6= 0|f(x)| |g(y)|= 2|f(x+y) +f(x−y)| ≤ |f(x+y)|+|f(x−y)| ≤2M, ∀x∈R
Bài tốn Tìm tất hàm liên tục D→R thỏa
f(x+y) = f(x) + f(y) 1−f(x)f(y)
Lời giải Ta biết hàm f(x) = tanx thỏa đề
Vì đặtA(x) =arctanf(x)thì A(x+y) = A(x) +A(y)±2k Suy A(x) =kx mod 2π , từ dẫn đến f(x) = tankcx
Bài tốn Tìm hàm f(x) xác định liên tục R thỏa mãn điều kiện
f(x) +f(y) =f
x+y
1−xy
, ∀x, y ∈:x+y >0
Lời giải Đặt x=cotgu, y=cotgv,0< u, v < π
Thì 1x−+xyy =cotg(u+v)
HayA(u+v) =A(u) +A(v),0< u, v < π A(u) = f(cotgu) f(x) = karcctgx, k∈, ∀x∈R (i)
Thử lại ta thấy hàmf(x)xác định theo (i) thỏa mãn điều kiện toán Kết luận f(x) = karcctgx, k∈R, ∀x∈R
Bài tốn Tìm cặp hàm f(x)vàg(x) xác định liên tục R thỏa mãn điều kiện
f(x−y) =f(x)g(y) +f(y)g(x)
Lời giải Nghiệm toán
f(x) =c
g(x) = ±√1−c2 hay
f(x) = coskx
(27)Bài toán (Putnam1991)
Cho hai hàm f :R→R, g :R→R, f(x), g(x)khác hằng, khả vi thỏa mãn điều kiện
f(x+y) =f(x)f(y)−g(x)g(y)
g(x+y) = f(x)g(y) +g(x)f(y),∀x, y ∈R vàf0(0) = 0.Chứng minh f2(x) +g2(x) =
Lời giải Ta cần chứng minh H(x) =f2(x) +g2(x) Thật lấy đạo hàm theo y thayy=
ta đượcf0(x) = −g0(0)g(x)vàg0(x) =g0(0)f(x)
Do đó2f(x)f0(x) + 2g(x)g0(x) = suy f2(x) +g2(x) =C
Ngoài raf2(x+y) +g2(x+y) = (f2(x) +g2(x))(f2(y) +g2(y)) ,nên C2 =C Nhưng C6= 0,nên C =
Nhận xét.Từ giả thiết toán ta thấy hai hàmf(x) = cosx, g(x) = sinx nghiệm toán, nên ta đặt E(x) = f(x) +ig(x) ,từ giả thiết tốn ta có E(x+y) =E(x)E(y)
Vì ta có cách giải khác sau
Do E hàm khả vi nên E0(0) =ib, E(x+y) =E(x)E(y) Lấy đạo hàmy ,rồi cho y = 0ta
E(x) = Ceibx,
Từ E(0 + 0) =E(0)E(0), ta rút C=
Cuối cùngf2(x) +g2(x) =|E(x)|=eibx
2
=
Bài toán a.Tìm cặp hàm f :R→R, g:R→R liên tục thỏa mãn điều kiện
f(x−y) = f(x)g(y)−f(y)g(x)
g(x−y) =g(x)g(y) +f(x)f(y), ∀x, y ∈R
Đáp số Nghiệm toán f(x) = sinbx, g(x) = cosbx, b số thực hay f(x) =g(x) =
b
f(x+y) = f(x)g(y) +f(y)g(x)
g(x+y) =g(x)g(y)−f(x)f(y), ∀x, y ∈R
Đáp số Nghiệm f(x) = ax/2sinbx, g(x) = ax/2cosbx, a > 0, bsố thực hay f(x) =
g(x) =
Bài tốn Tìm cặp hàm f(x) g(x) xác định liên tục R thỏa mãn
[f(x) +g(x)]2 = +f(2x), f(0) =
f(x+y) +f(x−y) = 2f(x)g(y),∀x, y ∈R
Đáp số Nghiệm toán làf(x) = sinbx, g(x) = cosbx, bsố thực hay f(x) =g(x) =
3 Phương trình hàm có chứa hàm số lượng giác
3.1 Trước hết xét hàm thực f xác định với x,y thuộc R thỏa
(28)Kết f(x) nghiệm 4.1 f(x) = cosx
Kết Phương trìnhf(x+y) +f(x−y) = sinxsiny , khơng có nghiệm Kết Phương trìnhf(x+y) +g(x−y) = sinxsiny
có nghiệmf(x) =c−cosx, g(x) = cosx−c
Do công thức biến đổi sinxsiny= cos(x−y)−cos(x+y) suy f(u) + cosu= cosv−g(v)
Kết Phương trìnhf(x+y)−f(x−y) = sinxsiny có nghiệm
f(x) =c−cosx, clà
Kết Phương trìnhf(x+y) +f(x−y) = cosxsiny khơng có nghiệm Kết Phương trìnhf(x+y) +g(x−y) = cosxsiny,
có nghiệmf(x) =c+ sinx, g(x) =−sinx−c
Kết Phương trìnhf(x+y) +f(x−y) = sinxcosy ,có nghiệm f(x) = sinx
3.2Xét hai hàm thực f ,g xác định R thỏa
f(x+y) +g(x−y) = sinxcosy, (3.2)
Kết Nghiệm (3.2) làf(x) =c+sinx, g(x) =sinx−c
Cho y = 0, thayy=−y vào (3.2)
ta đượcf(x) +g(x) = sinx f(x+y) +g(x−y)−f(x−y) +g(x+y) =
3.3Cho hàm thực f(x)thỏa f(x+y) +f(x−y) = 2f(x) cosy (3.3)
Khi hàmg(x) =f(x)−dcosx thỏa g(x+y) =g(x) cosy+g(y) cosx (3.4)
Kết Nghiệm tổng quát (3.4) g(x) =bsinx, f(x) =bsinx+dcosx
Thật vậy, cho x = 0, hoán đổi x, y (3.3) ta f(y) +f(−y) = 2dcosy 2f(x+
y) +f(x−y) +f(y−x) = 2f(x) cosy+ 2f(y) cosx
Tức là2f(x+y) + 2dcos(x−y) = 2f(x) cosy+ 2f(y) cosx Suy (3.4) Để có nghiệm (3.4) , dùng tính kết hợp hàm g(x)
g(x+y+z) = (g(x) cosy+g(y)) cosx+g(z) cos(x+y) =g(x) cos(y+z) + (g(y) cosz+g(z) cosy) cosx
Suy g(x) sinysinz =g(z) sinysinx, x, y, z
Cố định y, z với sinz6= 0, ta g(x) = bsinx
3.4 Cho hai hàm f :R→R, g :R→R thỏa
f(x−y)−f(x+y) = 2g(x) siny, với số thựcx, y(3.5)
khi f(x) = acosx−dsinx+c, g(x) =asinx+dcosx
Lời giải Cho x= 0, hoán đổi x, y (3.5) ta đượcf(−y)−f(y) = 2dsiny
f(x−y)−f(y−x) = 2g(x) siny−2g(y) sinx
Tức là−2dsin(x−y) = 2g(x) siny−2g(y) sinx
Suy (g(x)−dcosx) siny= (g(y)−dcosy) sinx,tức g(x) =dcosx+asinx (cố định y) Thayg(x) vào (3.5) ta có kết
3.5.Lập luận tương tự ta có vài phương trình hàm
sin (x+y) = f(x) siny+f(y) sinx(3.6) sin (x+y) = g(x) siny+g(y) sinx(3.7)
(29)Kết 10 Nghiệm (3.6 ) f(x) = cosx ,nghiệm (3.7 ) f(x) = cosx+dsinx, g(x) = cosx−bsinx , nghiệm của(4.8) f(x) = acosx=g(x)
3.6 Dùng tính kết hợp hàm
Xétf(x+y)f(x−y) = sin2(x)−sin2(y),(3.9)
Kết 11 (4.9) có nghiệm làf(x) = ksinx, k2=
Bài tốn Nghiệm f(x+y)f(x−y) = f2(x)−sin2(y),(3.10)
Làf(x) =asinx hay f(x) = bcosx+dsinx, a, b, d thỏa a2 = =b2+d2
Lời giải Cho x=y (4.10).Xét trường hợpf(0) = hay khác không Trường hợp khác không ta đưa dạngf(u)f(v) =bf(u+v) + sinusinv
Sau xét bf(u+v+z) =f(u)(f(v)f(z)−sinvsinz)−sinu(sinvcosz+ cosvsinz)
Tính chất Cho hàm f(x)xác định liên tục R thỏa mãn điều kiện
(S∗)f(x+y+ 2d) +f(x−y+ 2d) = 2f(x)f(y), với x, y, d khác khơng f hàm số lẽ
Tính chất
(i) Nếu f(0) = f(d) = hàm f có chu kỳ d
(ii) Nếuf(0) = f(d) = −1 hàm f có chu kỳ là2d
(iii) Nếuf(0) =−1thì f(d) = hàm f có chu kỳ làd
Bài tốn 10 Tìm hàm f(x) thỏa mãn (S*)
Lời giải Trường hợp (i) (ii) f thỏa phương trình d’Alembert hàm (C) Trường hợp (iii) thìg(x) =f(x+ 2d) nghiệm phương trình (C)
g(x+y) +g(x−y) = 2g(x)g(y), g(x) có chu kỳ 4d
Kết Nghiệm toán (S*) làf(x) = cos2nπxd hayf(x) = ±cos(2n+1)d πx
4 Phương trình hàm sin (S)f(x+y)f(x−y) = f2(x)−f2(y),
với mọi x, y
Tính chất Hàmf khác khơng, thỏa (S) hàm lẽ Bài toán 11 Cho f :R→R liên tục thỏa (S)
Thì f(x) = cx , f(x) = csinbxhay f(x) = sinhbx
Lời giải Do f liên tục, khả vi Lấy đạo hàm lần thứ theoy, lần thứ hai theox
Suy f00(x) = kf(x) Như ta có kết
Bài toán 12 (Corovei) Cho hai hàm f :R→R, g :R→R khác không thỏa
f(x)g(y−x) =f
y
2
g
y
2
−f
x−y
2
g
x−y
2
,∀x, y ∈R(∗)
Khi g nghiệm (S)
g(x) = A(x), f(x) =c+dA(x)(1), g(x) = b(E(x)−E(−x)), f(x) = c(E(x)−E(−x))+dE(−x)(2)
(30)Lời giải Thay x x+y, y 2x vào (*) f(x+y)g(x−y) = f(x)g(x)−f(y)g(y)(3),
sau đổi chổx y với ta f(x+y)g(y−x) = −f(x+y)g(x−y),
suy g(x)hàm lẽ ( f khác không)
Lấyy=−x thay y −y (1) , trừ (*) cho (3)
f(x−y)g(x+y)−f(x+y)g(x−y) =g(y)(f(−y) +f(y))
Xét trường hợp f(0), f(0) = 0,thì f vàg (1) nghiệm; f(0) khác không, f g (2) nghiệm
Kêt Trường hợp hàm liên tục, nghiệm khác không
f(x) = bx+c, g(x) = ax;
f(x) = csinax+dcosax, g(x) = bsinax;
f(x) = bsinhax+dcoshax, g(x) =bsinhax
5 Mở rộng phương trình hàm d’Alembert dạng lượng
giác W H Wilson1919
f(x+y) +f(x−y) = 2f(x)g(y),(1)với x, y f(x+y) +g(x−y) = h(x)k(y),(2) với x, y
Nhận xét khif(x) = 0thìg(x)là hàm tùy ý Nên ta xétf(x)6= 0,nên cóasao chof(a)6=
Trong (1) thay x =a, y = −y ta g(x) hàm số chẳn Nhờ phương pháp tách f thành hàm chẳn f1 hàm lẽ f2 Wilson thu gọn f1(x) = kg(x),với k = f1(0) g thỏa mãn hàm d’Alembert
Định lý (Định lý Wilson.) Nghiệm tổng quát phương trình(1)
f(x) = 0, g(x) tùy ý, hay
f(x) = kcosbx+Csinbx g(x) = cosbx,hay
f(x) = kcoshbx+Csinhbx g(x) = coshbx, hay
f(x) = k+Cx g(x) =
Nhận xét.Trong phạm vi nhằm xét phương trình (1) có nghiệm dạng lương giác, dạng (2) xét tổng quát khác
Bài toán 13 Nếu hàm f thỏa f(x−y) =f(x)f(y) +g(x)g(y),∀x, y ∈R.thì thỏa phương trình d’Alembert f(x+y) +f(x−y) = 2f(x) cosy,∀x, y ∈R
Lời giải Do toán 3.5
Bài tốn 14 ( Bình Định 2009 )Tìm tất hàm số f xác định trênR thỏa
(31)6 Mở rộng phương trình hàm dạng f[G(x, y)] = F(f(x), f(y))
Trong viết này, xét trường hợp đặc biệt G(x,y)=x+y, tức dạng phương trình hàmf(x+y) =F(f(x), f(y)), x, y ∈R
Tính chất F có tính kết hợp tức F[F[u, v], w] =F[u, F[v, w]] =f(x+y+w)
Trường hợp
F(u, v) =Auv+Bu+Bv+C, F đa thức đối xứng, tính kết hợp ta AC =B2−B
Khi A= B = tốn có dạngf(x+y) =f(x) +f(y) +C
ĐặtA(x) = f(x) +C,bài toán đưa A(x+y) = A(x) +A(y) (phương trình Cauchy I) Do đóf(x) = A(x)−C.Khi A6=0 , tốn có dạng
f(x+y) = Af(x)f(y) +Bf(x) +Bf(y) + B
2−B
A =
[Af(x) +B] [Af(y) +B]−B A
ĐặtE(x) =Af(x) +B, toán đưa E(x+y) =E(x).E(y)(phương trình Cauchy II) Do đóf(x) = E(xA)−B
Một số toán liên quan
Các nghiệm tổng quát đưa lớp hàm lượng giác xác định khoảng để hàm số liên tục đơn điệu khoảng xác định
6.1 f1(x+y) = f1(x) +f1(y) 1−f1(x)f1(y)
,
6.2 f2(x+y) =
f2(x)f2(y)−1 f2(x) +f2(y)
,
6.3 f3(x+y) =
f3(x) +f3(y)−2f3(x)f3(y)
1−f3(x)f3(y) , 6.4 f4(x+y) = f4(x) +f4(y)−1
2f4(x) + 2f4(y)−2f4(x)f4(y)−1 ,
6.5 f5(x+y) =
f5(x) +f5(y)−2f5(x)f5(y) cosa
1−f5(x)f5(y)
,
6.6 f6(x+y) =f6(x)f6(y)−p1−f2(x)p1−f2(y)
Có nghiệm
f1(x) = tankx, f2(x) = cotkx, f3(x)
=
1 + cotkx, f4(x) =
1
1 + tankx, f5
(x) = sinkx
(32)7 Lượng giác hóa tốn phương trình hàm
Bài tốn 15 (Putnam2000) Tìm hàmf(x)xác định liên tục [−1,1]và thỏa mãn
f 2x2−1= 2xf(x), ∀x∈[−1.1]
Lời giải Ta có x= 1, x=−12 thỏa mãn 2x2−1 =x suy raf(1) =f(−12) =
Nênf cos 23π + 2πn= 0.Từ f(cos 2a) = ta suy raf(cosa) =
Đặtx= cosa, Ta f cos 2−k 23π + 2πn= 0,∀n∈Z, k ∈N
Hơn nữa, tập số2−k(23π+ 2πn)trù mật trongRvà f(cosx)liên tục suy raf(cosr) = 0,mọi r
Bài toán 16 Tìm hàm f(x),chẳn, liên tục lân cận điểm O xác định f(x)>0 06x6π/2,f(x)60 π/26x6π,
và thỏa (*) f(2x) = 1−2f2 π2 - x,∀x∈R, f(x) = cosx, x
Bài tốn 17 Tìm hàm f(x) xác định liên tục [−1,1] thỏa mãn điều kiện
f(x) +f(y) = fxp1−y2+yp1−x2, ∀x, y ∈[−1,1] (18)
Lời giải Đặt x= sinu, y = sinv, ∀u, v ∈−2π,π2 Thì xp1−y2+y√1−x2 = sin (u+v)
Khi viết (18) dạng
g(u+v) =g(u) +g(v), u, v∈h−π
2 ,
π
2
i
vớig(u) = f(sinu)
Suy f(x) =aarcsinx, ∀x∈[−1,1], a∈R (i)
Thử lại ta thấy hàmf(x)xác định theo ( i ) thỏa mãn điều kiện toán
Bài toán 18 Tìm hàm số f(x) xác định liên tục [−1,1] thỏa mãn điều kiện
f
xy−p1−y2p1−x2=f(x) +f(y), ∀x, y ∈[−1,1]
Lời giải Đặt x= cosu, y = cosv, ∀u, v ∈[0, π] Khi đósinu>0,sinv >0
xy−p1−y2p1−x2 = cos (u+v), ∀u, v ∈[0, π]
Phương trình hàm cho viết dạng
f(cosu) +f(cosv) =f(cos (u+v)), ∀u, v ∈[0, π]
Đặtf(cosu) = g(u)ta
g(u+v) =g(u) +g(v), ∀u, v ∈[0, π]
Do vậy, g(u) =au, a=const , f(x) = aarccos x
(33)Bài tốn 19 Tìm hàm f(x) xác định liên tục R thỏa mãn điều kiện f(x) + f(y) = f
x + y - xy
, ∀x,y∈R:|xy|<1(17)
Lời giải Đặt x=tgu, y=tgv,−2π < u, v < π2 Do |xy|<1nên ta có 1x−+xyy =tg(u+v)
Vậy −2π < u+v < π2 Khi A(u+v) = A(u) +A(v) ,trong A(u) = f(tgu),−2π < u, v < π2
Vậyf(x) = aarctgx, a∈R, ∀x∈R (ii)Thử lại ta thấy hàmf(x)xác định theo (ii) thỏa mãn điều kiện toán Kết luận f(x) =aarctgx, a∈R, ∀x∈R
Bài tốn 20 Tìm tất hàm số f(x) liên tục R
f(x)f(y) - f(x + y) = sin x sin y,∀x,y∈R(∗)
Lời giải Với x=y= 0,ta thu được: f2(0)−f(0) = Suy f(0) = hay f(0) = -Nếuf(0) = 0thì với y= ta có −f(x) = Từ đóf(x)≡0, ∀x
Nhưng thay x=y = π2 ta thấy mâu thuẫn -Nếuf(0) = 1thay y=−x ta có
f(x)f(−x) = 1−sin2x=cos2x Thayx= π2 ta có f π2f −π2=
+Nếu f π2= 0, thayy= π2 , Ta có f x+ π2=−sinx= cos x+π2
+Nếu f −π2= thay y=−π2,
fx− π
2
=−sinx= cosx− π
2
Vậy f(x) = cosx Thử lại thấy
Bài toán 21 (Russia 2000): Tìm tất hàm số f(x)xác định R |f(x + y) + sinx + siny| < 2,∀x,y∈R
Lời giải Lần lượt thay (x, y) = (π2,π2),(−π2,32π),rổi áp dụng bất đẳng thức tam giác ta có kết
Bài toán 22 (Turkey 2000): Cho f :R→R thỏa
|f(x+y)−f(x)f(y)| ≤1,∀x, y ∈R Chứng minh tồn hàm g(x) xác định R cho
(34)Đáp số g(x) = lim
x→∞
f(2nx) 2n
Bài toán 23 (Indian TST2004) Tìm tất hàm số f(x) xác định R
f(x+y) =f(x)f(y)−csinxsiny,∀x, y ∈R c c >
Đáp số f(x) = ±√c−1sinx+ cosx,∀x∈R
Bài tốn 24 (Thái lan 2007) Tìm hàm xác đinh R thỏa
f(x+ cos(2007y)) = f(x) + 2007 cos(f(y))
Lời giải Đặt A(x) =f(x)−d, d=f(0), c =A(1) =f(1)−A(0), A(0) =
Ta có: A(x+ cos 2007y) = A(x) + 2007 cos(A(y) +d)
ta có :A(cos 2007y) = 2007 cos(A(y) +d)≤2007suy raA(y)62007, ∀y ∈[−1,1]vàA(x+y) =
A(x) +A(y) , với y đoạn [-1,1]
Bằng quy nạp ta cóA(x+ny) =A(x) +nA(y), A(ny) = A(0) +nA(y) =A(0)
Suy A thỏa mãn PTH Cauchy vàA(x) = cxvà f(x) = cx+d, x
Tài liệu tham khảo
[1] J Aczel,1966,Lectures on Functional equations and their applications, Academic press [2] Nguyễn Văn Mậu,1997,Phương trình hàm, NXB Giáo Dục
[3] Christopher G Small,2000, Functional equations and how to solve them, Springer [4] Razvan Gelca and Titu Andreescu,2007,Putnam and Beyond,Springer
(35)TÍNH ỔN ĐỊNH NGHIỆM CỦA MỘT SỐ PHƯƠNG TRÌNH HÀM CAUCHY
Lê Thị Anh Đoan, THPT Chuyên Lê Q Đơn -Khánh Hịa
Trong nội dung viết này, ta cần quan tâm đến khái niệm ổn định Chẳng hạn ta nói phương trình hàm (Cauchy) nhân tính ổn định thỏa mãn tính chất sau:
Giả sử G nhóm, H(d) nhóm metric f :G→ H, với ε > tồn
δ >0sao cho
d(f(xy), f(x)f(y))< δ, ∀x, y ∈G
và đó, tồn đồng cấu M :G→H cho
d(f(x), M(x))< ε, ∀x∈G
1 Tính ổn định phương trình hàm (Cauchy) cộng
tính
Trước hết ta nhắc lại phương trình hàm (Cauchy) cộng tính (A)
f(x+y) = f(x) +f(y) (A)
Giả sử hàm f : X → Y thỏa mãn (A), với X Y hai khơng gian Banach Khi f gọi hàm cộng tính
Định lý Giả sử , hàm f :X →Y thỏa mãn, với ε >0, ta có
kf(x+y)−f(x)−f(y)k ≤ε, ∀x, y ∈X (1) Khi tồn giới hạn sau
A(x) = lim
n→∞2 −n
f(2nx) (2)
với x∈X tồn hàm cộng tính A:X →Y thỏa mãn
kf(x)−A(x)k ≤ε, ∀x∈X (3)
Chứng minh Thay x=y vào (1) ta k
1
f(2x)−f(x)k ≤
1
ε (4)
Sử dụng phương pháp quy nạp, ta
(36)Thật vậy, (4) ta thayx 2x, ta k1
2f(2
2x)−f(2x)k ≤ 2ε
Khi
k[1 2f(2
2x)−2f(x)]−[f(2x)−2f(x)]k=k1 2f(2
2x)−f(2x)k ≤ 2ε
hay
k
22f(2
2x)−f(x)k − k1
2f(2x)−f(x)k ≤ 22ε,
nên
k1
22f(2
2x)−f(x)k ≤ε
+ 22 , k
2nf(2
nx)−f(x)k ≤ε
1 2+
1
22 +· · ·+ 2n
=ε
1−
2n
Bây ta chứng minh dãy{
2nf(2nx)} dãy Cauchy với x∈X.Chọn m > n,
k
2nf(2
nx)−
2mf(2
mx)k=
2nk
1 2m−nf(2
m−n.2nx)−f(2nx)k
≤
2nε
1−
2m−n
=ε
1 2n −
1 2m
Do dãy {1
2nf(2nx)} dãy Cauchy Y không gian Banach nên tồn A :X → Y
choA(x) := lim
n→∞2
−nf(2nx) với mỗix∈X, hay
kA(x)−
2nf(2 n
x)k ≤
2nε
Tiếp theo ta cần chứng minhA hàm cộng tính Thay x, y 2nx 2ny (1), ta k
2nf(2
n(x+y))−
2nf(2
nx)−
2nf(2
ny)k ≤
2nε
với n∈Z∗
+, x, y ∈X.Cho n → ∞,ta
kA(x+y)−A(x)−A(y)k ≤ε
Với x∈X, ta có
kf(x)−A(x)k=k[f(x)−
2nf(2 n
x)] + [ 2nf(2
n
x)−A(x)]k ≤ kf(x)−
2nf(2 n
x)k+k
2nf(2 n
x)−A(x)]k ≤ε
1−
2n
+ε
(37)Cuối cùng, ta cần chứng minhA Giả sử tồn hàm cộng tínhA1 :X →Y thỏa
mãn (3) Khi đó, với x∈X,
kA(x)−A1(x)k=
nk[A(nx)−f(nx)] + [A1(nx)−f(nx)]k
≤ 2ε
n theo (3)
Vậy A1 =A
Định lý Với dãy số thực an thỏa mãn
|an+m−an−am|<1, n, m∈Z∗+, (6)
thì tồn giới hạn hữu hạn
A := lim
n→∞
an
n
và
kan−nAk<1, n∈Z∗+
Chứng minh Áp dụng Định lý cho Y =R.Cố định x∈X đặt an := (1ε)f(nx), n ∈Z∗+
Khi đó, theo (1) dãy(an) thỏa mãn (6) Đặt
A := lim
k→∞ ak
k
=
εklim→∞
f(kx)
k
,
Theo định lý 1, ta có
|1
εf(nx)−nA(x)|<1,
với mọin∈Z∗
+ x∈X hay
|f(x)−nεAx n
|< ε
Vì theo (1), với mọix, y ∈X ta có
|εA(x+y)−εA(x)−εA(y)|= lim
n→∞
f(nx+ny)
n −
f(nx)
n −
f(ny)
n
≤ lim
n→∞
ε n = 0,
Do
(38)2 Tính ổn định phương trình hàm (Cauchy) nhân tính
Trong phần nghiên cứu phương trình
f(xy) = f(x)f(y) (M)
Giả sử hàm f : X →Y thỏa mãn (M), với X Y hai không gian Banach Khi f gọi hàm nhân tính
Định lý Giả sử δ >0, S nửa nhóm f :S →C cho
|f(xy)−f(x)f(y)| ≤δ, ∀x, y ∈S (1) Khi
|f(x)| ≤ + √
1 + 4δ
2 =:ε, ∀x∈S (2)
hoặc f hàm nhân tính với x, y ∈S
Chứng minh Trong (2), ta có 1+ √
1+4δ
2 =:εhay ε
2−ε=δ và ε >1.Giả sử (2) không xảy ra,
tức tồn a ∈S cho |f(a)| > ε, hay |f(a)|= ε+ρ, với ρ > Trong (1), chọn
x=y=a, ta
|f(a2)−f(a)2| ≤δ (3) Khi
|f(a2)|=|f(a)2 −(f(a)2−f(a2))| ≥ |f(a)2| − |f(a)2−f(a2)| ≥ |f(a)|2−δ theo (3) = (ε+ρ)2−δ
= (ε+ρ) + (2ε−1)ρ+ρ2 (do ε2−ε=δ)
> ε+ 2ρ (do ε >1)
Bằng phép chứng minh quy nạp, ta có
|f(a2n)|> ε+ (n+ 1)ρ, ∀n= 1,2,
Với x, y, z ∈S,
|f(xyz)−f(xy)f(z)| ≤δ, |f(xyz)−f(x)f(yz)| ≤δ
Ta có
(39)và
|f(xy)f(z)−f(x)f(y)f(z)| ≤ |f(xy)f(z)−f(x)f(yz)|
+|f(x)f(yz)−f(x)f(y)f(z)| ≤2δ+|f(x)|δ
Suy
|f(xy)−f(x)f(y)|.|f(z)| ≤2δ+|f(x)|δ
Chọnz =a2n, ta
|f(xy)−f(x)f(y)| ≤ 2δ+|f(x)|δ |f(a2n
)|
với mọix, y ∈S n = 1,2 Cho n → ∞, ta f(xy) =f(x)f(y), ∀x, y ∈S.Vậy f
là hàm nhân tính
3 Các ví dụ áp dụng
Ví dụ Nghiệm phương trình Jensen
Bài tốn Tìm hàm f :R→R thỏa mãn phương trình sau
f
x+y
2
= f(x) +f(y)
2 ∀x, y ∈R (1)
Thay y= vào (1), ta
f
x
2
= f(x) +f(0)
2 ∀x∈R (2)
Khi áp dụng (1) (2), ta
f(x) +f(y) =f
x+y
2
= f(x+y) +f(0)
hay
f(x) +f(y) = f(x+y) +f(0), ∀x, y ∈R
Đặt A(x) =f(x)−f(0) Ta có A(x) +A(y) = A(x+y), ∀x, y ∈ R Vậy A hàm cộng tính Rnên f(x) =A(x) +α, α=f(0)
Chú ý Nếu tốn có thêm giả thiết: hàmf liên tục nghiệm tìm làf(x) = ax+α,
với a, α số tùy ý
Tiếp theo ta xét tính ổn định nghiệm phương trình (1) Mệnh đề Giả sử hàm f thỏa mãn
f
x+y
2
−f(x) +f(y)
2
≤ε (3)
với ε số dương tùy ý cho trước với x, y ∈R Khi tồn hàm cộng tính A:R→R cho
(40)Chứng minh Thay y= vào (3), ta
fx
2
−f(x) +f(0)
2
≤ε ∀x∈R
Do
f
x+y
2
−f(x+y) +f(0)
2
≤ε ∀x, y ∈R
Ta có
f(x) +f(y) −
f(x+y) +f(0)
≤
f(x) +f(y) −
f(x+y)
+
f(x+y) −
f(x+y) +f(0)
≤2ε
hay
|f(x+y) +f(0)−f(x)−f(y)| ≤4ε (4) Đặtg(x) = f(x)−f(0) Thay vào (4), ta
|g(x+y)−g(x)−g(y)| ≤4ε
Theo tính ổn định hàm cộng tính, tồn hàm cộng tínhA cho |g(x)−A(x)| ≤4ε
Ta có
|f(x)−A(x)−f(0)|=|g(x)−A(x)| ≤4ε
Ví dụ Nghiệm phương trình Cauchy hai ẩn hàm
Bài tốn Tìm cặp hàm f, g :R→R thỏa mãn phương trình sau
f(x+y) = g(x) +g(y) ∀x, y ∈R (5) Thay y= vào (5), ta
f(x) = g(x) +g(0) ∀x∈R,
hay f(x) = g(x) +α, với α=g(0).Do g(x) =f(x)−α với x∈R
Thay vào phương trình (5), ta
f(x+y) = f(x) +f(y)−2α (6)
Đặtf(x) =A(x) + 2α Phương trình (6) trở thành
(41)hay
A(x+y) =A(x) +A(y) ∀x, y ∈R
Vậy A hàm cộng tính trênR nên
f(x) =A(x) + 2α g(x) = A(x) +α
Chú ý Nếu tốn có thêm giả thiết: hàm f, g liên tục nghiệm tìm
f(x) =ax+ 2α g(x) = ax+α
với a, α số tùy ý
Tiếp theo ta xét tính ổn định nghiệm phương trình (5) Mệnh đề Giả sử hàm f, g:R→R thỏa mãn
|f(x+y)−g(x)−g(y)| ≤ε (7) với ε số dương tùy ý cho trước với x, y ∈R Khi tồn hàm cộng tính A:R→R cho
|f(x)−A(x)−f(0)| ≤4ε
|g(x)−A(x)−g(0)| ≤3ε
với x∈R
Chứng minh Thay y= vào (7), ta
|f(x)−g(x)−g(0)| ≤ε, ∀x∈R, (8) suy
|f(0)−2g(0)| ≤ε (9)
Sử dụng (8), ta
|f(x+y)−g(x+y)−g(0)| ≤ε, ∀x, y ∈R (10) Ta có
|f(x+y)−g(x+y)−g(0)|=|f(x+y)−g(x)−g(y)−g(x+y) +g(x) +g(y)−g(0)| nên kết hợp (7) (10) thu
|g(x+y)−g(x)−g(y) +g(0)| ≤ |f(x+y)−g(x+y)−g(0)|+|f(x+y)−g(x)−g(y)| ≤2ε
hay
(42)với mọix, y ∈R Đặt
G(x) =g(x)−g(0), (12)
với mọix, y ∈R Thế vào (11) ta
|G(x+y)−G(x)−G(y)| ≤2ε, ∀x, y ∈R
Theo định lý tính ổn định hàm cộng tính, tồn hàm cộng tínhA :R→R cho
|G(x)−A(x)| ≤2ε, ∀x∈R (13) Từ (12) (13) ta
|g(x)−A(x)−g(0)| ≤2ε, ∀x∈R (14) Từ (8), (9) (14) ta
|f(x)−A(x)−f(0)|=|f(x)−g(x)−g(0) +g(x)−A(x)−g(0) + 2g(0)−f(0)| ≤ |f(x)−g(x)−g(0)|+|g(x)−A(x)−g(0)|+|f(0)−2g(0)| ≤ε+ 2ε+ε= 4ε
Ví dụ Nghiệm phương trình Pexider
Bài tốn Tìm tất hàm f, g, h:R→R thỏa mãn phương trình sau
f(x+y) =g(x) +h(y) ∀x, y ∈R (15) Thay y= vào (15), ta
f(x) =g(x) +h(0) ∀x∈R,
hay f(x) = g(x) +α, với α=h(0).Do g(x) =f(x)−α với x∈R
Thay x = vào (15), ta f(y) = h(x) +β, với β =g(0), hay h(x) = f(x)−β với
x∈R
Phương trình (15)trở thành
f(x+y) = f(x) +f(y)−α−β, ∀x, y ∈R (16) Đặtf(x) =A(x) +α+β thay vào phương trình (16), ta
A(x+y) +α+β =A(x) +α+β+A(y) +α+β−α−β
hay
A(x+y) =A(x) +A(y) ∀x, y ∈R
Vậy A hàm cộng tính trênR nên
(43)Chú ý Nếu toán có thêm giả thiết: hàm f, g, h liên tục nghiệm tìm
f(x) = ax+α+β g(x) =ax+β h(x) =ax+α
với a, α, β số tùy ý
Tiếp theo ta xét tính ổn định nghiệm phương trình (15) Mệnh đề Giả sử hàm f, g, h:R→R thỏa mãn
|f(x+y)−g(x)−h(y)| ≤ε (17) với ε số dương tùy ý cho trước với x, y ∈R Khi tồn hàm cộng tính A:R→R cho
|f(x)−A(x)−f(0)| ≤6ε
|g(x)−A(x)−g(0)| ≤4ε
|h(x)−A(x)−h(0)| ≤6ε
với x∈R
Chứng minh Thay y= vào (17), ta
|f(x)−g(x)−h(0)| ≤ε, ∀x∈R, (18) suy
|f(0)−g(0)−h(0)| ≤ε (19) Thayy = vào (17), ta
|f(y)−h(y)−g(0)| ≤ε, ∀y∈R, (20) Từ (18) (20)
|h(x)−g(x)−h(0) +g(0)|=|f(x)−g(x)−h(0) +h(x) +g(0)−f(x)| ≤ |f(x)−g(x)−h(0)|+|f(x)−h(x)−h(0)| hay
|h(x)−g(x)−h(0) +g(0)| ≤2ε, ∀x∈R (21) Sử dụng (18), ta
|f(x+y)−g(x+y)−h(0)| ≤ε, ∀x, y ∈R (22) Ta có
(44)nên kết hợp (17) (22) thu
|g(x+y)−g(x)−h(y) +h(0)| ≤ |f(x+y)−g(x+y)−h(0)|+|f(x+y)−g(x)−h(y)| ≤2ε
Mặt khác
|g(x+y)−g(x)−h(y) +h(0)|=|g(x+y)−g(x)−g(y) +g(0)−h(y) +g(y)−g(0) +h(0)| nên từ (21)
|g(x+y)−g(x)−g(y) +g(0)| ≤ |g(x+y)−g(x)−h(y)−h(0)|+|h(y)−g(y) +g(0)−h(0)| ≤4ε
hay
|[g(x+y)−g(0)]−[g(x)−g(0)]−[g(y)−g(0)]| ≤4ε, (23) với mọix, y ∈R Đặt
G(x) =g(x)−g(0), (24)
với mọix, y ∈R Thế vào (23) ta
|G(x+y)−G(x)−G(y)| ≤4ε, ∀x, y ∈R
Theo định lý tính ổn định hàm cộng tính, tồn hàm cộng tínhA :R→R cho
|G(x)−A(x)| ≤4ε, ∀x∈R (25) Từ (24) (25) ta
|g(x)−A(x)−g(0)| ≤4ε, ∀x∈R (26) Từ (18), (19) (26) ta
|f(x)−A(x)−f(0)|=|[f(x)−g(x)−h(0)] + [g(x)−A(x)−g(0)] + [g(0) +h(0)−f(0)]| ≤ |f(x)−g(x)−h(0)|+|g(x)−A(x)−g(0)|+|f(0)−g(0)−h(0)| ≤ε+ 4ε+ε= 6ε
Từ (21) (26) ta
|h(x)−A(x)−h(0)|=|[h(x)−g(x)−h(0) +g(0)] + [g(x)−A(x)−g(0)]| ≤ |h(x)−g(x)−h(0) +g(0)|+|g(x)−A(x)−g(0)| ≤2ε+ 4ε= 6ε
Ví dụ Tìm tất hàm liên tục f :R→R thỏa mãn phương trình sau
f(x+y ) =
p
(45)Từ phương trình (27), ta có f(x)≥ 0, ∀x ∈R Giả sử tồn tạix0 ∈ R chof(x0) =
Khi
f(x0+y ) =
p
f(x0)f(y) = ∀y∈R,
hay f(x) = với x∈R
Xét f(x)>0,∀x∈R Khi lấy logarit hai vế phương trình (27), ta
lnf(x+y ) =
lnf(x) + lnf(y)
2 , ∀x, y ∈R
Đặtg(x) = lnf(x)ta có
g(x+y ) =
g(x) +g(y)
2 , ∀x, y ∈R
hayg nghiệm phương trình Jensen, tức làg(x) =ax+b Suy nghiệm phương trình (27) làf(x) = eax+b với a, b∈
R
Tiếp theo ta xét tính ổn định nghiệm phương trình (27) Mệnh đề Giả sử hàm f :R→R+ thỏa mãn
|f(x+y )−
p
f(x)f(y)| ≤ε (28)
với x, y ∈R
|f(x)−f(−x)| ≤δ (29) với ε, δ số dương tùy ý cho trước Giả sử tồn tạif(a)−1, tồn hàmE :
R→ R+ cho
|E(x+y)−E(x)−E(y)| ≤α, ∀x, y ∈R (30)
|f(x)−1
2(E(x)−E(−x))| ≤β, ∀x∈R (31)
với α, β số
Chứng minh Đặt m= sup
x∈R p
f(x)f(a) Từ điều kiện (29) m hữu hạn Khi đó, ta có p
f(x)f(−a)≤pf(−x)f(a) +|pf(x)f(−a)−pf(−x)f(a)| ≤m+|f(x−a
2 )−
p
f(x)f(−a)|+|f(−x+a )−
p
f(−x)f(a)|
+|f(x−a )−f(
−x+a
2 )|
≤m+ 2ε+δ
Đặth :R→R+ thỏa
(46)Khi h hàm chẵn
|h(x)−f(x)|=pf(x)|pf(x)−pf(−x)| ≤2 m
f(a), x∈R, |h(x)
p
f(a)| ≤m (32) ĐặtE :R→R+ thỏa mãn
E(x) = h(x) +pf(a) ∀x∈R Áp dụng (32) ta có
|E(x+y)−E(x)E(y)|=|h(x+y) +pf(a)−h(x)h(y)−(h(x) +h(y))pf(a)−f(a)| ≤ |h(x+y)|+|h(x)h(y)|+|(h(x) +h(y))pf(a)|+|f(a)|
≤ |h(x+y)−f(x+y)|+|f(x+y)|+|h(x)h(y)f(a)f−1(a)|
+|h(x)pf(a)|+|h(y)pf(a)|+pf(a) +|f(a)| ≤2 m
2
f(a)+
m
p
f(a) +
m2
f(a) + 2m+
p
f(a) +|f(a)|=α
và
|f(x)−1
2(E(x)−E(−x))|=
|f(x)−h(x) +h(x)−
2(h(x) +h(−x))−
p
f(a)| ≤ |f(x)−h(x)|+pf(a)≤2 m
2
f(a)+
p
f(a) = β
Tài liệu tham khảo
[1] Nguyễn Văn Mậu,Phương trình hàm, Nhà xuất Giáo dục, 1997
(47)MỘT SỐ LỚP PHƯƠNG TRÌNH HÀM ĐA ẨN SINH BỞI PHI ĐẲNG THỨC
Trần Viết Tường, Trường THPT Trần Phú - Đà Nẵng
Trong tốn học phổ thơng tốn phương trình hàm loại tốn thường khó, thường xuyên xuất đề thi học sinh giỏi quốc gia, Olympic Toán khu vực Quốc tế, Olympic sinh viên trường Đại học cao đẳng Liên quan đến dạng toán toán đặc trưng khác hàm số tính chất liên quan với chúng
Để hệ thống phương trình hàm, cần thiết phải hệ thống kiến thức nâng cao dạng phương trình hàm ứng dụng chúng
Đối với tốn phương trình hàm với nhiều ẩn hàm lớp hàm cụ thể: liên tục, khả vi, tuần hoàn, lồi lõm, cần nắm số kĩ thuật biến đổi hàm số, khảo sát tính chất hàm thực phép biến hình trục thực
1 Phương trình hàm sinh phi đẳng thức a2 + b2 6≡
g(a + b)h(a − b)
Bài tốn Tìm hàm số f, g, h xác định liên tục R thỏa mãn điều kiện
f(x2+y2) =g(x+y).h(x−y), ∀x, y ∈R (1)
Giải Xét trường hợp g(0) =
Cho y=−x , phương trình cho trở thành
f(2x2) = 0, ∀x∈R Suy
f(x) = 0, ∀x≥0
Thayf(x)vào phương trình cho ta
g(x+y).h(x−y) = 0, ∀x, y ∈R
g(u).h(v) = 0, ∀u, v ∈R
Do
(
g(x) =
h(x) liên tục tùy ý R (
h(x) =
(48)Vậy nghiệm trường hợp :
h f(x) = với x≥0
f(x) =q(x) với q(x) liên tục tùy ý (−∞; 0] q(0) =
g(x)≡0
h(x) hàm liên tục tùy ý
h f(x) = với x≥0
f(x) =q(x) với q(x) liên tục tùy ý (−∞; 0] q(0) =
g(x) hàm số liên tục tùy ý g(0) =
h(x)≡0
Xét trường hợp h(0) =
Cho y=x, phương trình cho trở thành
f(2x2) = 0, ∀x∈R Suy
f(x) = 0, ∀x≥0
Thế f(x) vào cho ta
g(x+y).h(x−y) = 0, ∀x, y ∈R
g(u).h(v) = 0, ∀u, v ∈R
Do (
g(x) =
h(x) liên tục tùy ý R;h(0) =
(
h(x) =
g(x) liên tục tùy ý R
Vậy nghiệm phương trình trường hợp :
h f(x) = với x≥0
f(x) =q(x) với q(x) liên tục tùy ý (−∞; 0] q(0) =
g(x) hàm liên tục tùy ý
h(x)≡0
hoặc
h f(x) = với x≥0
f(x) =q(x) với q(x) liên tục tùy ý (−∞; 0] q(0) =
g(x)≡0
(49)Xét trường hợp g(0)6= h(0)6= Ta có f(0) 6=
Cho x=y, phương trình cho trở thành
f(2x2) = g(2x).h(0), g(2x) = f(2x 2)
h(0) , ∀x∈R, g(x) =
f(x 2)
h(0) , ∀x∈R
Cho x=−y, phương trình cho trở thành
f(2x2) = g(0).h(2x), h(2x) = f(2x 2)
g(0) , ∀x∈R, h(x) =
f(x 2)
g(0) , ∀x∈R
Thayg(x) h(x)vào phương trình ta
f(x2+y2) =f[(x+y) 2 ].f[
(x−y)2 ]
1
g(0)h(0), ∀x, y ∈R
Đặt
u= (x+y) 2
v = (x−y) 2
.Khi ta có
f(u+v) =f(u).f(v)
g(0)h(0), ∀u, v ≥0 (2)
Đặtf(u) =g(0)h(0)F(u), ∀u≥0
Phương trình (3.2) trở thành
g(0)h(0)F(u+v) = g(0)h(0)F(u).g(0)h(0)F(v)
g(0)h(0), ∀u, v ≥0
⇔F(u+v) = F(u)F(v), ∀u, v ≥0
Ta có
F(u) =au u≥0;a >0
f(u) =b.au với b=g(0)h(0);a >0
f(x) =b.ax, ∀x≥0;a >0, b6=
Suy g(x) =
f(x 2)
h(0) , ∀x∈R=g(0).a
x2
2 ∀x∈R=m.a
x2
2 với m =g(0)
h(x) =
f(x 2)
g(0) , ∀x∈R=h(0).a
x2
2 , ∀x∈R=n.a
x2
(50)Vậy nghiệm phương trình trường hợp :
h f(x) =b.ax với x≥0;a >0;b 6=
f(x) =q(x) với q(x) liên tục tùy ý (−∞; 0] q(0) =
g(x) = m.ax
2
2 , ∀x∈R
h(x) = n.ax
2
2 , ∀x∈R
Tóm lại nghiệm tốn
h f(x) = với x≥0
f(x) =q(x) với q(x) liên tục tùy ý (−∞; 0] q(0) =
g(x)≡0
h(x) hàm liên tục tùy ý
h f(x) = với x≥0
f(x) =q(x) với q(x) liên tục tùy ý (−∞; 0] q(0) =
g(x) hàm số liên tục tùy ý g(0) =
h(x)≡0
hoặc
h f(x) = với x≥0
f(x) =q(x) với q(x) liên tục tùy ý (−∞; 0] q(0) =
g(x) hàm liên tục tùy ý
h(x)≡0
hoặc
h f(x) = với x≥0
f(x) =q(x) với q(x) liên tục tùy ý (−∞; 0] q(0) =
g(x)≡0
h(x) hàm liên tục tùy ý với h(0) =
hoặc
h f(x) =b.ax với x≥0;a >0;b 6=
f(x) =q(x) với q(x) liên tục tùy ý (−∞; 0] q(0) =
g(x) = m.a x2
2, ∀x∈R
h(x) = n.a x2
(51)Bài tốn Tìm hàm số f, g, h xác định liên tục R thỏa mãn điều kiện
f(x2+y2) = g(x+y) +h(x−y), ∀x, y ∈R (3)
Giải Nghiệm toán
h f(x) =ax+b+c với x≥0
f(x) =q(x) với q(x) liên tục tùy ý (−∞; 0] q(0) =
g(x) = ax
2
2 +b, ∀x∈R
h(x) = ax
2
2 +c, ∀x∈R
với b=g(0);c=h(0)
Bài tốn Tìm hàm số f, g, h xác định liên tục R thỏa mãn điều kiện
f(x2+y2) = g(x2)−h(y2), ∀x, y ∈R (4)
Giải Nghiệm toán
h f(x) =ax+b với x≥0
f(x) =q(x) với q(x) liên tục tùy ý (−∞; 0] q(0) =
g(x) = ax+c, ∀x∈R
h(x) = −ax−d, ∀x∈R
2 Phương trình hàm sinh đẳng thức a2 − b2 = (a +
b)(a − b)
Bài toán Tìm hàm số f, g, h liên tục xác định R thỏa mãn điều kiện
f(x2−y2) = (x+y)g(x−y), ∀x, y ∈R (5)
Giải Cho y= 0, phương trình cho trở thành
f(x2) =x.g(x), ∀x∈R
Nếux=
f(x) = (6) Nếux6= , ta có
g(x) = f(x 2)
(52)Do đó, phương trình cho trở thành
f(x2−y2) = (x+y)f[(x−y) 2]
x−y , ∀x6=y f(x2−y2)
x+y =
f[(x−y)2]
x−y , ∀x6=±y f(x2−y2)
x2−y2 =
f[(x−y)2]
(x−y)2 , ∀x6=±y
Đặth(x) = f(x)
x với x6= Khi đó, ta có
h(x2−y2) = h[(x−y)2]
Cho x=y+ , ta có
h(2y+ 1) =h(1), ∀y ∈R
h(x) =a, ∀x6=
Do
f(x) = ax với x6= (7)
Kết hợp (10) (7) ta có
f(x) =ax, ∀x∈R
Suy
g(x) = ax
x =ax, ∀x6=
Vậy nghiệm toán (
f(x) =ax g(x) = ax
Bài toán Tìm tất hàm sốf, g, h xác định liên tục R thỏa mãn điều kiện
f(x2−y2) = g(x−y) +h(x+y), ∀x, y ∈R (8)
Giải Nghiệm toán
f(x) = a, ∀x∈R
g(x) =b, ∀x∈R
h(x) =c, ∀x∈R
trong a, b, c∈R;a =b+c
Bài tốn Tìm tất hàm sốf, g, h xác định liên tục R thỏa mãn điều kiện
(53)Giải Nghiệm toán
f(x) =mx+a−b, ∀x∈R;a, b, m≥0
g(x) =√mx2+a
h(x) =√mx2+b
hoặc
f(x) =mx+a−b, ∀x∈R;a, b, m≥0
g(x) =√mx2+a
h(x) =−√mx2+b
hoặc
f(x) =mx+a−b, ∀x∈R;a, b, m≥0
g(x) =−√mx2+a
h(x) =√mx2+b
hoặc
f(x) =mx+a−b, ∀x∈R;a, b, m≥0
g(x) =−√mx2+a
h(x) =−√mx2+b
3 Một số tốn phương trình đa ẩn hàm khác
Bài tốn Tìm hàm số f, g, h liên tục xác định R thỏa mãn điều kiện
f(x)−g(y) =xh(y)−yh(x), ∀x, y ∈R (10)
Giải Cho x=y, phương trình cho trở thành
f(x)−g(x) = 0⇔f(x) = g(x), ∀x∈R
Cho y= 0, phương trình cho trở thành
f(x)−g(0) =x.h(0), ∀x∈R
f(x) =x.h(0) +g(0), ∀x∈R
f(x) =ax+b với a=h(0);b =g(0)
Thayf, g vào phương trình cho ta
(ax+b)−(ay+b) =xh(y)−yh(x), ∀x, y ∈R
ax−ay=xh(y)−yh(x), ∀x, y ∈R
a y−
a x=
h(y)
y −
h(x)
x , ∀x, y ∈R a
x− h(x)
a =
a y−
h(y)
(54)Suy
a x−
h(x)
x =C với C số h(x) = −Cx+a
h(x) = cx+a với c=−C
Thử lại phương trình ta thấy f, g, h thỏa mãn Vậy nghiệm phương trình
(
f(x) =g(x) = ax+b h(x) = cx+a
Bài tốn Tìm tất hàm sốf, g xác định liên tục R thỏa mãn điều kiện
f(x)−f(y) = (x+y)g(x−y), ∀x, y ∈R (11)
Giải Nghiệm toán (
f(x) =ax2+b
g(x) = ax với mọia, b∈R
Bài tốn Tìm tất hàm sốf, g xác định liên tục R thỏa mãn điều kiện
f(x) +f(y) + 2xy= (x+y)g(x+y), ∀x, y ∈R (12)
Giải Nghiệm toán f(x) = x2+ax g(x) =
(
x+a với x6=
c với x=
Bài tốn 10 Tìm tất hàm sốf, g xác định liên tục R thỏa mãn điều kiện
f(x).g(y) = x2−y2, ∀x, y ∈R (13)
Giải Nếu tồn x0 cho f(x0) = Khi ta có
0 =f(x0).g(y) =x20−y2, ∀y∈R (vơ lý)
Suy
f(x)6≡0, ∀x∈R
Tương tự ta có
g(x)6≡0, ∀x∈R
Cho x=y, phương trình (15) trở thành
f(x).g(x) = ⇔f(x) = g(x) = (loại f(x)6≡0;g(x)6≡0)
(55)Bài tốn 11 Tìm tất hàm sốf, g, h xác định liên tục R thỏa mãn điều kiện
f(x+y) +g(x−y) = h(xy), ∀x, y ∈R (14)
Giải Nghiệm toán
f(x) = mx
4 +b, ∀x∈R
g(x) =−mx
2
4 +a, ∀x∈R
h(x) =mx+a+b, ∀x∈R
Bài tốn 12 Tìm tất hàm số dương f, g, h xác định liên tục R thỏa mãn điều kiện
f(x+y).g(x−y) =h(xy), ∀x, y ∈R (15)
Giải Do f, g, h hàm số dương nên phương trình (??) tương đương
lnf(x+y) + lnf(x−y) = lnh(xy), ∀x, y ∈R
Đặt
lnf(x) = F(x) lng(x) =G(x) lnh(x) =H(x)
Khi ta có F(x+y) +G(x−y) = H(xy), ∀x, y ∈R
Ta có
F(x) = mx
4 +b, ∀x∈R
G(x) = −mx
2
4 +a, ∀x∈R
H(x) =mx+a+b, ∀x∈R
Suy
f(x) = emx
2
4 +b, ∀x∈R
g(x) =e−mx
2
4 +a, ∀x∈R
h(x) =emx+a+b, ∀x∈
R
Thử lại ta thấy hàm f, g, h thỏa mãn điều kiện toán Vậy nghiệm toán
f(x) = emx
2
4 +b, ∀x∈R
g(x) =e−mx
2
4 +a, ∀x∈R
h(x) =emx+a+b, ∀x∈
R
Bài tốn 13 Tìm tất hàm sốf, g xác định liên tục R thỏa mãn điều kiện
f(x) +g(x) +f(y)−g(y) = sinx−cosy, ∀x, y ∈R (16)
Giải Nghiệm toán
f(x) =
2(sinx−cosx), ∀x∈R
g(x) =
(56)Tài liệu tham khảo
[1] Nguyễn Văn Mậu, 1997, Phương trình hàm, NXB Giáo Dục
[2] Nguyễn Văn Mậu, 2006, Các toán nội suy áp dụng, NXB Giáo Dục
(57)TỪ CÔNG THỨC EULER ĐẾN CÁC BÀI TOÁN SỐ PHỨC
Lê Sáng, Nguyễn Đinh Huy, Trường THPT chuyên Lê Quý Đôn - Khánh Hịa
Áp dụng số phức hình học phẳng có nhiều tài liệu tác giả Đồn Quỳnh,Nguyễn Hữu Điển sách giáo khoa dùng tham khảo.Trong viết đề cập đến phương pháp hiệu giải hệ phương trình, chứng minh đẳng thức, hay dùng để tính tổng gọi phương pháp số phức Trước xét thêm khai triển chuổi Taylor để xây dựng công thức Moivre mà sách giáo khoa có từ cơng thức nhân số phức dạng lượng giác chứng minh quy nạp I Khai triển Taylor công thức Moivre
1 Khai triển Taylor công thức Moivre
Cho z số phức
ez = + z 1! +
z2 2! +
z3
3! + +
zn
n! +
Trường hợp đặc biệt, với góc x, ta có
eix = +ix
1!−
x2 2! −i
x3 3! +
x4 4! +
Phần thực phần ảo củaeix lần lượt là
Re(eix) = 1− x
2 2! +
x4 4! −
x6
6 +· · ·, Im(e
ix) = x
1!−
x3 3! +
x5 5! −
x7 7! +· · ·
Hai chuổi khai triển Taylor củacosx sinx
Ta có cơng thức Euler saueix = cosx+isinxvà ta có cơng thức tuyệt đẹp: eiπ =
−1Ngồi doeinx = (e(ix)n)nên ta suy cơng thức Moivre:cosnx+isinnx= (cosx+isinx)n
Ví dụ mở đầu
Tìm khai triển Taylor hàmf(x) =excosθcos(xsinθ) tại điểm 0, với θ là tham số.
Giải Đặtg(x) = excosθsin(xsinθ) viết :
f(x) +ig(x) =excosθ(cos (xsinθ) +isin (xsinθ)) =excosθ.eixsinθ =ex(cosθ+isinθ)
Dùng công thức Moivre khai triển Taylor:
1 + x
1!(cosθ+ sinθ) +
x2
1!(cos 2θ+isin 2θ) +· · ·+
xn
(58)Khai triển rút gọn ta thu
f(x) = + cosθ 1! x+
cos 2θ
2! x
+· · ·+ cosnθ
n! x
n
+· · · Sau số toán áp dụng phương pháp số phức
2 Chứng minh đẳng thức
Bài toán Chứng minh cos 61 +
1 sin 240 +
1 sin 480 =
1 sin 120 Lời giải Đặt z = cos 60+isin 60 Ta có z15 =i
Màcos 60 = z22+1z ,sin 120 = z24iz−21,sin 24
0 = z8−1
2iz4 ,sin 48
0 = z16−11
2iz8
Đẳng thức cần chứng minh trở thành chứng minh z22+1z −
2iz2
z4−1+
2iz4
z8−1+
2iz8
z16−1 = 0.Quy đồng mẫu
số, thu gọn Ta có z16−1−iz(z14+ 1) = 0z16−1−iz(z14+ 1) = 0tức là iz−1−i2−iz = 0
Điều hiển nhiên
Bài toán Cho a, b, c số thực thỏa cosa+ cosb+ cosc= sina+ sinb+ sinc=
Chứng minh
a)cos 2a+ cos 2b+ cos 2c= sin 2a+ sin 2b+ sin 2c=
b)3(cos(a+b+c) = cos 3a+ cos 3b+ cos 3cvà 3(sin(a+b+c) = sin 3a+ sin 3b+ sin 3c
Lời giải Đặt x= cosa+isina, y = cosb+isinb, z = cosc+isinc
Từ giả thiết ta cóx+y+z =x−1+y−1+z−1 = suy xy+yz+zx=
a)x2+y2+z2 = (x+y+z)2−2(xy+yz+zx) = 0 Suy kết quả
b)x3+y3+z3 = 3xyz Suy kết Bài toán Chứng minh đẳng thức:
1 +itant
1−itant
n
= +itannt
1−itannt, n>1
Lời giải Nếu ta nhân tử mẫu vế bên trái vớicost, vế bên phải bởicosnt,thì ta có đuợc đẳng thức:
eit e−it
n
= ee−intint
Bài toán Chứng minh đẳng thức n + n k + n 2k
+ =
n
k
k
X
i=1
cosnjπ
k cos njπ
k
Lời giải Let C1, C2, , Ck k nghiệm đơn vị, tức là, Cj = cos2jπk +isin2jπk , j =
1,2, , k
Như
k
X
j=1
(59)Vì1 +Cj = 2cosjπk cosjπk +isinjπk
Nên áp dụng công thức Moivre
k
X
j=1
(1 +Cj)n= k
X
j=1
2ncosnjπ
k
cosnjπ
k +isin njπ
k
So sánh phần thực vế ta kết Bài toán Chứng minh đẳng thức
1− n + n − n
+ = 2n2
cosnπ
4 +isin
nπ
4
, n ≥1
Lời giải Dùng công thức Moivre, ta được:
(1 +i)n=h√2cosπ
4 +isin
π
4
in
= 2n2
cosnπ
4 +isin
nπ
4
Khai triển (1 +i)n và cho hai vế phần thực,ta có điều phải chứng minh
ban đầu
3 Tính tích tổng
Bài toán Chứng minh tổng cos2π
7 + cos 4π
7 + cos 6π
7 =−
Lời giải Nếu z = cosπ7 +isinπ7 z7 = 1 Điều cần chứng minh tương đương với
2
z+
z
+
z2+
z2
+
z3+
z3
+1 =
Nhân cho 2z3 , xếp số hạng z6+z5+z4+z3+z2+z+ = zz7−−11 =
Bài tốn Tính tổng T =q3 cos2π
9 +
3 q
cos49π +q3 cos8π
9
Lời giải Xét phương trình z9−1 = 0 có nghiệm z
k, k = 0,1, ,8 có tổng
Phương trìnht4+t3 −3t2−2t+ = (t+ 1) (t3−3t+ 1) = có nghiệm:
2 cos2π ,2 cos
4π
9 ,2 cos 6π
9 ,2 cos 8π
9
Đặtt =z+ 1z, phương trình (t3−3t+ 1) = có nghiệm2 cos29π,2 cos49π,2 cos89π
Dùng biểu thức đối xứng nghiệm phương trình bậc 3, ta có tổng q
3√39−6
(60)Lời giải Đặt z = cos 200+isin 200
Ta có cos 200 = z+1
z
,2 cos 400 = z2+
z2
,2 cos 800 = z4+
z4
, z9 =−1
Suy (z− −z8) (z2− −z7) (z3− −z5) = (z− −z2+z3− −z4+z5− −z6+z7 − −z8) =
Bài tốn Tính tích sin2πnsin22πnsin23πn· · ·sin(n−2n1)π
Lời giải Xét đa thức P(X) =X2n−1
Có nghiệm xk = coskπn +isinkπn, k = 0,1,· · · ,2n−1, x0 = 1, xn =−1, xk =x2n−kkhi 16
k6n−1
Khi đóP(X) = (X2−1)nQ−1
k=1
(X−xk)(X−xk) = (X2−1) n−1
Q
k=1
(X2−2 coskπ
n + 1),doxk+xk =
2 coskπn, xkxk=
Chia vế chox2−1 , ta có
x2n−2 +x2n−4+· · ·x4+x2+ =
n−1
Y
k=1
x2−2xcoskπ
n +
Lấyx= 1, ta n= 2n−1
n−1
Q
k=1
1−coskπn=2n−1
n−1
Q
k=1
2sin2kπ2n,suy
n−1
Q
k=1
sinkπ2n =
√
n
2n−1
Bài toán 10 Tính tổng:
n
1
cosx+
n
2
cos 2x+ +
n n
cosnx
Lời giải Gọi tổng cần tìm S1 cho
S2 =
n
1
sinx+
n
2
sin 2x+ +
n n
sinnx
Dùng công thức Euler, ta viết
1 +S1+iS2 =
n + n
eix+ +
n
2
ei2x
Nhờ tính nhân lũy thừa, ta có:
n X k=0 n k
eixk = +eixn=2cosx
n
ei2x n
Tổng câu hỏi phần thực khai triển bé 1, điều dẫn đến kết tổng
2ncosnx 2cos
nx
(61)Bài tốn 11 (USAMO 1999) Tính (cosα)(cos 2α)(cos 3α) .(cos 999α)
với α= 19992π
Lời giải Xét toán tổng quát, với n số nguyên lẻ, tính:
S = (cosα)(cos 2α)(cos 3α) .(cosnα) với α= 2π 2n+
Chúng ta choς =ei ? vàS = 2−n
n
Q
k=1
εk+ε−k
Khi εk+ε−k =ε2n+1−k, k= 1,2, , n, chúng chứa:
S2 = 2−2x Q2n
k=1
εk+ε−k
= 2−2n. Q2n
k=1
ε−k. Q2n
k=1
1 +ε2k
Màε−(1+2+ +2n) = 1 Bởi vì (1 + + .+ 2n) = n(2n+ 1) là bội của 2n+ 1.
Xét
2n
Q
k=1
1 +ε2k
,chú ý ta viết
2n
Q
k=1
1 +εk
,từ số ε2k có hơn(2n+ 1)nghiệm.
Trong khai triển
zn+1−1 = (z−1) 2n
Y
k=1
z−εk
Cho z =−1 chia vế -2
2n
Q
k=1
1 +εk= 1.Nên S2 = 2−2n, S =±2−n
Khi1≤k ≤n, coskα π2 < kα < π.Giá trị k chẳn , lẻ, tùy theon số dư chia cho
S=
(
+2−n n ≡1 hay 2(mod4)
−2−n n ≡0hay (mod4)
Khi n= 999≡3 (mod4)
Ta có kết là−2−999.
4 Giải phương trình -hệ phương trình
Bài tốn 12 Giải phương trìnhcosx+ cos 2x+ cos 3x= sinx+ sin 2x+ sin 3x
Lời giải Đặt z = cosx+isinx
Phương trình thành z22−iz1 + z24iz−21 +
z6−1
2iz3 =
z2+1
2z + z4+1
2z2 +
z6+1
2z3
Thu gọn ta được(z4−i) (z2+z+ 1) =
z2+z+ = suy z = −1±i √
3 =cos
±2π
3 +isin
±2π
3 , x=
±2π
3 + 2kπ, k ∈Z
z4 =i suy cos 4x+isin 4x= cosπ
2 +isin
π
2, x=
π
8 +k
π
(62)Bài toán 13 (VMO 1996 ) Giải hệ phương trình
√
3x(1 +
x+y) =
p
7y(1−
x+y) =
√
2
Lời giải Bài có nhiều giải, vân dụng phương pháp số phức Thế √x=u,√y=v, ta có hệ
u(1 +
u2+v2) =
√
3
v(1−
u2+v2) = 4√2
√
7
Đặt z =u+iv, u2+v2 là bình phương modun số phức z, nên nhân phương trình sau
cho i, cộng vế ta có phương trình phức z2−(√2
3 +i 4√√2
7)z+ = có nghiệm
√
3 ±
√
21 +i( 2√√2
7 ±
√
2),dấu cộng, trừ tương ứng Do nghiệmx, y (√1
3 ±
√
21) (2√√2
7 ±
√
2)2
Bài toán 14 (Chọn đội tuyển chuyên Vĩnh Phúc 2010-2011 ) Giải hệ phương trình :
(
x+x32x+−yy2 = (1)
y− x32x++yy2 = (2)
Lời giải Nhân phương trình (2) với i cộng theo vế với (1) ta :
x+yi+ x−yi
x2 +y2 −
i(x−yi)
x2+y2 = (3)
Đến ta đặtz =x+yi xx2−+yiy2 =
1
2 (3) trở thành :z+ 3−i
z =
Giải phương trình số phức ta nghiệm :z = 1−i z= +i
Từ ta suy hệ phương trình ban đầu có cặp nghiệm : x= 1, y = vàx= 2, y =
Như dựa vào việc xếp ẩn số thích hợp để đưa phương trình số phức ta dễ dàng tìm nghiệm hệ ban đầu thơng qua nghiệm phức
Bài tốn tương tự Giải hệ phương trình :
(
x+ xx2+2+yy2 = (1)
x+ 2xx2−+3yy2 = (2)
Hà nội 2007 :
(
x− 3x+10y
x2+y2 = (1)
(63)Bài toán 15 Giải hệ phương trình :
2x1 + x2+1y2
= (1) 2y
1 + x2+1y2
= (2)
Lời giải Tương tự toán ta nhân (2) với cộng với (1) ta :
2(x+yi) + x−yi
x2+y2 = +i(3)
Đặtz =x+yi phương trình (3) trở thành:2z+2
z = +i
Phương trình có nghiệm: z = +ivà z = 12 − 2i
Do hệ phương trình ban đầu có cặp nghiệm: x=y= vàx= 12, y =−1
Bài toán 16 ( Moldova TST 2011) Giải hệ phương trình :
(
x+y+ = 12xx2+11+y2y(1)
y−x+ = 11xx2−+12y2y(2) Lời giải Nhân (2) với cộng với (1) ta :
x+yi+y−xi+ + 3i= 12x−yi
x2+y2 + 11
y+xi x2+y2 (3)
Đặtz =x+yi phương trình (3) trở thành:z(z−iz+ + 3i) = 12 + 11i
Phương trình có nghiệm :2 +i −5 −
9 2i
Do hệ phương trình ban đầu có cặp nghiệm: x= 2, y = x=−5
2, y =−
Từ ví dụ ta nhận thấy hệ phương trình có đại lượng x2 +y2 nằm mẫu số
ở hai phương trình hệ nên nghĩ đến phương pháp sử dụng số phức để tận dụng tính chấtz.z¯=|z|2 số phức liên hợp modul số phức để giải Việc cịn lại giải phương trình số phức sau biến đổi từ hệ phương trình ban đầu
Bài tốn 17 Giải hệ phương trình : (
x2+x−y2+ x x2+y2 =
3 2(1) 2xy+y− x2+yy2 =
5 (2)
Lời giải Vẫn ví dụ trước ta thu :
x2+ 2xyi−y2+x+yi+ x−yi
x2+y2 = 2+
5 2i (3)
Đặtz =x+yi phương trình (3) trở thành:
z2+z+1
z =
3 +
5 2i
(64)Bài toán 18 Giải hệ phương trình : (
x2−y2+ x2−y2
(x2+y2)2 +x+x2+xy2 =−
3 2xy− 2xy
(x2+y2)2 +y−
y
x2+y2 =−1
Lời giải Hướng dẫn : Đặt z =x+yi hệ phương trình dẫn đến phương trình số phức :
z2 +
z2 +z+
z =−
3 2−i
Bài tốn 19 Giải hệ phương trình : (
cos 2x+ cos 2y+2+cos 24 cosx−xcos 2y = 1+
√
3 cosx.siny− 2+cos 2sinx−ycos 2y =
√
3−1
Lời giải Hướng dẫn : Đặt z = cosx+isiny, hệ phương trình dẫn đến phương trình số phức :
z2+
z =
1 +√3 +
√
3−1
5 Các dạng khác
Bài toán 20 (Indian TST 2005) Cho a, b, c, d số thực không đồng thời Cho f : R→R
f(x) = a+bcos 2x+csin 5x+dcos 8x
Giả sử có số thựct chof(t) = 4a Chứng minh tồn số thựcs chof(s)<0
Lời giải Cho g(x) =be2ix −ice5ix+de8ix. Khi đó f(x) =a+Reg(x).
Để ý
g(x) +g
x+2π
+g
x+4π
=g(x)
1 +e2πis +e
4πi s
=
Do đóf(x) +f x+23π+f x+ 43π= 3a
Nếua <0, lấy s=t, a= số hạng có số âm, a >0, thay x=t
trong đẳng thức vàf(t) = 4a, ta có f t+ 2π
3
+f t+ 4π
3
=−a <0
Vậy f t+23π hay f t+ 43π Bài toán chứng minh xong
Bài toán 21 Cho z1, z2, z3 sơ phức phân biệt có modun nhau, khơng phải số thực
(65)Lời giải Cho zj =r(costj+isintj), với r 6= với rj ∈(0, π), j = 1,2,3
Với giả thiết
sint1+rsin(t2+t3) = sint2+rsin(t3+t1) = sint3+rsin(t1+t2) =
Đặtt =t1+t2+t3, sintj =−rsin(t−tj) =−rsintcostj −rcostsintj, với j = 1,2,3
Có nghĩa là:costjsint= 1r −costj, j = 1,2,3
Nếusint 6= 0thì cost1 = cost2 = cost3 Chỉ có giá trị xảy t1, t2, t3 nhận giá
trị 2π Điều dẫn tới sint = mặt khác ta lại có rcost = nên ta có cost =
hoặc −1 Mà điều xãy cost = r= nên ta có đpcm
Bài toán 22 (Putnam 2006) Cho n số nguyên dương lẻ θ số thực cho θn số vô tỉ
Đặt ak = tan θ+kπn
, k= 1,2, , n
Chứng minh : a1+a2+ +an
a1.a2 an số nguyên tìm giá trị nguyên Lời giải Xét số phức có dạng ω=cosθ+ i sinθ.Ta phải chứng minh
1 + ix
1−ix
n
=ω2n
Có nghiệm ak = tan θ+kπn
, k= 1,2, , n
Viết lại đa thức bậc n, ta
0 = (1 + ix)2−ω2n(1−ix)n
= (1−ω2x) +ni(1 +ω2x)x+ +in−1(1−ω2x)x+in(1 +ω2x)xn
Tổng nghiệm theo khải triển
−nin−1(=a1−ω2x)
in(1 +ω2x)
và tích chúng −(1−ω
2x) in(1+ω2x)
Do đó: a1+a2+ an
a1a2 an =n.i
n−1 =n(−1)(n−21). Đpcm
Bài toán 23 Cho số nguyên dương n hàm f:F(n) = xnsin(nA) +ynsin(nB) +znsin(nC),
x, y, z, A, B, C số thực A +B + C = kπ với k nguyên Chứng minh rằng,
F(1) =F(2) = 0, F(n) = 0, với số dương n
Lời giải Chọn số phức cho p =x.eiA, q =y.eiB, r =z.eiC và f(n) = pn+qn+zn.
ĐặtF(n) =Im(f(n)) Ta chứng qui nạp f(n)là số thực với mọin, tức làF(n) =
(66)sửf(k)là thực cho tất cảk ≤nvà n≤3, ta chứng minh f(n+ 1) số thực Chú ý rằng, a =p+q+r = f(1), b =pq+qr+rp = 21(f(1)2−f(2)), và c=pqr =xyz.ei(A+B+C)
là số thực Các sốp, q, r nghiệm tam thức P(t) =t3−at2+bt−c
Dùng
f(n+ 1) =pn+1+qn+1+rn+1
=a(pn+qn+rn)−b(pn−1+qn−1+rn−1) +c(pn−2+qn−2+rn−2) =af(n)−bf(n−1) +cf(n−2)
Khi f(n), f(n−1) f(n−2)là số thực, theo phương pháp qui nạp ta có f(n+ 1) số thực Ta có đpcm
Tài liệu tham khảo
[1] R Gelca and T Andresscu, 2007,Putnam and beyond, Springer
(67)MỘT SỐ ỨNG DỤNG CỦA PHƯƠNG TRÌNH PELL
Nguyễn Thị Tình, Trường THPT Lý Thường Kiệt, TX Ayun Pa, Gia Lai
Các phương trình Diophant đóng vai trò quan trọng lý thuyết số học Từ thời Trung cổ, phương trình nhà toán học ấn Độ quan tâm nghiên cứu đạt kết sâu sắc, chẳng hạn tìm dạng nghiệm tổng quát phương trình Diophant tuyến tínhax+by =c từ năm 499
Trong phương trình Diophant bậc hai, có dạng phương trình bậc hai tắc quan trọng x2−dy2 = x2−dy2 =−1, d số ngun dương khơng phương
Phương trình thường gọi phương trình Pell, lấy theo tên nhà toán học người Anh John Pell (1610 - 1685) mà theo câu chuyện toán học truyền lại, nhầm lẫn nhà tốn học thiên tài Euler tìm cơng lao người khai mở phương trình
Cho đến nay, Việt Nam có số tài liệu, sách tham khảo có đề cập cách trọn vẹn phương pháp giải phương trình Pell số vấn đề liên quan GS Nguyễn Văn Mậu (chủ biên), GS Hà Huy Khoái, GS Đặng Hùng Thắng,
Trên sở tài liệu sưu tầm được, xin giới thiệu lại số vấn đề nhất, dành cho hệ phổ thơng chun tốn, phương trình kinh điển Trong đó, khơng q sâu vào lý thuyết phương trình Pell, điều có số tài liệu nêu
1 Một số ví dụ dẫn đến phương trình Pell
Từ lâu đời, mà chưa có cách giải phương trình Pell, tốn học có nhiều câu hỏi tự nhiên đặt Để trả lời vấn đề nhiều lại dẫn đến yêu cầu tìm tất số nguyên dương thỏa mãn phương trình bậc có hai biến Có nhiều tốn mà nhiều khơng thấy có liên quan đến phương trình Pell, phép biến đổi đại số lại dẫn đến u cầu chung, tìm nghiệm nguyên dương phương trình x2−dy2 =n, d khơng phải số phương, n số nguyên Chẳng hạn ta có kết đơn giản là1 + + +· · ·+ 14 = 15 + 16 + 17 + 18 + 19 + 20
hoặc + + + = 3.6 Như vấn đề tự nhiên đặt liệu cặp số ngun dương (m, n) có tính chất hay khơng? Chúng tơi xét ví dụ sau đây: Ví dụ Xác định số nguyên dương m n cho
(68)Ta có
(1) ⇔ (m+n)(n−m+ 1)
2 =mn
⇔n2+n−m2 +m = 2mn
⇔(n−m)2−2m2+n+m=
Đặtu=n−m, ta có
u2−2m2+u+ 2m= ⇔u2−u−2(k2−k) =
⇔
u+1
2 −
4 −2
"
m−
2
2 −
2
#
=
⇔(2u+ 1)2−2(2m−1)2 =−1
Đặtx= 2u+ = 2(n−m) + 1; y= 2m−1, ta phương trình
x2−2y2 =−1 (2)
Như yêu cầu toán đặt giải phương trình (2) giải Ví dụ Tìm tất số nguyên dương k, m cho k < m
1 + +· · ·+k= (k+ 1) + (k+ 2) +· · ·+m
Giả sử k, m(k < m) hai số nguyên dương thỏa mãn hệ thức:
1 + +· · ·+k= (k+ 1) + (k+ 2) +· · ·+m (3) Ta có
(3) ⇔2(1 + +· · ·+k) = + +· · ·+k+ (k+ 1) + (k+ 2) +· · ·+m
⇔k(k+ 1) = m(m+ 1)
⇔2k2+ 2k=m2 +m
⇔8k2+ 8k= 4m2+ 4m
⇔2(2k+ 1)2−1 = (2m+ 1)2
Đặtx= 2m+ 1; y= 2k+ 1, ta có phương trình
x2−2y2 =−1 (4)
Như vấn đề đặt giải phương trình (4) giải
(69)Ví dụ Tìm số ngun dương m để tam giác với cạnh
1 2(m
3+m2)−1;1 2(m
3−m2) + 1;m2 hoặc m3−
2(m−1);m 3−
2(m+ 1);m có diện tích số
chính phương
Gọi 2p, S chu vi diện tích tam giác 1)Trường hợp cạnh tam giác
a= 2(m
3+m2)−1; b = 2(m
3−m2) + 1; c=m2, ta có
p= 2(m
3+m2); p−a= 1; p−b =m2−1; p−c= 2(m
3−m2)
Theo công thức Hêrơng, ta có
S =pp(p−a)(p−b)(p−c) =
r
1 4(m
3+m2)(m2−1)(m3−m2)
= 2m
2(m2−1)
2)Trường hợp cạnh tam giác
a=m3−
2(m−1); b =m 3−
2(m+ 1); c=m , ta có
p=m3; p−a=
2(m−1); p−b=
2(m+ 1); p−c=m 3−m
S=pp(p−a)(p−b)(p−c) =
r
1 4m
3(m−1)(m+ 1)(m3−m)
= 2m
2(m2−1)
Để tam giác với số đo cạnh cho có diện tích số phương
1 2(m
2−1) là số phương, tức là
2(m
2−1) =n2 ⇔m2−1 = 2n2 ⇔m2−2n2 = 1
Chúng ta thấy điều kiện để tam giác có cạnh cho trước có diện tích số phương phụ thuộc vào việc tìm nghiệm nguyên dương phương trình
m2−2n2 =
Một số vấn đề khác, ta biết √2 số vô tỉ Một chứng minh thường gặp cho khẳng định phương pháp phản chứng Chứng minh sau đây, dài, thú vị dẫn đến phương trình Pell
(70)Chứng minh Ta cần chứng minh √2 biểu diễn thành liên phân số vô hạn Xét dãy số
r1 =
rn= +
1 +rn−1
, n = 2,3,4, Ta dễ dàng nhận thấy với giá trị n, rn
được biểu diễn dạng pn
qn
, với (pn, qn) = n tăng rn tăng n lẻ
rn giảm khin chẵn
Ta chứng minh:
1≤r1 ≤r3 ≤ · · · ≤r2k+1 ≤ · · · ≤r2l ≤ · · · ≤r4 ≤r2 ≤
2 (5)
|rn+1−rn| ≤
1
4|rn−rn−1| (6)
1.Chứng minh(5)
Trước hết ta chứng minh
1≤r1 ≤r3 ≤ · · · ≤r2n+1 ≤ · · · (7)
Ta có r3−r1 =
1 −1 = >0
Giả sử (7) với số tự nhiênn=k bất kỳ, tức ta có r2k+1−r2k−1 >0 Ta cần chứng
minhr2(k+1)+1−r2(k+1)−1 >0
r2k+3+r2k+1 = + 1 +r2k+2
−1−
1 +r2k
= r2k−r2k+2 (1 +r2k+2)(1 +r2k)
=
1 + 1 +r2k−1
−1−
1 +r2k+1 (1 +r2k+2)(1 +r2k)
= r2k+1−r2k−1
(1 +r2k+2)(1 +r2k)(1 +r2k−1)(1 +r2k+1)
>0
(bất đẳng thức theo giả thiết quy nạp)
Vậy 1≤r1 ≤r3 ≤ · · · ≤r2n+1 ≤ · · ·, với n nguyên dương
Chứng minh tương tự phương pháp quy nạp bất đẳng thức ta có:
3
2 ≥r2 ≥r4 ≥ · · · ≥r2n≥ · · · r2n−1 ≤r2n, với n nguyên dương
Vậy với cặp số nguyên dương(k;l) ta có:
1≤r1 ≤r3 ≤ · · · ≤r2k+1 ≤ · · · ≤r2l ≤ · · · ≤r4 ≤r2 ≤
(71)Ta có
|rn+1−rn|=
1 + 1 +rn
−1−
1 +rn−1
= |rn−rn−1| (1 +rn)(1 +rn−1)
≤ |rn−rn−1|
4√rnrn−1
≤
4|rn−rn−1|
Gọi α cận nhỏ tập hợp phần tử {r1, r3,· · · , r2k+1,· · · },
khi r2m−1 ≤α≤r2m
Ta có
|α−r2m−1|<|r2m−r2m−1| ≤
42(m−1)|r2−r1|= 24m−3
|α−r2m|<|r2m−r2m−1| ≤
42(m−1)|r2−r1|= 24m−3
suy lim
n→∞rn= limn→∞rn−1 =α suy α= +
1
1 +α hay α
2 = 2
⇒α =√2 Vậy √2 biểu diễn dạng liên phân số vô hạn nên√2 số vô tỉ Định nghĩa Các số1,3,6,10,15,21,28,36,45, ,
tn=
1
2n(n+ 1), gọi số tam giác
Về mơ hình hình học, số tam giác phân bố cạnh tam giác, thể hình vẽ 1.2
Nhận xét rằng, tổng số tam giác kề số phương Thật vậytn+tn+1 =
1
2n(n+ 1) +
2(n+ 1)(n+ 2) = (n+ 1)
Một cách tự nhiên nảy sinh liệu có tìm tất số tam giác số phương hay khơng Chúng ta xét ví dụ sau
Ví dụ Tìm số tam giác số phương
Từ dãy số liệt kê trên, số 36 số phương Một cách tổng quát, ta cần xác định giá trị n∈N∗ sao chot
n số phương, hay ta tìm nghiệm nguyên dương
của phương trình:
2n(n+ 1) =m 2.
Dễ thấy phương trình tương đương với phương trình:
(2n+ 1)2−8m2 =
Như việc tìm số tam giác số phương quy tốn tìm nghiệm nguyên dương phương trình :
(72)Cũng liên quan đến số tam giác, vấn đề đặt liệu có hay khơng ba số tam giác liên tiếp mà tích chúng tổng chúng số phương Vấn đề đề cập hai ví dụ sau
Ví dụ Tìm ba số tam giác liên tiếp cho tích chúng số phương Giả sử số tam giác liên tiếp có dạng
2(n−1)n,
2n(n+ 1),và
2(n+ 1)(n+ 2) Ta có
2(n−1)n·
2n(n+ 1)·
2(n+ 1)(n+ 2) = 22 ·
1 2n
2(n−1)(n+ 1)2(n+ 2).
Để tích số tam giác liên tiếp số phương
2(n−1)(n+ 2) phải
số phương, tức
1
2(n−1)(n+ 2) =m
⇔ (n−1)(n+ 2) = 2m2
⇔ (2n+ 1)2−8m2 = 9, m∈Z Đặtx= 2n+ 1; y=m, ta phương trình
x2−8y2 = (8)
Như toán làm sáng tỏ phương trình(8) giải
Ví dụ Tìm ba số tam giác liên tiếp cho tổng số số phương Định nghĩa Với số nguyênn k, ta định nghĩa
n k
= n!
k!(n−k)!
Như liên quan đến toán tổ hợp, vấn đề đặt là:
Ví dụ Có tồn số ngun a, bkhơng âm cho
a b
=
a−1
b+
(9) Ví dụ Giả sử có n viên bi lọ, có rviên bi màu đỏ n−r viên bi màu xanh Lấy viên bi cách ngẫu nhiên Biết rằng, xác suất để lấy viên bi màu
2 Xác định giá trị có n r
(73)Bổ đề Với số nguyênk, m, n, số sau ba số Pitago:
k(m2−n2),2kmn, k(m2+n2) (10) Hệ Với k = 1, m=y, n=z ba
(y2−z2,2yz, y2+z2) (11)
là ba số Pitago
Ví dụ 10 Ta biết 22+ 42+ 62+ 82+ 102 = 4.5 + 5.6 + 6.7 + 7.8 + 8.9.
Vấn đề đặt tìm ví dụ khác, tương tự, nghĩa là: "Tìm đẳng thức mà vế tổng bình phương củan số chẵn vế tổng củan cặp số nguyên liền kề" Qua số ví dụ trình bày trên, nhận thấy có nhiều vấn đề đơn giản, lại tổng quát hoá thành toán khó thú vị Việc giải số ví dụ phụ thuộc vào việc giải phương trình dạng:
x2 −dy2 = (12)
hoặc
x2 −dy2 =−1 (13)
hoặc
x2−dy2 =n, (14)
ở đâyd số nguyên dương, số phương; n số nguyên
2 Tìm nghiệm ngun dương phương trình.
Bài tốn Tìm nghiệm nguyên dương phương trình
x2−6xy+y2 =
Giải
Ta có x2−6xy+y2 = 1⇔(x−3y)2−8y2 = Đặt u=x−3y, ta phương trình:
u2−8y2 = (15)
Nghiệm nguyên dương nhỏ phương trình (15) là(a;b) = (3; 1)nên nghiệm phương trình (15) cho dãy sau:
u0 = 1;u1 = 3;un+2 = 6un+1−un
y0 = 0;y1 = 1;yn+2= 6yn+1−yn
(74)Màu=x−3y nên x=u+ 3y số nguyên dương u, y nguyên dương Do
xn+2 =un+2+ 3yn+2
= 6un+1−un+ 3(6yn+1−yn)
= 6(un+1+ 3yn+1)−(un+ 3yn)
= 6xn+1−xn
Theo định lí (??) nói cơng thức nghiệm phương trình Pell loại I, ta có nghiệm phương trình cho có nghiệm ngun dương thoả mãn hệ thức sau đây:
x0 = 1;x1 = 6;xn+2 = 6xn+1−xn
y0 = 0;y1 = 1;yn+2 = 6yn+1−yn
, n = 0,1,2,· · ·
Bài tốn Tìm nghiệm ngun dương phương trình
(x−1)2+ (x+ 1)2 =y2+
Giải Nếu dùng phương pháp xây dựng nghiệm chứng minh phương trình có vơ số nghiệm nguyên dương, biến đổi phương trình phương trình Pell tất nghiệm nguyên dương lời giải gọn gàng
Ta có (x−1)2+ (x+ 1)2 =y2+ ⇔2x2+ =y2+
⇔y2−2x2 = (16)
Phương trình (16) phương trình Pell loại I, có nghiệm ngun dương nhỏ (a;b) = (3; 2).Vậy theo định lí (??) ta có nghiệm phương trình (16) phương trình cho là:
x0 = 0;x1 = 2;xn+2 = 6xn+1−xn
y0 = 1;y1 = 3;yn+2 = 6yn+1−yn
, n = 0,1,2,· · ·
Bài tốn Tìm số ngun dương (x, y, z, w) thỏa mãn x2 +y2 +z2 = w2 sao cho x =
y;z =x±1
3 Tính tổng số nguyên liên tiếp.
Bài tốn Tìm tất số ngun dương k, m cho k < mvà
1 + +· · ·+k= (k+ 1) + (k+ 2) +· · ·+m
Giải Giả sửk, m(k < m) hai số nguyên dương thỏa mãn hệ thức:
(75)Ta có:
(17))⇔2(1 + +· · ·+k) = + +· · ·+k+ (k+ 1) + (k+ 2) +· · ·+m
⇔k(k+ 1) = m(m+ 1)
⇔2k2+ 2k=m2+m
⇔8k2+ 8k= 4m2+ 4m
⇔2(2k+ 1)2−1 = (2m+ 1)2
Đặtx= 2m+ 1;y = 2k+ 1, ta có phương trình Pell loại II:
x2−2y2 =−1 (18)
Liên kết với (18) phương trình Pell loại I:
x2−2y2 = (19)
Phương trình Pell loại I có nghiệm nguyên dương bé
(x, y) = (3,2)
Xét hệ phương trình sau:
u2+ 2v2 =
2uv = (20)
Dễ thấy (u, v) = (1,1) nghiệm nguyên dương bé hệ (20) Theo lý thuyết xây dựng dãy phương trình Pell loại II có nghiệm
x0 = 1;x1 = 7;xn+2 = 6xn+1−xn
y0 = 1;y1 = 5;yn+2 = 6yn+1−yn
, n = 0,1,2,· · ·
Ta thấy xk ≡ (mod 2);yk ≡ (mod 2), với k = 0,1,· · ·.Từ suy dãy nghiệm
mi =
xi−1
2 ;ki =
yi−1
2 cho công thức:
m0 = 0;m1 = 3;mi+2 = 6mi+1−mi+
k0 = 0;k1 = 2;ki+2= 6ki+1−ki+ 2,
với i= 0,1,2,· · · Bốn đáp số là:
(m, k) = (3,2); (20,14); (119,84); (696,492)
Bài toán Xác định số nguyên dươngm vàn cho
m+ (m+ 1) +· · ·+n =mn (21) Bài tốn Ta có 22+ 42+ 62+ 82 + 102 = 4.5 + 5.6 + 6.7 + 7.8 + 8.9
(76)4 Số tam giác tính chất số tam giác
Bài tốn Tìm số ngun dươngnsao cho số tam giác
2n(n+ 1)là số phương
Giải
Ta có
n(n+ 1) =y
2 ⇔n(n+ 1) = 2y2
⇔4n2 + 4n+ = 8y2 +
⇔(2t+ 1)2−8y2 =
Đặt x= 2n+ (x;y) nghiệm phương trình:
x2−8y2 = (22)
Ngược lại (x;y) nghiệm phương trình (22) x lẻ nên n = x−1
2 thỏa mãn đề
bài
Nghiệm nhỏ phương trình (22) (3; 1) nên phương trình có nghiệm dãy
x0 = 1;x1 = 3;xi+2 = 6xi+1−xi
y0 = 0;y1 = 1;yi+2= 6yi+1−yi
, i= 0,1,2,· · ·
Khi vớixi = 2ni+ 2ni+2+ = 6(2ni+1+ 1)−(2ni+ 1) Dãy(ni)cần tìm xác định
bởin0 = 0;n1 = 1;ni+2 = 6ni+1−ni+
Đó số 1; 8; 49; 288;· · ·
Bài tốn Tìm ba số tam giác liên tiếp cho tích chúng số phương Bài tốn Tìm ba số tam giác liên tiếp cho tổng số số phương
Bài toán 10 Xác định số nguyên dương b cho số (111· · ·1)b, gồm k chữ số 1, với b số, số tam giác với giá trị củak
5 Tìm số phương
Một áp dụng quan trọng tốn học có nhiều tốn liên quan đến số phương Việc giải phương trình Pell giúp tìm số phương thoả mãn yêu cầu cho trước
Bài tốn 11 Tìm tất số ngun dương t cho tổng t số nguyên dương số phương
(77)Bài tốn 12 Tìm tất số ngun dươngnsao cho trung bình cộng củansố phương số phương
Bài tốn 13 Tìm số nguyên dương m để tam giác với cạnh :
1 2(m
3+m2)−1;1 2(m
3−m2) + 1;m2 hoặc m3−
2(m−1);m 3−
2(m+ 1);m có diện tích số
chính phương
Bài tốn 14 Tìm tất số ngun dương n có tính chất n2 + (n+ 1)2
là số phương
Bài tốn 15 Chứng minh tồn vô hạn số nguyên dương n cho n! chia hết chon2+ 1.
Giải
Xét phương trình Pell loại II :
x2−5y2 =−1 (23)
Phương trình Pell loại I liên kết với (23) phương trình sau:
x2−5y2 = (24)
Phương trình (24)có nghiệm ngun dương nhỏ (9; 4) Xét hệ phương trình
u2+ 5v2 = 9 2uv =
Hệ có nghiệm nguyên dương nhỏ là(u, v) = (2,1) Vì phương trình (23) có dãy nghiệm là:
x0 = ;x1 = 38 ; xn+2= 18xn+1−xn; n= 0,1,2,· · ·
y0 = ;y1 = 17 ; yn+2 = 18yn+1−yn;n = 0,1,2,· · ·
Các dãy nghiệm có tính chất 5< yk<2yk < xk,với k = 1,2,· · ·
Thật vậy, ta có 5< yk<2yk Do yk >5nên 4yk2 <5y2k−1 = x2k suy 2yk< xk
Do 5< yk<3yk < xk, ∀k ∈N∗
Vì (xk)! = 1.2.3.4.5· · ·yk· · ·(2yk)· · ·(xk)
Từ suy (xk)! 5yk(2yk) = 2(xk2+ 1) nên xk (x2k+ 1),∀k ∈N
∗. Bài toán chứng minh
Bài toán 16 Xét dãy số{un}
∞
n=0 =
q
n2+ (n+ 1)2
∞
n=0
Chứng minh tồn vô hạn sốn cho ta có đồng thời
un−un−1 >1; [un+1]−[un] =
Bài tốn 17 Chứng minh phương trình 5x−3y = 2 có nghiệm dương là
(78)Tài liệu tham khảo
[1] Phan Huy Khải, "Các chuyên đề số học bồi dưỡng học sinh giỏi tốn trung học ", Phương trình nghiệm nguyên, NXB Giáo dục
[2] Phan Huy Khải(2004), "Chuyên đề bồi dưỡng học sinh giỏi toán trung học phổ thơng ", Các tốn số học, NXB Giáo dục
[3] Hà Huy Khoái, "Chuyên đề bồi dưỡng học sinh giỏi tốn trung học phổ thơng",Số học, NXB Giáo dục
[4] Nguyễn Văn Mậu (chủ biên), Một số vấn đề số học chọn lọc, NXB Giáo dục [5] Đặng Hùng Thắng (1995), Bài giảng số học, NXB Giáo dục
[6] Đặng Hùng Thắng, Nguyễn Văn Ngọc, Vũ Kim Thủy(1997), Bài giảng số học,Tuyển tập 30 năm tạp chí tốn học tuổi trẻ, NXB Giáo dục
[7] Vũ Dương Thụy (chủ biên)(2006), Tuyển tập 40 năm Olympiads Toán học quốc tế(IMO 1959-2000),NXB Giáo dục
(79)PHÉP THẾ LƯỢNG GIÁC LÀ CƠNG CỤ GIẢI TỐN TRONG CÁC BÀI THI CHỌN HỌC SINH GIỎI
Huỳnh Bá Lộc - Sở Giáo dục Đào tạo Khánh Hòa
Việc chọn phép lượng giác linh hoạt số lớn công thức lượng giác làm cho toán đơn giản Phép thường cho dạng biểu thức đại số, việc lượng giác hóa tốn xét nhiều dạng toán thường gặp sau
1 Chứng minh đẳng thức
Bài toán (IMO 1985) Cho x, y, z ∈R, cho x+y+z =xyz Chứng minh
x(1−y2)(1−z2) +y(1−z2)(1−x2) +z(1−x2)(1−y2) = 4xyz
Lời giải Rõ ràng đẳng thức vớixyz = 0,nên cần chứng minh vớix, y, z 6=
Chia vế cho4xyz ta có
1−y2 2y
1−z2 2z +
1−z2 2z
1−x2 2x +
1−x2 2x
1−y2 2y =
Từ điều kiệnx+y+z =xyz ta nghĩ đến việc lượng giác hóa tốn cách
Đặtx= tanA, y= tanB, z = tanCvớiA, B, C góc tam giác, ta đưa toàn trở thành
cot 2Bcot 2C+ cot 2C.cot 2A+ cot 2A.cot 2B =
⇔tan 2A+ tan 2B + tan 2C = tan 2A.tan 2B.tan 2C
Đây rõ ràng đẳng thức vìtan(2A+ 2B+ 2C) = tan 2π=
Bài toán chứng minh
Bài toán Cho a, b, c số thực khác ±√1
3 Chứng minh abc =a+b+ckhi 3a−a3
3a2−1 −
3b−b3 3b2−1−
3c−c3 3c2−1 =
3a−a3 3a2−1+
3b−b3 3b2−1 +
3c−c3 3c2−1
Lời giải Đặt a= tanx; b= tany;c= tanzvớix, y, z ∈ 0;π2.Từ đẳng thức
tan(x+y+z) = tanx+ tany+ tanz−tanx−tany−tanz
x−tanxtany−tanytanz−tanxtanz
Ta thấyabc =a+b+c x+y+z =kπ(k ∈Z)
Suy tan(3x+ 3y+ 3z) = tan 3kπ=
Hay
tan 3xtan 3ytan 3z = tan 3x+ tan 3y+ tan 3z
⇔ 3a−a
3 3a2−1
3b−b3 3b2−1
3c−c3 3c2−1 =
3a−a3 3a2−1 +
3b−b3 3b2−1+
(80)Bài toán Chứng minh Σn
k=0
−1
k
cos3 3k−ππ
= 34h −1
n+1
+ cos3 π
3n
i
Lời giải Từ đẳng thức cos 3x= cos3x−3 cosx,ta có cos3x=
4(cos 3x+ cosx)
Suy n Σ k=0 −1 k
cos3 3ka= n Σ k=0 " −1 k
cos 3k+1a−
−1
3
k−1
cos 3ka
#
Cho a= 3−nπ ta kết toán Bài toán Chứng minh
27 sin390+ sin2270+ sin3810 + sin32430 = 20 sin 90
Lời giải Từ sin3x= 14(3 sinx−sin 3x) ta có
273 sin
0−sin 270 +
3 sin 270−sin 810 +
3 sin 810−sin 2430
4 +
3 sin 2430−sin 7290
= 81 sin
0−sin 7290
4 =
81 sin 90−sin 90
4 = 20 sin
Bài toán Tính (1−cot 10)(1−cot 20) .(1−cot 440).
Lời giải Ta có ((1−cot 10)(1−cot 20) .(1−cot 440) =
1− cost
0
sint0 1− cos 20 sin 20
1− cos 44
0 sin 440
= (sint
0−cost0) (sin 20−cos 20) sin 440−cos 440
sint0sin 20 sin 440
Dùng đẳng thứcsina−cosa=√2 sin a−450 ta đưa √
2 sin 10−450√2 sin 20−450 √2 sin 440−450 sin 10sin 20 sin 440
=
√
244(−1)44sin 440sin 430 sin 20sin 10 sin 440sin 430 sin 20sin 10 =
22
Bài toán Chứng minh
a) −cos
π
7
1
2 −cos 3π
7
1
2 −cos 9π
7
=−1
(81)Lời giải
a) Để ý rằng1−2 cos 2x−1−2(2 cos2x−1) = 3−4 cos2x= −cos 3x
cosx
Tích biến đổi lại thành
−1
2
3cos3π
7 cosπ7
cos9π
7 cos3π
7
cos27π
7 cos9π
7
=−1
8
cos27π
7 cosπ7 =−
1
b) Ta có1 + cos 2x= + 2(1−2 sin2x) = 3−4 sin2x= sin 3sinxx
Tích viết lại thành
1 24
sin320π sin20π
sin920π sin320π
sin2720π sin920π
sin8120π sin2720π =
1 16
sin8120π sin20π =
1 16
Bài toán Chứng minh
a) Π24
n=1
cos (2n)0 =−224.tan 20 b)
25 Π
n=2
2 cos (2n)0− cos (2n)0
=−1
Lời giải
a) Ta có cos1x = 2 sin2 sinxcosx x = 2sin 2sinxx
Áp dụng ta
24 Π
n=1
cos (2n)0 =
24 24 Π
n=1
sin (2n)0 sin (2n+1)0 =
24 sin 20 sin (225)0
Ta cần chứng minh sin(225)0 = cos 20 hay cần chứng minh
225−2−90 180 ⇔223−23 45 = 5.9
Ta có 2(22)11−3 = 2(−1)11−3 = (mod 5)
4(23)7−5 = 4(−1)7 −5 = 0(mod 9)⇒223−23 .45⇒ đpcm.
b) Ta có cosx− cosx =
2cos2x−1 cosx =
cos 2x
cosx Suy 25 Π n=2
2 cos (2n)0−
cos (2n)0
= 25Π
n=2
cos (2n+1)0 cos (2n)0 =
cos (226)0 cos 40
Vậy ta cần chứng minh
cos(226)0 =−cos 40 ⇔264−4 180 226−
360
Vì226−4 = 4(224−1)nên226−4chia hết cho264−1và chia hết cho 26−1 Từ ta có
(82)2 Chứng minh bất đẳng thức - Tìm giá trị lớn nhất, giá trị nhỏ nhất
Bài toán Chứng minh
−1
2
(x+y) (1−xy) (1 +x2) (1 +y2)
1
Lời giải Đặt x= tana y= tanb Ta có −16sin (a+b)61
Bài tốn (USA MO 2002) Tìm GTLN biểu thức
S = (1−x1)(1−y1) + (1−x2)(1−y2)
Với x12+x22 =y12+y22 =c2 với c >0
Lời giải Ta thấy điểm có tọa độ (x1;x2) (y1;y2) nằm đường tròn (O;c) Ta
đặt (x1;x2) = (c.cosϕ;csinϕ)và (y1;y2) = (c.cosψ;c.sinψ) Khi
S = 2−c(cosϕ+ sinϕ+ cosψ+ sinψ) +c2(cosϕ.cosψ+ sinϕ.sinψ)
S= +c√2−sinϕ+ π
−sinψ+π
+c2cos (ϕ−ψ)
S 62 + 2c√2 +c2 =c+√22
Dấu “=’ xảy khiϕ=ψ = 54π
Vậy maxS = (c+√2)2 khi x
1 =x2 =y1 =y2 =
√
2 c
Bài toán 10 Chứng minh ∀a, b, c∈R, ta có |a−b|
√
1 +a2√1 +b2
|a−c| √
1 +a2√1 +c2 +
|b−c| √
1 +b2√1 +c2
Lời giải Đặt a= tanα; b = tanβ; c= tanγ; α, β, γ ∈ −π
2;
π
2
Khi đóa2+ =
cos2α; b2+ =
1
cos2β; c2+ =
1 cos2γ
Bất đẳng thức trở nên đơn giản
|sin (α−β)|6|sin (α−γ)|+|sin (β−γ)| Ta chứng minh bất đẳng thức sau
|sin (α−β)|=|sin (α−γ+γ−β)|
=|sin (α−γ)|cos(γ−β) + sin(γ−β) cos(α−γ)
6|sin (α−γ)| |cos(γ−β) + sin(γ−β)| |cos(α−γ)| 6|sin (α−γ)|+|sin(γ−β)|
(83)Bài toán 11 Cho a, b, c∈R Chứng minh
(ab+bc+ca−1)2 6 a2+ b2+ c2+
Lời giải Từ a2 + 1; b2 + 1; c2 + 1 ta nghĩ đến việc đặt a = tanu, b = tanυ và c = tanω với u, υ, w ∈ −π
2;
π
2
Bất đẳng thức trở thành
−16(tan u tan υ+tan υ tan w+ tanw tanu−1) cosu.cosυ.cosw61
(103 Trigonometry Problems) Bài toán 12 Chứng minh
x
√
1 +x2 +
y
p
1 +y2 +
z
√
1 +z2 3√3
2
Với x+y+z =xyz x, y, z∈R
Lời giải Đặt x= tanA, y= tanB, z = tanC với A, B, C góc ∆ Bài toán đưa sinA+ sinB+ sinC6
√
3
Bài toán giải dễ dàng BĐT Jensen vớif(x) = sinx, x∈ 0;π2
Bài toán 13 Chứng minh
x
1−x2 +
y
1−y2 +
z
1−z2 > 3√3
2
Với 0< x, y, z <1và xy+yz+xz =
Lời giải Nhân cho hai vế, BĐT trở thành
2x
1x2 + 2y
1−y2 + 2z
1−z2 >3
√
3
Từ điều kiện toánxy+yz+yz = 0< x, y, z <1,ta nghĩ đến việc đặtx= tanA2; y = tanB
2; z= tan
C
2 với A, B, C góc tam giác nhọn Bài toán đưa dạng tanA+ tanB+ tanC >3√3(đúng theo BĐT Jensen)
3 Giải phương trình - Bất phương trình
(84)Lời giải Đặt x= cost, t∈[0, π] ⇒√1−x2 = sint
Biểu thức trở thành:sint+ cost>a, mà sint+ cost=√2cos t− π
4
Vậy a không vượt √2
Bài toán 15 Cho số phân biệt khoảng (0; 1) Chứng minh tồn số x, y thỏa mãn
0< xp1−y2−y√1−x2 <
Lời giải Cho số = sinbi, góc bi phần tư thứ
Bài toán số k, m cho 0<sinbkcosbm−sinbmcosbk < 12
Vậy cần chứng minh tồn sốk, m cho bk> bm bk−bm < π6
Điều có nguyên lý Dirichlet có số số nằm khoảng(0,π6], (π6 , π6],(π6, π6]
Bài toán 16 Giải phương trình x3−3x=√x+ 2 trên tập số thực.
Lời giải Hệ số x3 là 1, biểu thức x3−3x làm ta liên tưởng 4 cos3x – 3 cosx
Ta thấy∀x >2 x3−3x >4x−3x=x >√x+ 2
Vậy −26x62 Ta đặt x= cosα với α∈[0;π]
Phương trình trở thành
2 cos 3∝=p2(1 + cos∝) = cos∝
2 ⇔2 sin 72
4 sin 52
4 =
Suy ∝= 0; 47π;45π Vậy x= 2, cos47π;−1 2(1 +
p
5)
Bài toán 17 (IMO Shortlisted 1995) Cho số thực dươnga, b, c Xác định tất số thực dương x, y, z cho
x+y+z =a+b+c 4xyz−(a2x+b2y+c2z) =abc
Lời giải Từ giả thiết ta có
4 = a
yz + b2 zx +
c2 xy +
abc xyz
Đặtx1 = √ayz, y1 = √bzx, z1 = √cxy, ta
4 =x21+y12+z12+x1y1z1
Với < x1; y1; z1 < Xét phương trình phương trình bậc theo biến x1, ta
có định thức 4−x21
4−y2
Điều gợi ý ta đặtx1 = sinu, y1 = sinv; 0< u, v < π2 Khi phương trình trở thành = 4sin2u+ 4sin2v+z21 + sinusinv.z1
⇔(z1+ sinusinv)2 = 4(1−sin2u)(1−sin2v)
(85)Vì z1, sinu, sinv số dương nên ta suy
z1 = 2(cosucosv−sinusinv) = cos(u+v)
Như ta
a= 2√yzsinu, b= 2√xzsinv, c= 2√xycos(u+v)
Từ giả thiếtx+y+z =a+b+cta : √
xcosv−√ycosu2
+ √xsinv−√ysinu−√z2
=
Suy √z =√xsinv+√ysinu=√xy1
2 +
√
yx1
2
Do đó√z =√x2√b
xz +
√
y2√a
yz ⇒z = a+b
2
Tương tự ta cóy = c+2a, x= b+2c
Rõ ràng (x, y, z) = b+2c,c+2a,a+2b thỏa mãn điều kiện đề nghiệm tìm
Bài tốn 18 Giải hệ phương trình
x3−3x=y y3−3y=z z3−3z =x
Lời giải Đặt x= cosu, y = cosv, z = cosw; u, v, w ∈[0, π]
Hệ phương trình cho ta cos 27u= cosu có nghiệm
u= kπ14, k = 0,1,· · · ,14, u= kπ13, k = 1,· · · ,12nên
x= coskπ14, y = cos314kπ, z = cos914kπ, k= 0,1,· · · ,14
Và x= coskπ13, y = cos313kπ, z= cos913kπ, k= 1,· · · ,12
Vì hệ phương trình cho có tối đa 27 nghiệm, nên 27 nghiệm nghiệm hệ Bài tốn 19 Giải hệ phương trình
2x+x2y=y
2y+y2z =z
2z+z2x=x
Lời giải Xét x= 1, x=−1,
Xétx6=±1 Đặtx= tana, đưa đến phương trình tan 8a= tana, a ∈(−π/2, π/2)
Kết nghiệm hệ (tankπ7 ,tan2kπ7 ,tan4kπ7 ), k=−3,−2,−1,0,1,2,3
Bài toán 20 Giải hệ phương trình
x1−1/x1 = 2x2
x2−1/x2 = 2x3
x3−1/x3 = 2x4
(86)Lời giải Liên hệ công thức cot 2a= cota− cota
Đặtx1 = cota, đưa đến phương trình cot 16a= cota, a ∈(0, π)
Kết nghiệm hệ
x1 =cotkπ15, x2 = t cot215kπ, x3 = cot417kπ, x4 = cot815kπ), k = 1,2,· · · ,14
Bài tốn 21 Giải hệ phương trình
3x−y x−3y =x
2 3y−z y−3z =y
2
3z−x z−3x =z
2
Lời giải Nếu x= 0⇒x=y=z = 0(loại)
Vậy x, y, z 6= Hệ phương trình viết lại thành
y = 31x−−3xx33
z = 31v−−3yy32
x= 31z−−3zz32
Đặtx= tanu(u∈ −π
2;
π
2
Suy x= tanu= tan 27u, suy u= kπ26.Với −π
2 <
kπ
26 <
π
2 hay k =±1;±2; ;±12
Vậy hệ phương trình có nghiệm
x= tankπ
26; y= tan 3kπ
26 ; z = tan 9kπ
26
Với k=±1;±2; ;±12
4 Lượng giác dãy số
Bài toán 22 (Romani TST 1986) Cho dãyan thỏa
√
an+2+ 26an 62, n >1 Tìm a1986
Lời giải Đặt an = cosbn, 06b6π/2, 06an62
Từ bất đẳng thức bên trái ta liên hệ công thứccos 2a+ = cos2a, ta có cosbn+2
2 6cosbn suy
ra bn+2
2 >bn⇒
bn+2k
2k >bn quy nạp, hàmcosgiảm
Vìk tiến đến vô hạn nên bn = n suy an = 2, n
Bài toán 23 (AIME 1996) Cho dãy xn thỏa x1 =x, xn+1 = 1−1xn − 1+1xn, ∀n ∈N
Khi xn= hay xn =−1 dãy kết thúc Hỏi có dãy có số hạng thế?
Đáp số: x = tan b, x8 = tan 128 b, Có 256 dãy thỏa điều kiện Bài toán 24 Cho dãy xn thỏa
xn+1 =
√
3xn−1
xn+
√
3 , n >1
(87)Lời giải Liên hệ công thức tan(a−b), đặt x1 = tant suy xn+6 =xn
Bài toán 25 Cho a0 =
√
2;b0 =
an+1 =
q
2−p
4−a2
n; bn+1 = 2bn 2+√4+b2
n
;n >0
a) Chứng minh dãy (an)n (bn)n giảm tiến
b) Chứng minh dãy (2na
n) tăng, dãy (2nbn) giảm hai dãy tiến đến giới hạn
Lời giải
a) Đặt an = sintn , ta có
2 sintn+1=an+1 =
q
2−p4−4 sint2
n=
√
2−2 costn= sin
tn
2
Suy tn+1 = tn2 với t = π4, từ đótn = 2nπ+2; n >0 Vậy an = sin2nπ+2 với n>0
Đặtbn= tanun (n >0, un ∈
0;π2)
Ta tanun+1 =bn+1 = tanun 2+
√
4+4tan2un =
4 tanun
2+ cosun
= 21+cossinunun = tanun2
Suy un+1 = vn2 n0 = π4, từ un= 2nπ+2n >0
Vậy bn = tan2nπ+2 với n>0
Trở lại toán ta thấy dãy(an)n (bn)n giảm liman =limbn=
b) Ta có hàm sinxx tăng tanxx giảm khoảng xét Do đó2na
n = π2
sin π
2n+2
π
2n+2
tăng 2nb n= π2
tan π
2n+2
π
2n+2
giảm Ta có lim
n−>∞
na
n = π2 lim n−>∞
sin π
2n+2
π
2n+2
= π2 lim
n−>0 sinx
x = π
2
Tương tự lim
n−>∞
nb n = π2
Bài toán 26 Cho dãy (xn) (yn)
(x1) = y1 =
√
3; xn+1 = 2n+
p
1 +x2
n; yn+1 = yn 1+√1+y2
n
; n >1
Chứng minh 2< xnyn<3 ∀n >1
Lời giải Đặt xn= tanan+
p
1 + tan2a
n= tanan+cos1an
= 1+sincosanan = tan9002+an
Vìa1 = 600, a2 = 750; a3 = 82,50 từ ta có an = 900− 30
0
2n−1
Nênxn = tan(900− 30
0
2n−1) = cot(
300
2n−1) = cotθnvới θn = 30
0
2n−1
Tương tự ta cóyn= tan 2θn = 12 tan−tan2θθnn
Suy xnyn= 1−tanz2θn
Vì00 < θn <450 nên 0<tan2θn<1 xnyn>2
Với n >1, ta cóθn<300 nên tan2θn< 13 suy xnyn<3.Vậy 2< xnyn<3
Bài toán 27 Với n >0, cho Un=arcsin
√
n+1−√n
√
n+2√n+1
(88)Lời giải Đặt bn = √n1+1, ta cósinun=
√
n+1−√n
√
n+2√n+1 =bn
p
1−b2
n+1−bn+1
p
1−b2
n
Vậy S= lim
n→∝
N
Σ
n=0
arcsin √1
n+1 −arcsin
√
n+2
=arcsin 1− lim
n→∝arcsin
1
√
N+2 =
π
2
Bài toán 28 (MOSP 1996) Cho dãy an không giảm [-1, 1] Chứng minh n−1
X
i=1
r
1−aiai+1±
q
(1−a2
i)(1−a2i+1)<
π√2
Lời giải Đặt = cosai = cosxi; xi ∈[0,π] , i= 1, , n
Biến đổi vế trái thành √2
n−1
P
i=1
sinxi+1−xi
2
Do hàm sin lồi xuống đoạn [0,π], nên áp dụng bất đẳng thức Jensen ta có √
2
n−1
X
i=1
sinxi+1−xi
2 6(n−1)
√
2 sin xn−x1 2(n−1)
√
2(n−1) sin π
2(n−1),doxn−x1 ∈(0;π)
Mà khix >0, ta lại có sinx < xnên ta có kết
Bài toán 29 (China MO 1996) Cho x0, xi >0, i= 1, , ncó tổng số hạng Chứng
minh n P k=1 √
1+x0+···+xk−1
√
xk+···xn
< π2
Lời giải Đặt x0+x1+· · ·+xk = sinak, k= 0,1,· · · , n, a0 = < a1 <· · ·< an =π/2 n
X
k=1
sinak−sinak−1
√
1 + sinak−1
√
1−sinak−1 =
n
X
k=1
2 sinak−ak−1
2 cos
ak+ak−1
2 cosak−1
< π
2
Hàmcosx giảm phần tư thứ sinx < xnên ta có kết
Bài tốn 30 Chứng minh ΠN
n=0
1−tan22−n = tan
Lời giải Ta có tan 2x= 12 tan−tan2xx = tan
Suy ΠN
n=0 1−tan22−n =
N
Π
n=0 tan 2 tan 2−n =
2−N
tan 2−N tan
Vì lim
x→0 tanx
x = nên
2−N
tan 2−N >1 ⇒đpcm
Tài liệu tham khảo
(89)SỬ DỤNG VÀNH CÁC SỐ NGUYÊN ĐỂ GIẢI MỘT SỐ BÀI TOÁN SỐ HỌC
Nguyễn Trung Hưng, Trường THPT Chuyên Lê Qúy Đơn – Khánh Hịa
Trong tập số nguyên, khái niệm chia hết khái niệm số nguyên tố hai khái niệm quan trọng Bài viết quan tâm đến việc giải số toán Số học tập số nguyên không đứng mà dựa việc xây dựng tính chất số học vành Nội dung sử dụng tính chất: “sự phân tích thành phần tử bất khả qui phần tử vànhZ[i] vành số nguyên Qh√di” để giải toán liên quan
1 Các khái niệm số tính chất
1.1 Nhắc lại lý thuyết vành
Định nghĩa Một vành tập hợp với hai phép tốn hai ngơi, phép cộng phép nhân thỏa: a)(R; +)là nhóm Abel; b)(R;.)là vị nhóm; c)(a+b)c=ac+bc, ∀a, b, c∈
K Vành R gọi vành giao hoán phép nhân có tính chấn giao hốn
Định nghĩa Một trường F vành giao hoán phần tử khác khơng có phần tử khả nghịch, tức với x∈F, x6= 0,tồn x−1 ∈F sao cho:
x.x−1 =
Định nghĩa Cho vành R Phần tử a ∈ R, a 6= gọi ước không tồn
a∈R, a6= cho: ab= ba= Vành giao hoán R gọi miền ngun khơng có ước khơng Từ ta có khái niệm chia hết miền nguyên R:
Định nghĩa Cho a, b∈R (b6= 0).Ta nói a chia hết cho b, kí hiệu a b (hoặcb|a)) tồn q ∈ R cho: a =b.q Hai phần tử a6= 0, b 6= đươc gọi liên kết, kí hiệu a ∼b , nếua|b b|a
Định nghĩa Một phần tử p 6= R gọi bất khả qui khác ước đơn vị khơng có ước thực R, tức p=ab với a, b∈ R a∼ p, b∼1
a∼1, b ∼p
(90)Định nghĩa Cho phần tử a1, a2, , an ∈R Một ước chung chúng phần tử
chia hết phần tử cho Ước chung d củaa1, a2, , an gọi ước chung lớn
chúng, kí hiệu d= (a1, a2, , an), chia hết cho ước chung a1, a2, , an Từ
ta rút ra, nếud= (a1, a2, , an),thì có phần tử liên kết với d ước chung lớn
a1, a2, , an
Định nghĩa Các phần tử a1, a2, , an R gọi nguyên tố ước
chung lớn chúng liên kết với đơn vị R Tiếp theo ta nêu số loại vành đặc biệt:
Định nghĩa Một miền nguyên R gọi vành nhân tử hóa phần tử khác khơng có phân tích thành phần tử bất khả qui, nghĩa với a ∈ R,
tồn ước đơn vị u phần tử bất khả qui pi (i= 1,2, , r) cho:a=u r
P
i=1
pi
Chú ý: Trong vành nhân tử hóa, ab=cn, với nguyên tố tồn ước
u1, u2 đơn vị phần tử c1, c2 cho: a=u1cn1, b=u2cn2 với u1.u2 =
Định nghĩa 10 Một vành C miền nguyên mà tất ideal chính, tức có dạng xC= (x) với x∈C
Định nghĩa 11 Miền nguyên E gọi vành Euclide tồn ánh xạg :E∗ → cho:
a) g(ab) ≥ g(a), ∀a, b ∈ E∗ b) Với hai phần tử a b 6= E, tồn hai phần tử q r E cho:a = bq+r với r = g(r) < g(b) Từ ta có kết quan trọng:
Định lý Mọi vành vành nhân tử hóa Định lý Mọi vành Euclide vành
1.2 Vành Z[i]
Tập [i] = {a+ib|a, b∈} gọi tập số nguyên Gauss a+ib Khi đó, Z[i] miền nguyên Trên Z[i] ta xây dựng hàm chuẩn N : [i] → cho bởi: N(z) = z.z với Do đó, với z =a+ib N(z) =a2+b2.Từ ta rút ra:
N(z1.z2) = N(z1).N(z2), ∀z1, z2 ∈Z[i]
Một số kết vành Z[i]
Định lý Nếu z1, z2 ∈Z[i], z2 6=
a) N(z1z2)≥N(z1)
b) Tồn q, r∈Z[i] cho z1 =qz2+r N(r)< N(z2)
Vì vành Z[i] vành Euclide
(91)Định lý Các số nguyên tố Z[i] là: a) +i;
b) Các số nguyên tố p củaN có dạng 4k+
c) Các sốa+bi với a, b∈Z a2+b2 là số nguyên tố.
1.3 Vành số nguyên trường Qh√d
i
Xét trường Qh√di với d số nguyên dương khơng phương Với z = a+b√d, ta kí hiệu z =a−b√d Khi đó, hàm chuẩn N :Qh√di→N cho bởi:
N(z) =|z.z|=a2−db2 Từ ta rút ra:
a)N(z1z2) = N(z1).N(z2), ∀z1, z2 ∈Q
h√
di
b)z1z2 =z1.z2, ∀z1, z2 ∈Q
h√
di Ta có kết quan trọng sau đây:
Định lý Nếud≡2,3 (mod 4), vành số nguyên Q h√
d
i
h√
d
i
=Q+Q√d
Nếu d ≡ (mod 4), vành số nguyên Qh√di Zh−1+ √
d
2
i
= Z+Z−1+
√
d
2
Trường hợp d <0 , ta có kết sau:
Định lý Vành số nguyên Qh√di , với d <0 , vành nhân tử hóa
d∈ {−1;−2;−3;−7;−11;−19;−43;−67;−163}
Đối với d >0 ta chưa biết hết trường hợp Ta nêu đại diện số trường hợp chẳng hạn d= 2,3,5,6,7,11,
2 Một số toán số học
2.1 Sử dụng vành Z[i]
Bài tốn Giải phương trình tìm nghiệm nguyên: y7−2x=x2+
Lời giải Viết phương trình dạng:
(92)Ta có nhận xét y lẻ ngược lại (x+ 1)2 ≡ −1 (mod 4) : mâu thuẫn Do đó, x lẻ kéo theo x+ +i, x+ 1−i nguyên tố vànhZ[i]
Vì(x+ +i) (x+ 1−i) = y7 nên tồn a, b∈Z cho:
x+ +i=u(a+ib)7
=ua a6−C72a4b2+C74a2b4−C76b6+ib C71a6−C73a4b2+C75a2b4−C77b6,
với u ước đơn vị Suy ra|a(a6−C2
7a4b2+C74a2b4−C76b6)|=
b C71a6−C73a4b2+C75a2b4−C77b6
=
⇒a = 0; b=±1 b= 0; a=±1 Vậy nghiệm nguyên phương trình là:(−1, 1)
Nhận xét:Điểm mấu chốt để giải toán chứng minh đượcx+ +i vàx+ 1−i nguyên tố Do đó, sử dụng phân tích thành phần tử bất khả qui trongZ[i] để từ rút dạng x+ +i=u(a+ib)7
Bài tốn Giải phương trình tìm nghiệm ngun: x2+ =yp, với p số nguyên tố có dạng 4k+
Lời giải Viết phương trình dạng:
(3 +xi) (3−xi) = yp
Vì(x+ 3i, x−3i) = nên +xi= (a+ib)p Khai triển nhị thức ta được:
3 = ap−Cp2ap−2b2+ + (−1)p−21Cp−1
p ab p−1.
Suy a|3và 3≡ap (mod p).
Theo định lí Fermat nhỏ ap ≡a (mod p), suy ra a = 3. Từ đó,
1 = 3p−1−Cp23p−3b2+ + (−1)p−21Cp−1
p b p−1.
Suy ra, 3p−1−1 =C2
p3p
−3b2−C4
p3p
−5b4+ −(−1)p−21Cp−1
p bp
−1. (*)
Do p= 4k+ , nên từ (*) suy rab2 chẵn Ta lại có 3p−1−1chia hết cho 23 và khơng chia hết
cho24 vơ lí Do đó, phương trình vơ nghiệm
Bài toán Cho a, b, c số nguyên thỏa:a=b2cvà c không chia hết cho số nguyên
tố p≡3 (mod 4) Chứng minh a tổng hai số phương
Lời giải Xét số nguyên tố p trongZ gọi π =a+ib số nguyên tố Z[i]sao cho π|p
Khi đó,N(π)|N(p) =p2. Suy ra, N(π) = phoặc N(π) = p2 Và đó,
a2+b2 =p a2+b2 =p2
Trường hợpp≡1 (mod 4), giả sửp= 4k+ 1,thì sử dụng định lí Wilson ta chứng minh
p|(n2+ 1) , với n= (2k)! Do đó, π|(n+i) (n−i) Suy ra:
(93)Mặt khác, p|(n±i)thì p|n p|1 : vơ lí Do đó, N(p)6=N(π) Suy ra,
a2+b2 =p
Trường p= hiển nhiên p tổng hai số phương Do c khơng chia hết cho số ngun tố p ≡ (mod 4), nên c tích số nguyên tố p ≡ 1; (mod 4) Suy ra, tồn
t, r∈Z cho: c=N(t+ir) = t2+r2. Từ đó,
a=b2c=N(b).N(t+ir) = N(tb+irb) = (tb)2+ (rb)2
2.2 Sử dụng vành số nguyên của Qh√d
i
Bài tốn Tìm tất cặp số nguyên dương (x, y) cho: 13x+ =y2
Lời giải Biến đổi phương trình dạng:
y−√3 y+√3=4−√3
x
4 +√3
x
Trong vànhZ√3 , giả sử có số nguyên tốp∈Z√
3 cho
p|y−√3, p|y+√3,
suy ra:
N(p)|Ny+√3=y2−3 = 13x Mặt khác, p|2√3 nên N(p)|N 2√3
= 12 Từ đó, N(p)|(12,13x) = 1. Suy ra:N(p) = : vơ
lí Vì vậy, y−√3, y+√3= , đóy+√3 số bậc x Ngoài ra, 4−√3,4 +√3= nên
y+√3 =
4 +√3
x
=
x
X
k=0
Cxk4x−k
√
3
k
⇒1 = X
k
Cx2k+13k4x−(2k+1) =x4x−1+X
k6=0
Cx2k+13k4x−(2k+1) ⇒x= 1⇒y=
Vậy nghiệm nguyên dương phương trình là:(1,4)
Bài tốn Giải phương trình tìm nghiệm ngun: x2+x+ =y3
Lời giải Viết phương trình dạng:
2x+ 1−√−7 ·
2x+ +√−7 =y
(94)Vì
2x+1−√−7 ,
2x+1+√−7
= vành số nguyên Q√−7
nên có a, b∈Z cho:
2x+ +√−7
2 =
a+b√−7
3 ·
Do đó, 3a2b−7b3 = ⇒ b|4 ⇒ b = −1; a = ±1 Vậy tất nghiệm nguyên phương trình là: (2,2) (−3,2)
Chú ý: Do −7≡1 (mod 4) nên vành số nguyên Q√−7 làZ h
−1+√−7
i
Bài toán (VMO, 2010) Chứng minh với số nguyên dương n, phương trình: x2 + 15y2 = 4n có n nghiệm tự nhiên(x, y).
Lời giải Ta chứng minh toán phương pháp quy nạp: Với n = phương trình có nghiệm(2,0)
Với n= phương trình có nghiệm (4,0),(1,1)
Chú ý rằng, nếu(x0, y0)là nghiệm phương trình x2+ 15y2 = 4nthì (2x0,2y0)là nghiệm
phương trìnhx2+ 15y2 = 4n+1.Do đó, ta cần chứng minh phương trình có nghiệm tự nhiên
lẻ vớin≥2 Giả sử vớin ≥2,có cặp số nguyên dương lẻ (xn, yn) cho:x2n+ 15y2n= 4n.Xét
vànhZh1+ √
−15
i
, ta có:
N xn+
√ −15yn
=N xn−
√ −15yn
=x2n+ 15y2n= 4n
Mặt khác,N1+ √ −15 = ⇒N
xn+√−15yn
· 1+ √ −15 =N
xn−√−15yn
· 1+ √ −15
= 4n+1
⇒ xn−215yn2
−15 xn+2yn2 = xn+152 yn2−15 xn−2yn2 = 4n+1
Doxn, yncùng lẻ nên số xn+2yn, xn−2yn phải lẻ Nếu xn+2yn lẻ xn−2yn chẵn xn−15yn
2 lẻ, tương tự cho trường hợp lại Vậy trường hợp,
xn
−15yn
2
,
xn+2yn
xn+15yn
2
,
xn−yn
2
là nghiệm tự nhiên lẻ phương trình:
x2+ 15y2 = 4n+1
Vậy phương trình cho có n nghiệm tự nhiên Một số toán khác
Bài toán Giải phương trình nghiệm nguyên sau: a)x
5−1 =y2
b)x2+ =y3
Bài toán Giải phương trình sau tìm nghiệm nguyên với n số nguyên lớn 1:
a)x2+ =yn
(95)Bài toán Chứng minh phương trìnhx2+y2 =pcó nghiệm khip≡1 (mod 4)
Bài toán 10 Giả sử x, y, z số tự nhiên thỏa mãn xy =z2+ 1. Chứng minh rằng
tồn số nguyên dương a, b, c, d chox=a2+b2, y =c2+d2 z =ac+bd
Bài tốn 11 Tìm tất nghiệm ngun dương phương trình x2+y2 =z2.
Bài tốn 12 (American Mathematical Monthly) Cho p= 4m−1 số nguyên tố x, y số nguyên tố cho
x2 +y2 =z2m
với số nguyên z Chứng minh p|xy
Bài toán 13 (Romanian Mathematical Olympiad) Cho S tập số nguyên dương có dạng
a2 + 2b2, với a, b số nguyên và b 6= 0 Chứng minh số nguyên tố và
p2 ∈S p∈S
Bài tốn 14 Cho p≡1 (mod 6) , chứng minh tồn a, b∈Z cho
p=a2−ab+b2
Bài toán 15 (IMO, 2001) Cho a > b > c > d số nguyên dương giả sử rằng:
ac+bd= (b+d+a−c) (b+d−a+c)
Chứng minh ab+cd không số nguyên tố
Bài toán 16 Xét dãy số (un) cho bởi: u1 = un+1 = 2u2n−1, ∀n≥ Chứng minh
nếu có số lẻ p cho p|an, p≡ ±1 (mod 2n+2)
Tài liệu tham khảo
[1] M Ram Murty, J Esmonde,in Algebraic Number Theory, Springer, 2004
[2] Serge Lang, Đại số, Phần I, Nhà xuất Đại học Trung học Chuyên nghiệp, 1978 [3] Ngô Thúc Lanh, Đại số số học, Tập 2, Nhà xuất Giáo dục, 1986
(96)NỘI SUY THEO YẾU TỐ HÌNH HỌC CỦA ĐỒ THỊ
Phạm Thị Thúy Hồng, Trường THPT Sào Nam, Quảng Nam
Bài toán xác định biểu thức hàm số biết giá trị hàm số (hoặc giá trị đạo hàm hàm số) số điểm cho trước gọi toán nội suy Các toán nội suy vấn đề liên quan đến phần quan trọng đại số giải tích tốn học Các tốn nội suy có vị trí đặc biệt quan trọng tốn học khơng đối tượng để nghiên cứu mà đóng vai trị cơng cụ đắc lực mơ hình liên tục mơ hình rời rạc giải tích lý thuyết phương trình, lý thuyết xấp xỉ,
Sử dụng hàm (đa thức) nội suyP(x), ta dễ dàng tính giá trị hàm sốf(x)tạix∈R tương đối xác Từ ta tính gần đạo hàm tích phân
R Vì đa thức đại số hàm số đơn giản nhất, nên trước tiên ta nghĩ đến việc xây dựng
P(x) dạng đa thức đại số
Tuy nhiên, trường phổ thơng tốn nội suy mẻ bỡ ngỡ giáo viên giảng dạy tốn Vì vậy, việc tìm lời giải tốn nội suy niềm say mê khơng người, đặc biệt người dạy toán học toán Các toán nội suy đa dạng đề tài, phong phú chủng loại, phù hợp cho đối tượng cấp học
1 Các toán nội suy cổ điển
1.1 Nội suy Lagrange
Định lý (Đồng thức Lagrange) Nếux1, x2, , xm m (m >1)giá trị tùy ý, đôi
khác f(x) đa thức bậc nhỏ thua m ta có đồng thức sau
f(x) =f(x1) (x−x2)(x−x3) .(x−xm) (x1−x2)(x1−x3) .(x1−xm)
+
+f(x2)
(x−x1)(x−x3) .(x−xm)
(x2−x1)(x2−x3) .(x2−xm)
+· · ·+f(xm)
(x−x1)(x−x2) .(x−xm−1) (xm−x1)(xm−x2) .(xm−xm−1)
(1)
1.2 Bài toán nội suy Lagrange
Bài toán (Bài toán nội suy Lagrange) Cho x0i, a0i ∈ R, với x0i 6= x0j ∀i 6= j, (i, j =
1,2, , N) Hãy xác định đa thức L(x) có bậc khơng qN−1 (degL(x)6N−1) thỏa mãn điều kiện
(97)1.3 Nội suy Taylor
Bài toán (Bài toán nội suy Taylor) Cho x0, ak ∈ R, với k = 0,1, , N −1 Hãy xác định
đa thứcT(x) có bậc khơng q N −1 (degP(x)6N −1)và thỏa mãn điều kiện:
T(k)(x0) = ak, ∀k = 0,1, , N −1 (3)
Định lý (Taylor) Giả sử f :U(a, δ)→R hàm khả vi liên tục đến cấp n−1 δ- lân cận U(a, δ) điểm a có đạo hàm hữu hạn cấp n điểm a Khi đó, hàm f biểu diễn dạng
f(x) =
n
X
k=0
f(k)(a)
k! (x−a)
k+o((x−a)n) (4)
khi x→a, 0! = 1, f(0)(a) = f(a).
1.4 Nội suy Newton
Bài toán (Bài toán nội suy Newton) Cho xi, ∈ R, với i = 1,2, , N Hãy xác định đa
thức N(x) có bậc khơng qN (degN(x)≤N −1)và thỏa mãn điều kiện:
N(i−1)(xi) =ai, ∀i= 1,2, , N (5)
1.5 Nội suy Hermite
Trong số trường hợp, ta cần tìm hàm đa thức qua điểm cho trước mà phải thõa mãn điều kiện đạo hàm điểm Ta gọi đa thức đa thức nội suy Hermit
Bài toán (Nội suy Hermite) Cho xi, aki ∈ R, với i = 1,2, , n; k = 0,1, , pi −1
và xi 6= xj ∀i 6= j, p1 + p2 +· · · +pn = N Hãy xác định đa thức H(x) có bậc
degH(x)≤N −1 thỏa mãn điều kiện
H(k)(xi) =aki, ∀i= 1,2, , n; ∀k= 0,1, , pi−1 (6)
2 Nội suy theo hệ thống điểm cực trị đồ thị
Định lý Giả sử f(x) khả vi đến cấp liên tục khoảng chứa x0, f00(x0) 6=
f0(x0) = Khi
Nếu f00(x0)>0 hàm số đạt cực tiểu x0
Nếu f00(x0)<0 hàm số đạt cực đại x0
Bài tốn (Mở rộng định lí) Giả sử f(x) khả vi đến cấp n x0 giả sử
f0(x0) = f00(x0) =· · ·=fn−1(x0) = 0, fn(x0)6=
(98)• Nếu n chẵn f(x) đạt cực trị x0 Hơn fn(x0)>0 f(x) đạt cực tiểu
x0, fn(x0)<0 f(x) đạt cực đại x0
• Nếu n lẻ f(x) khơng đạt cực trị x0
Nhận xét.Hàm f(x) đạt cực trị x =x0 f0(x0) = f0(x0) khơng tồn Những
điểm gọi điểm dừng hàm số
2.1 Nội suy theo hệ thống điểm dừng bậc một
Bài toán (Điểm dừng đơn) Xác định đa thứcP(x) cho đồ thị có điểm dừng đơn x1, x2, , xn(x1 < x2 <· · ·< xn)
P0(x1) =P0(x2) =· · ·=P0(xn) =
(P00(x1)= 06 , , P00(xn)),, tức
P0(x) = a(x−x1)(x−x2) .(x−xn)
Giải Ta viết
P0(x) =a(x−x1)(x−x2) .(x−xn)
=a0+a1x+· · ·+anxn
=
n
X
k=0
akxk,
trong
an=a
an−1 = (−1)a(x1+x2+· · ·+xn)
an−2 = (−1)2(x1x2+x1x3+· · ·+xn−1xn)
an−k = (−1)ka(x1x2 xk+x1x2 xk−1xk+1+· · ·+
+x1xk+2 xn−1xn+x2x3 xk+1+· · ·+xk+1xk+2 xn)
a1 = (−1)n−1a(x1x2 xn−1+x1x2 xn−2xn+· · ·+x2x3 xn)
a0 = (−1)nax1x2 xn
Suy
P(x) = a0x+a1
x2
2 +· · ·+an
xn+1
n+ +C1 =
n
X
k=0
k+ 1akx
k+1+C
(99)Bài toán (Điểm dừng bội) Xác định đa thức P(x) cho đồ thị có điểm dừng x1, x2, , xn(x1 < x2 <· · ·< xn), cho
P0(x) =a(x−x1)2α1+1(x−x2)2α2+1 .(x−xn)2αn+1
Giải Đặt(2α1+ 1) + (2α2+ 1) +· · ·+ (2αn+ 1) = 2(α1+· · ·+αn) +n=N Ta viết
P0(x) = a(x−x1)2α1+1(x−x2)2α2+1 .(x−xn)2αn+1
=a(x−t1)× · · · ×(x−t2α1+1)×(x−t2α1+2)× · · · ×(x−t2(α1+α2)+2))
×(x−t2(α1+α2)+3))× · · · ×(x−t2(α1+α2+α3)+3))× · · · ×
×(x−t2(α1+···αn−1)+n)× · · · ×(x−tN)
=a0+a1x+· · ·+aNxN = N
X
k=0
akxk,
trong
t1 =t2 =· · ·=t2α1+1 =x1
t2α1+2 =t2α1+3 =· · ·=t2(α1+α2)+2 =x2
t2(α1+···αn−1)+(n−1)+1=t2(α1+···αn−1)+(n−1)+2 =· · ·=tN =xn
Suy
P(x) =
N
X
k=0
ak
k+ 1x
k+1+C, C : hằng số,
với
a0 = (−1)Nat1 tα1 tN
a1 = (−1)N−1a(t1t2 tN−1+t1t2 tN−2tN +· · ·+t2t3 tN)
ak= (−1)N−ka(t1t2 tN−k+t1t2 tN−k−1tN−k+1+· · ·+t2t3 tN)
aN−1 = (−1)a(t1+t2+· · ·+tα1 +· · ·+tN−1+tN) =
= (−1)a(α1x1+α2x2+· · ·+αnxn)
aN =
2.2 Áp dụng
Bài toán Xác định đa thức P(x) bậc nhỏ với hệ số bậc cao cho đồ thị hàm số y=P(x) nhận x= 1, x= điểm cực trị
(100)Bài toán 10 Xác định đa thức P(x) bậc nhỏ với hệ số bậc cao cho đồ thị hàm sốy =P(x)nhận x= 1, x= 2, x= 3, x= điểm cực trị qua điểm A(0,1) Bài toán 11 Xác định đa thức P(x) bậc nhỏ cho đồ thị hàm số y = P(x) có điểm cực đại, cực tiểu A(0,1) B(1,0)
Bài toán 12 Xác định đa thức bậc ba P(x) thỏa mãn điều kiện nhận điểm M(1; 1) làm tâm đối xứng A(0,1) điểm cực tiểu
Bài toán 13 Xác định đa thức P(x) bậc nhỏ cho đồ thị hàm số y =P(x) có hệ số cao điểm cực tiểu A(0,1) B(1,1)
Bài toán 14 Xác định đa thức P(x) bậc nhỏ cho đồ thị hàm số y = P(x) đạt cực đại, cực tiểu A(−2,1) B(1,3)
Bài toán 15 Xác định đa thức P(x) bậc nhỏ cho đồ thị hàm số y =P(x) có hệ số cao điểm cực tiểu A(0,1) B(1,1)
Bài toán 16 Xác định đa thức P(x) bậc nhỏ cho đồ thị hàm số y = P(x) có điểm cực đại A(−1,0)và B(2,0)
Bài toán 17 Xác định đa thức P(x) bậc nhỏ cho đồ thị hàm số y =P(x) có hệ số cao 1, qua điểm A(2,0)và có điểm cực đại B(0,0) C(1,0) Bài toán 18 Xác định đa thức P(x) bậc nhỏ cho đồ thị hàm số y = P(x), qua điểm A(2,0) có điểm cực đại B(−1,0) C(0,0)và D(1,0)
3 Nội suy theo hệ thống điểm uốn đồ thị
Định nghĩa Nếu hàm sốf(x)có đạo hàm cấp hai khoảng chứa điểmx0, f00(x0) =
vàf00(x)đổi dấu khixqua điểmx0 thìM(x0;f(x0))là điểm uốn đồ thị hàm sốy =f(x)
Bài toán 19 Thay điểm dừng bậc điểm dừng bậc 2(P00(x1) = 0, P00(x2) = 0, , P00(xn)) =
0
TH1 Điểm dừng đơn
P00(x) =a(x−x1)(x−x2) .(x−xn)
Ta viết
P00(x) = a(x−x1)(x−x2) .(x−xn)
=a0+a1x+· · ·+anxn
=
n
X
k=0
(101)trong
a0 = (−1)nax1x2 xn
a1 = (−1)n−1a(x1x2 xn−1+x1x2 xn−2xn+· · ·+x2x3 xn)
ak = (−1)n−ka(x1x2 xn−k+x1x2 xn−k−1xn+· · ·+x2x3 xn)
an−1 = (−a)(x1+x2+· · ·+xn)
an=a
Suy
P0(x) = a0x+a1
x2
2 +· · ·+an
xn+1
n+ +C1 =
n
X
k=0
k+ 1akx
k+1+C 1,
hay
P(x) =
n
X
k=0
ak
1
k+ 1
k+ 2x
k+2
+C1x+C2
TH2 Điểm dừng bội
P00(x) =a(x−x1)2α1(x−x2)2α2 .(x−xn)2αn
Giải Đặt2α1 + 2α2+· · ·+ 2αn = 2(α1+· · ·+αn) = N
Ta viết
P0(x) =a(x−x1)2α1(x−x
2)2α2 .(x−xn)2αn
=a(x−t1)× · · · ×(x−t2α1)×(x−t2α1+1)× · · · ×(x−t2(α1+α2))
×(x−t2(α1+α2)+1))× · · · ×(x−t2(α1+α2+α3))× · · · ×
×(x−t2(α1+···αn−1)+1)× · · · ×(x−tN)
=a0+a1x+· · ·+aNxN = N
X
k=0
akxk,
trong
t1 =t2 =· · ·=t2α1 =x1
t2α1+1 =t2α1+2 =· · ·=t2(α1+α2) =x2
t2(α1+···αn−1)+1 =t2(α1+···αn−1)+2 =· · ·=tN =xn
Suy
P0(x) =
N
X
k=0
ak
k+ 1x
(102)hay
P(x) =
N
X
k=0
ak
1
k+ 1
k+ 2x
k+2+C
1x+C2,
với
a0 = (−1)Nat1 tα1 tN
a1 = (−1)N−1a(t1t2 tN−1+t1t2 tN−2tN +· · ·+t2t3 tN)
ak = (−1)N−ka(t1t2 tN−k+t1t2 tN−k−1tN−k+1+· · ·+t2t3 tN)
aN−1 = (−1)a(t1+t2+· · ·+tα1 +· · ·+tN−1+tN) =
= (−1)a(α1x1+α2x2+· · ·+αnxn)
aN =
Bài toán 20 Xác định đa thức P(x) bậc nhỏ cho đồ thị hàm số y =P(x) có hệ số cao 1, qua A(0,−1)và điểm uốn A(−1,1)
Bài toán 21 Xác định đa thức P(x) bậc nhỏ cho đồ thị hàm số y = P(x) qua điểm A(−1,0) điểm uốn A(1,0)
Bài toán 22 Xác định đa thức P(x) bậc nhỏ cho đồ thị hàm số y = P(x) qua điểm A(−1,0) có điểm uốn B(0,0) C(0,1)
Bài toán 23 Xác định đa thức P(x) bậc nhỏ cho đồ thị hàm số y = P(x) có điểm uốn A(−2,0), B(0,0), C(3,0) qua D(100)
4 Nội suy theo hệ thống cực trị điểm uốn đồ
thị
Bài toán 24 Xác định đa thức P(x) bậc nhỏ cho đồ thị hàm số y = P(x) có điểm uốn điểm cực tiểu A(0,0) B(1,0)
Giải Vì đồ thị hàm sốy=P(x)có điểm uốn điểm cực tiểu x= 0,x= từ giả thiếtP(0) = P(1) = 0nên theo định lí Rolle, ta tìm khoảng (0,1)điểm x0
choP0(x0) = Do phương trìnhP0(x) = phải có nghiệm Suy radegP0(x)≥4
NếudegP0(x) = 4, phương trình P(x) = có5 nghiệm Khi đó, theo giả thiết (
P(0) =P00(0) =
P(1) =P0(1) =
Khi theo tốn nội suy Hermite
(103)Để đồ thị hàm số có điểm uốn cực tiểu tạiA, B a >0 Vậy đa thức bậc năm có dạng
P(x) =ax3(x−1)2, a >0
Thử lại, ta thấy điều kiện đảm bảo để đồ thị hàm số y = P(x) có điểm uốn điểm cực tiểu tạiA(0,0)và B(1,0)là thỏa mãn Vậy đa thứcP(x) bậc nhỏ cho đồ thị hàm sốy=P(x)có điểm uốn điểm cực tiểu A(0,0) B(1,0) đa thức
P(x) =ax3(x−1)2, a >0
Bài toán 25 Xác định đa thức P(x) bậc nhỏ cho đồ thị hàm số y = P(x) có điểm uốn điểm cực đại A(0,0) B(1,0)
Giải Vì đồ thị hàm sốy=P(x)có điểm uốn điểm cực đại tạix= 0,x= từ giả thiếtP(0) =P(1) = 0nên theo định lí Rolle, ta tìm khoảng(0,1)điểmx0
choP0(x0) = Do phương trìnhP0(x) = phải có nghiệm Suy radegP0(x)≥4
NếudegP0(x) = 4, phương trình P(x) = có5 nghiệm Khi đó, theo giả thiết (
P(0) =P00(0) =
P(1) =P0(1) =
Khi theo toán nội suy Hermite
P(x) =ax3(x−1)2, a6=
Để đồ thị hàm số có điểm uốn cực tiểu tạiA, B a <0 Vậy đa thức bậc năm có dạng
P(x) =ax3(x−1)2, a <0
Thử lại, ta thấy điều kiện đảm bảo để đồ thị hàm số y = P(x) có điểm uốn điểm cực tiểu tạiA(0,0)và B(1,0) thỏa mãn Vậy đa thứcP(x) bậc nhỏ cho đồ thị hàm sốy=P(x)ó điểm uốn điểm cực tiểu A(0,0)và B(1,0)là đa thức
P(x) =ax3(x−1)2, a <0
Bài toán 26 Xác định đa thức P(x) bậc nhỏ cho đồ thị hàm số y = P(x) có điểm uốn A(−2,0), B(1,0) đạt cực đại C(0,0)
(104)5 Một số toán liên quan
Bài toán 28 Cho hàm số f(x)liên tục [a, b], khả vi (a, b) vàf(a) =f(b) = Chứng minh với k∈R∗, phương trình
f(x) +kf0(x) = (7) ln có nghiệm x∈(a, b)
Chứng minh Xét hàm số
g(x) =exkf(x)
Ta có g(x)liên tục [a, b], khả vi (a, b) g(a) =g(b) = Theo định lí Rolle, phương trìnhg0(x) = ln có nghiệmx∈(a, b) Mà
g0(x) =
ke
x
kf(x) +e x
kf0(x) =
ke
x
k[f(x) +kf0(x)]
Vì
ke
x
k 6= 0,∀x∈R nên
g0(x) = 0⇔f(x) +kf0(x) =
Vậy phương trình(3.1) ln có nghiệmx∈(a, b) Đây điều cần chứng minh
Bài toán 29 Chứng minh lim
x→∞f(x) = f
0(x) có số lượng khơng điểm trong khoảng (a,∞) khơng so với f(x) khoảng Kết thay +∞ −∞
Bài toán 30 Giả sử hàm số f(x) có n khơng điểm khoảng (a,+∞) Chứng minh với số thực α hàm số
αf(x) +f0(x)
có nhấtn−1 khơng điểm khoảng Hơn nữa, thỏa mãn điều kiện
lim
x→+∞e
αxf(x) = 0
thì hàm nêu có n khơng điểm
Bài tốn 31 Nếu đa thức f(x)∈ R[x] có k nghiệm thực đa thức f0(x) có (k−1)
nghiệm thực
Bài toán 32 Cho 0≤x1 < x2 < x3 < x4 Chứng minh bất đẳng thức sau:
3 r
1
4(x1x2x3+x1x2x4+x1x3x4+x2x3x4)
<
r
1
(105)Bài toán 33 Cho hàm số f(x) có đạo hàm liên tục đoạn [a, b], thỏa mãn điều kiện
f(a) = f(b) = 0, f(x)6= 0,∀x∈(a, b)
Chứng minh tồn dãy {xn} với xn∈(a, b) cho
lim
n→+∞
f0(xn)
(√ne−1)(f(x n)
= 2011 (8)
Chứng minh Xét hàm số
gn(x) = e−
2011x
n f(x), x∈[a, b], n ∈N
Rõ ràng hàm số gn(x) xác định liên tục đoạn [a, b], khả vi khoảng (a, b) Hơn
nữa, ta cógn(a) = gn(b) nên theo định lí Rolle, tồn xn∈(a, b)sao cho g0n(xn) =
Với mỗin, tồn mộtxn Điều có nghĩa ta xây dựng dãy{xn} Ta
chứng minh dãy thỏa mãn đẳng thức (3.3) Thật vậy, ta có
gn0(xn) =−
2011
n e
−2011nxn
f(xn) +e−
2011xn n f0(x
n) =
Vì
f0(xn)
f(xn)
= 2011
n
Suy
lim
n→+∞
f0(xn)
(√ne−1)f(x n)
= lim
n→+∞
2011
en1 −1
= 2011 (đpcm)
Bài toán 34 Giả sử
f(x) = (x−x1)(x−x2)(x−x3),
với x1 < x2 < x3 Chứng minh
f00(x1)
f0(x
1) + f
00(x
2)
f0(x
2) +f
00(x
3)
f0(x
3) =
Bài toán 35 Cho c0, c1, , cn số thực thỏa mãn điều kiện
c0 +
c1 +
c2
3 +· · ·+
cn
n+ =c0+c1 +
c2.22 +
c3.23
4 +· · ·+
cn.2n
n+ =
Chứng minh phương trình
c1+ 2c2x+· · ·+ncnxn−1 =
(106)Bài tốn 36 (Olympic Nga) Cho phương trình
a0xn+a1xn1+· · ·+an−1x+an = 0, a0 6=
có n nghiệm phân biệt Chứng minh
(n−1)a21 >2na0a2
Bài toán 37 (Olympic 30.4 - 2003) Tồn hay khơng số thực a, b, c để phương trình sau có bốn nghiệm thực phân biệt
x+5 =a.e
3x+b.e2x+c.ex−
4.e
−x.
Bài toán 38 Giả sử hàm f(x) khả vi liên tục n lần [a, b], đoạn có khơng n khơng điểm tính bội Chứng minh rằng:
max
[a,b] |f(x)| ≤
(b−a)n
n! max[a,b]
f
(n)(x)
Bài tốn 39 (Olympic sinh viên tồn quốc- 1993 vịng 1) Cho hàm số f(x) xác định có đạo hàm bậc hai liên tục không đồng đoạn R Biết đồ thị hàm số y =f(x) cắt đường thẳng ax+by+c = ba điểm phân biệt Chứng minh tồn x0 ∈R cho f00(x0) = f00(x) đổi dấu quax=x0
Bài toán 40 (Olympic sinh viên toàn quốc- 1994) Cho n số nguyên dương, ak, bk ∈R(k =
0,1, , n) Chứng minh phương trình
x+
n
X
k=1
(aksinkx+bkcoskx) =
có nghiệm khoảng (−π, π)
Bài toán 41 (Olympic sinh viên toàn quốc - 1999) Giả sử đa thức với hệ số thực
P(x) =a0+a1x+· · ·+anxn
có n nghiệm thực phân biệt chứng minh
ak−1ak+1 <(ak)2, k= 1,2, , n−1
Bài tốn 42(Olympic sinh viên tồn quốc - 2001) Chứng minh tồn số thựcx∈(0,1)
sao cho
1
Z
x
t2000dt
(1 +t)(1 +t2)· · ·(1 +t2001) =
x2000
(107)Bài tốn 43 (Olympic sinh viên tồn quốc - 2003) Cho đa thức với hệ số thực P(x) bậc
n(n≥1) có m nghiệm thực Chứng minh đa thức
f(x) = (x2+ 1)P(x) +P0(x)
có nhấtm nghiệm thực
Bài toán 44 (Olympic - 2006) Tìm tất dãy số thực a1, a2, , an, với n ≥ 1, an 6= 0,
thỏa mãn tính chất sau:
Nếu f :R→R hàm số khả vi cấp n x0 < x1 <· · ·< xn số thực thỏa mãn
f(x0) =f(x1) =· · ·=f(xn) =
thì tồn tạiξ ∈(x0, xn) cho
a0f(ξ) +a1f0(ξ) +· · ·+anf(n)(ξ) =
Bài toán 45 Kí hiệu T tấp hợp tất đa thức bậc 2011 có 11 nghiệm thực kể nghiệm bội Với P(x)∈T, đặt
QP(x) = (x20+ 1)P(x)−P0(x)
và gọi SP số nghiệm thực đa thức QP(x) Tìm P∈TSP
Tài liệu tham khảo
[1] Nguyễn Văn Mậu, 2005, Đa thức đại số phân thức hữu tỷ,NXB Giáo Dục [2] Nguyễn Văn Mậu, 2006, Đa thức áp dụng, NXB Giáo Dục
[3] Nguyễn Văn Mậu, 2007, Nội suy áp dụng, NXB Giáo Dục
[4] Nguyễn Văn Mậu,2006, Tuyển tập Olympic sinh viên toàn quốc, NXB Giáo Dục [5] Một số báo liên quan
(108)BẤT BIẾN NHƯ LÀ MỘT PHƯƠNG PHÁP CHỨNG MINH VÀ ỨNG DỤNG TRONG GIẢI TOÁN
Lê Sáng – Vũ Đức Thạch Sơn, Trường THPT Chuyên Lê Quý Đôn – Khánh Hòa
Ta thường làm quen với phương pháp chứng minh toán học như: trực tiếp, quy nạp, phản chứng, nguyên tắc Dirichlet ,tập thứ tự nguyên tắc cực hạn Bất biến không đổi đại lượng qua nhiều phép biến đổi qua hữu hạn bước thực Chính yếu tố giúp ta phản bác tình khơng thể xảy đại lượng tốn Yếu tố bất biến thường gặp thi toán học thường tốn địi hỏi nhạy bén lập luận chặt chẽ, số toán Ở bất biến nửa bất biến xét xem phương pháp chứng minh quan trọng, dạng tốn thi chọn học sinh giỏi đề cập tương đối phong phú giúp có nhìn rõ tốn tổ hợp, giảng cho đội tuyển Khánh Hịa
1 Phương pháp đồng dư:
Mục đích tính chất bất biến dạng tốn nghiên cứu tính chẵn lẻ đồng dư với 2, 3, trạng thái toán thay đổi theo yêu cầu
Bài toán Trên hịn đảo có ba giống thằn lằn: 133 màu xám, 155 màu đỏ 177 màu xanh Nếu hai thằn lằn khác màu gặp chúng đồng thời chuyển sang màu thứ ba (ví dụ màu xanh gặp màu xám hai chuyển sang màu đỏ) hai màu gặp khơng chuyển màu Liệu có xảy tình tất thằn lằn đảo màu không?
Lời giải Như bất biến số dư chia cho số thằn lằn màu 0-1-2 Tổng số thằn lằn 133 + 155 + 177 = 465 chia hết cho xảy tình tất thằn lằn màu số dư chia cho ba loại thằn lằn 0-0-0 trái với tính bất biến 0-1-2 Như khơng thể xảy tình
Phương pháp đồng dư phát đại lượng bất biến gắn với số dư chia cho số mà chủ yếu modul modul Bài tốn “thằn lằn”trên modulo 3, cịn sau số toán sử dụng phương pháp đồng dư:
Bài tốn Trên bảng có 2011 dấu cộng 2012 dấu trừ Một học sinh thực trò chơi sau : thay hai dấu bảng dấu cộng hai số bị xóa dấu thay dấu trừ chúng trái dấu Hỏi sau 4022 lần dấu lại dấu cộng hay dấu trừ ?
(109)(-) (+)→ (-)
Ta phát bất biến khơng đổi đấu tích dấu bảng Mà lúc đầu bảng có 2011 (+) 2012 (-) tích dấu (+) Do dấu cịn lại bảng phải dấu (+)
Bài toán Trên bảng cho ba số 2,2,2 Ta xóa ba số thay vào tổng hai số cịn lại trừ Hỏi sau số lần thực ta thu số 11,1,2011 hay không?
Lời giải Hãy thử kiểm tra: 2,2,2 → 2,3,2
Lúc ba số có số lẻ số chẵn Nếu ta tiếp tục thực thì: Chẵn + Lẻ - → Chẵn
Chẵn + Chẵn - 1→ Lẻ
Như trường hợp ba số thu ln có số lẻ số chẵn Thế 11,1,2011 lại số lẻ khơng thể xảy trường hợp
Bài tốn Trong giải đấu bóng đá, đội đấu vòng tròn lượt với theo quy định trận thắng điểm, hòa điểm,thua điểm Hỏi có hai đội 10 điểm thời điểm giải đấu không?
Lời giải Ta thấy sau trận đấu có đội thắng số điểm hai đội đạt Khi hiệu hai số điểm hai đội đạt cịn hịa hai đội điểm đội hiệu số Vậy rõ ràng hiệu hai điểm số hai đội phải số chẵn Thế 10 – = số lẻ khơng tồn hai đội lúc đươc 10 điểm
Bài toán Viết 2012 số từ đến 2012, lần xóa số thay trị tuyệt đối hiệu số Hỏi số cuối số chẵn hay lẻ?
Lời giải Gọi S(n) tổng số lại sau lần thứ n S(0)là tổng 2012 số số chẳn Mà S(n) bất biến modulo số cuối phải số chẵn
Bài toán Trên bảng ta viết ba số nguyên Sau ta xóa số viết vào tổng hai số cịn lại trừ Thao tác lặp lại số lần cuối ta nhận số 29, 1876, 2011 Hỏi ba số 2,2,2 không?
Lời giải Sau bước từ ba số 2,2,2, ta nhận 2,2,3, ba số có hai chẵn lẻ Từ bước thứ hai trở kết ln ln có hai số chẵn số lẻ dù ta thực số (vì số chẵn tổng số chẵn số lẻ trừ 1; số lẻ tổng hai số chẵn trừ 1) Nhưng kết cho có hai số lẻ, số chẵn nên với thao tác cho xuất phát từ 2,2,2 cho kết
Bài toán giải nhờ phát tính chẵn lẻ ba số khơng thay đổi, nên từ trạng thái xuất phát nhận trạng thái kết
(110)Lời giải Nhận thấy đường tròn sau trình ln cịn lại chữ số Do số chữ số số chữ số khác tính chẵn lẻ Giả sử sau số lần lặp lại q trình Đường trịn cịn lại tồn chữ số suy trước đường trịn khơng có chữ số giống đứng cạnh nên số chữ số số chữ số ( vơ lí ! ), suy đpcm
Bài tốn ( Corolado MO 1997) Mỗi bảng 1997 X 1997 điền số (+1) (-1) Mỗi hàng ta tính tích Ri số hàng Mỗi cột ta tính tích Ci số cột
Chứng minh
1997
P
i=1
(Ri+Ci) khác không
Lời giải Bất biến sử dụng số dư
1997
P
i=1
(Ri+Ci)cho 4, nói cách khác ta sử
dụng bất biến theo modulo Đại lượng không đổi với cự thay đổi dấu dấu viết bảng Thật vậy, giả sử có thay đổi phần tử hàng thứ i cột thứ j cho ta −(Ri+Ci) thay (Ri+Ci) Vì (Ri+Ci) có giá trị 2, -2 theo
modulo nên tổng ban đầu thay đổi bội số Do bất biến khơng phụ thuộc vào cách chọn số (+1) (-1) Cho nên ta cần xét trường hợp tất ô điền (+1) Khi
1997
P
i=1
(Ri+Ci)≡2(mod 4) Vậy với cách điền số (+1) (-1)
1997
P
i=1
(Ri+Ci)
ln khác khơng
Bài tốn ( Hungary MO 1989) Mỗi đỉnh hình vng đặt hịn sỏi Thực thay đổi số sỏi theo quy luật sau: ta lấy số sỏi đỉnh thêm vào hai đỉnh kề bên số sỏi gấp đôi Hỏi nhận 1989, 1988, 1990, 1989 viên sỏi đỉnh liên tiếp hình vng hay không?
Lời giải Gọi đỉnh liên tiếp hình vng A, B, C, D ứng với số sỏi a, b, c, d Khi bước gọi x số sỏi lấy đi, giả sử đỉnh A, số sỏi đỉnh là:a−x, b+ 2x, c, d
hoặca−x, b, c, d+ 2x Ta có(b+d+ 2x)−(a+c−x) =b+d−a−c+ 3x Mà ban đầu số sỏi đỉnh (1 , , , ) nên từ ta có bất biến toán này: hiệu tổng số sỏi hai đỉnh A, C hai đỉnh B, D bội Mà với ( 1989, 1988, 1990, 1989) hiệu có số dư chia cho khơng xảy
Bài tốn 10 (VMO 1992) Cho bảng hình chữ nhật 1991 1992 với 1991 hàng 1992 cột Kí hiệu vuông nằm giao hàng thứ m (kể từ xuống) cột thứ n(kể từ trái sang phải) (m ; n) Tô màu ô vuông bảng theo cách sau: lần thứ tô ô (r ; s), (r + ; s + 1), (r + 2; s + 2) với r; s hai số tự nhiên cho trước thỏa mãn1≤r≤1989 1≤s≤1991; từ lần thứ hai lần tô ô chưa có màu nằm cạnh hàng cột Hỏi cách tơ màu tất vng bảng cho hay không?
(111)hết cho 3.Như ba số ghi vào ba ô (r ; s), (r + ; s + 1), (r + ; s + 2) s, s + s + mà tổng chúng số chia cho dư Vậy tô màu hết ô vuông bảng cho tổng S tất số ghi vào bảng phải số chia cho dư Nhưng S = 1991.(1 + + +1992) = 1991.1993.993 chia hết cho 3, mâu thuẫn! Do khơng thể tơ màu tất ô vuông bảng cho
2 Bảng biểu :
Bài toán 11 Trên bàn cờ 8×8có 32 qn trắng 32 qn đen, quân chiếm ô vuông Tại bước người chơi thay tất quân trắng thành quân đen tất quân đen thành quân trắng hàng cột Hỏi sau hữu hạn bước cịn lại xác quân trắng bàn cờ không?
Lời giải Nếu trước chuyển có xác k qn trắng hàng(cột) định chuyển số quân đen hàng(cột)ấy – k Sau chuyển, – k quân đen trở thành – k quân trắng k quân trắng lại trở thành k quân đen Như số quân trắng bàn cờ sau chuyển thêm vào – k k quân, tức số quân trắng thay đổi bàn cờ (8 – k) – k = – 2k Số số chẵn ,mà số quân trắng bàn cờ lúc đầu 32 qn số qn trắng bàn cờ ln số chẵn Vậy khơng thể cịn lại bàn cờ quân trắng
Bài toán 12 (IMO 2004) Ta định nghĩa viên gạch hình móc câu hình gồm vng đơn vị hình vẽ đây, hình nhận lật hình (sang trái, sang phải, lên trên, xuống dưới) hình nhận xoay hình góc:
Hãy xác định tất hình chữ nhật m×n m, n số nguyên dương cho lát hình chữ nhật viên gạch hình móc câu?
(112)Do đó, để lát hình chữ nhật m x n m.n phải chia hết cho 12 Nếu hai số m, n chia hết cho lát Thật vậy, giả sử m chia hết cho Ta viết n dạng: n= 3a+ 4b, lát
Xét trường hợp m×n khơng chia hết cho Ta chứng minh trường hợp lát Giả sử ngược lại, m , n chia hết cho không chia hết cho Ta tạo bất biến sau: Xét ô (p ; q) Nếu hai tọa độ p , q chia hết cho điền số vào Nếu hai tọa độ p, q chia hết cho điền số Các lại điền số Với cách điền số ta thu bất biến tổng số Hình tổng số Hình số lẻ Do m, n chẵn nên tổng số tồn hình chữ nhật m× n số chẵn Để lát tổng số Hình Hình sử dụng phải số chẵn Khi đó, m × n chia hết cho 24, vơ lý !
Bài tốn 13 (VMO 2006).Xét bảng vngm×n (m, n ≥ 3) Thực trị chơi sau: lần đặt viên bi vào ô bảng, ô viên bi, cho tạo thành hình đây:
Hỏi sau số lần ta nhận bảng mà số bi ô không nếu:
a) m = 2004, n = 2006 ? b) m = 2005, n = 2006 ?
Bài toán 14 (VMO 1993)Cho đa giác lồi A1A2 A1993 mà đỉnh ghi dâu cộng
(+) dấu trừ (-) cho 1993 dấu có dấu (+) (-)
Thực việc thay dấu sau: lần, thay dấu đồng thời tất đỉnh Ai(i =
1,2, ,1993) đa giác theo quy tắc:
(113)• Nếu dấu Ai Ai+1 khác dấu Ai thay dấu (-) Quy ước coi
A1994 A1
Chứng minh tồn số nguyên k≥2 cho sau thực liên tục k lần phép thay dấu nói trên, ta đa giác A1A2 A1993 mà dấu đỉnh Ai(i = 1,2, ,1993) trùng với
dấu đỉnh sau lần thay dấu thứ
Bài toán 15 (Liên bang Nga 1998) Trên bảng cho số nguyên, người ta ghi nhớ chữ số cuối số này, sau xóa cộng thêm vào với số lại bảng lần chữ số xóa Giả sử ban đầu ghi số hỏi sau số lần thực thu số hay khơng?
3 Bất biến tốn Đại số -Giải tích:
Các đại lượng không đổi số hạng dãy số có tính chất cần phát Dựa vào đại lượng mà ta tìm cơng thức tổng qt dãy số, chứng minh tính chất tìm giới hạn , xét tính hội tụ dãy số Đây tạm gọi tính bất biến Sau số toán :
Bài toán 16 ( German MO 1996 ) Từ điểm ( , 1) di chuyển sỏi mặt phẳng tọa độ thỏa điều kiện sau : (a) Từ điểm ( a , b ) đến ( 2a , b ) ( a , 2b ) (b) Từ điểm ( a , b ) đến ( a – b , b ) a > b ( a , b – a ) a < b Với số nguyên dương x y hịn sỏi đến điểm ( x , y)?
Lời giải Ta chứng minh điều kiện cần đủ toán (x , y) = 25 với s số ngun khơng âm, ký hiệu ( x , y ) ước chung lớn hai số tự nhiên x y Thật vậy: Điều kiện cần : ( p , q ) = ( p , q – p) ta thấy số ước chung lẻ bất biến sau hai phép biến đổi Ban đầu số lượng 1, số dư nên ( x , y ) lũy thừa Điều kiện đủ : giả sử (x , y) = 25 Trong tất cặp ( p , q ) đến
được (x , y) ta chọn cặp cho p + q nhỏ Nếu p q số chẵn điểm p2, q
hoặc p,2q
cũng thỏa mãn , mâu thuẫn với giả thiết tính nhỏ cặp p q Nếu p > q nhận từ điểm p+2q, q mâu thuẫn với giả thiết tính nhỏ tổng p + q Tương tự với trường hợp p < q ,mâu thuẫn Do p = q mà (p , q) lũy thừa Từ ta suy p = q = 1, nên (x , y ) điểm thỏa mãn Điều kiện chứng minh xong
Bài toán 17 Cho dãy số thỏa:
a1 = 3, b1 =
an+1 =a2n+ 2b2n, bn+1 =anbn
, ∀n∈N Chứng minh an, bn số nguyên tố
Lời giải Ta chứng minh quy nạp a2n − 2b2n = Thật vậy: Với n = 1, ta có:
a2
1−2b21 = 32−2.22 =
Với n = k, giả sử: a2
k−2b2k =
Với n = k+1, ta có:a2k+1−2b2k+1 = (a2k+ 2bk2)2−2(2akbk)2 = (a2k−2b
2
k)
2 = 1
Vậy a2
n−2b2n= 1, ∀n∈.Gọi d UCLN an, bn ta suy d ước số
đód=
(114)Bài toán 18 Cho dãy số {un}:
u1 =
u2 =
un = 4un−1−un−2 >0, n≥3, Sn= n
P
i=1
arccot(u2i)
Tìm lim
n→∞Sn
Lời giải Trước hết ta chứng minh u2n−un−1un+1 = 4, ∀n ≥2 Thật ta có:
un(4un−1) = un−1(4un)⇒un(un+un−2) =un−1(un+1+un−1)
⇒u2n−un+1un−1 =u2n−1−unun−2 ⇒u2n−un+1un−1 = =u22−u1u3 =
⇒arccotu2n= arccotun 4un4
= arccotun(un+1+un−1)
u2
n−un+1un−1
= arccot
un+1
un un un−2+1
un un−1−
un+1
un
= arccot un+1
un −arccot un
un−2 ⇒Sn= arccot
un+1
un
Hơn nữaun = 4un−1−un−2 ⇒1 = 4unun−1 −unun−−21.unun−1
Lại có (
0< un−1
un <1 un
un+1 >
un−1
un
⇒ ∃ lim
n→∞
un un+1
Chuyển qua giới hạn ta được: Do lim
n→∞
un+1
un = +
√
3⇒ lim
n→∞Sn = arccot(2 + √
3) = 12π
Vậy lim
n→∞Sn=
π
12
Bài tốn 19 Chứng minh phương trình sau có vơ hạn nghiệm ngun dương xxy2+−y12 =
Lời giải Đây toán Số học nhiên ta sử dụng tính Bất biến dãy số để chứng minh phương vô số nghiệm nguyên dương Ta xét dãy số (un) xác định bởi:
u0 = 1, u1 = 1, un+1 = 5un−un−1, ∀n≥1
Ta có
un+1+un−1
un =
un+2+un
un+1 = 5⇔(un+1+un−1)un+1= (un+2+un)un
⇔un+2un−u2n+1 =un+1un−1−un2 ⇒un+1un−1−u2n=u3u1−u22 =
⇔(5un−un−1)un−1 −u2n = 5⇔u2n+u2n−1−5unun−1+ =
⇔ u2n+u2n−1
unun−1−1 =
Vậy (x, y) = ( un−1, un) nghiệm phương trình cho với mọi∀n≥1 Cũng với cách
khai thác tính bất biến dãy số truy hồi bậc có cách giải khác độc đáo cho toán số học quen thuộc sau :
Bài toán 20 Chứng minh phương trình sau có vơ hạn nghiệm nguyên dương :x2+y2+z2 = 3xyz
Lời giải Chọn z = ta được:x2 +y2+ = 3xy.
Xét dãy số (un)được xác định sau :
(115)Ta có un+1un−1 −u2n =u3u1−u22 =−1
Suy : (3un−un−1)un−1−u
2
n =−1
⇔u2n+u2n−1+ = 3unun−1
Vậy (x, y, z) = (un−1, un,1)là nghiệm phương trình cho với ∀n ≥1
Bài tốn 21 Chứng minh phương trình sau có vơ hạn nghiệm ngun dương :x2+y2+z2 =
xyz
Lời giải Đặt x= 3x1, y = 3y1, z= 3z1 ta có x12+y12+z12 = 3x1y1z1
Xét dãy số (un)được xác định sau :
u0 = 1, u1 = 1, un+1 = 3un−un−1, ∀n≥1
Theo tốn (x1, y1, z1) = (un−1, un,1) nghiệm phương trình
x12+y12+z12 = 3x1y1z1
Do đó(x, y, z) = (3un−1,3un,3) nghiệm phương trình cho với ∀n≥1
Bài tốn 22 Chứng minh phương trình sau có vơ hạn nghiệm nguyên dương: x2+y2 +
z2+t2 = 4xyzt.
Lời giải Chọn z = t = ta có x2+y2+ = 4xy
Xét dãy số (un)được xác định sau:
u0 = 1, u1 = 1, un+1 = 4un−un−1, ∀n≥1
Ta có un+1un−1−u2n =u3u1−u22 =−2
Suy
(4un−un−1)un−1−u2n =−2
⇔u2
n+u2n−1+ = 4unun−1
Vậy (x, y, z, t) = (un−1, un,1,1)là nghiệm phương trình cho với ∀n ≥1
Bài tốn 23 Chứng minh phương trình sau có vơ hạn nghiệm ngun dương : x2+y2+
z2+t2 =xyzt
Bài toán 24 Chứng minh phương trình sau có vơ hạn nghiệm ngun dương : x2+y2 = 4(x+ 1)(y−1)
Bài toán 25 Chứng minh phương trình sau có vơ hạn nghiệm ngun dương : x+2y +3yx+4 = 10
Bài toán 26 ( Bulgari MO) Chứng minh với n ≥3 phương trình sau có nghiệm ngun dương (x, y) thỏa mãn x, y lẻ 7x2+y2 = 2n
(116)Bài toán 28 Chứng minh với số ngun dương n phương trình sau có nghiệm nguyên dương thỏa mãn (x, y, z) = 1:
x2+y2 +z2 = 72n
Bài toán 29 Trên bảng cho số 3,4,5,6 Mỗi lần xóa số x, y số thay vào hai số x+y+px2 +y2 và x+y−px2+y2. Hỏi sau số lần thực bảng
có thể xuất số nhỏ khơng?
4 Các tốn khác :
Bài toán 30 ( Rusian MO 1995 ) Cho ba đống sỏi khác Sisyphus thực di chuyển viên sỏi từ ba đống sỏi sang đống sỏi lại Mỗi lần chuyển sỏi, Sisyphus nhận từ Zeus số tiền hiệu số số sỏi đống sỏi lấy đống sỏi nhận thêm trước di chuyển Nếu số chênh lệch âm Sisyphus phải trả cho Zeus số tiền chênh lệch Sau số bước thực số sỏi đống trở ban đầu Hỏi số tiền tối đa mà Sisyphus nhận bao nhiêu?
Lời giải Ta chứng minh tổng sau bất biến a(a2−1) +b(b2−1) + c(c2−1) +s
Với a, b, c số sỏi đống ban đầu s số tiền Sisyphus nhận thời điểm Thật ta chuyển viên sỏi từ đống sỏi có a viên sỏi sang đống sỏi có b viên sỏi Khi số tiền Sisyphus nhận ( ) a – b Ta có :
(a−1)(a−2) +
b(b+1) +
c(c−1)
2 +s+a−b
= a(a2−1) +b(b2−1) +c(c−21) +s
Do đến số sỏi trở ban đầu số tiền Sisyphus nhận số tiền ban đầu Mà ban đầu Sisyphus khơng có tiền dó đến thời điểm Sisyphus khơng có tiền
Bài toán 31 ( IMO Shortlisted 1994, Thụy Điển ) Có 1994 gái ngồi quanh bàn trịn, họ chơi chung cỗ gồm n Ban đầu, cô giữ tất Cứ nước đi, có gái giữ tối thiểu bài, gái phải chuyển cho hai gái bên cạnh Trị chơi kết thúc cô gái giữ nhiều a) Chứng minh n ≥ 1994 trị chơi khơng thể kết thức b) Chứng minh n <1994 trị chơi bắt buộc phải kết thúc
Lời giải a) Nếu n > 1994 theo ngun tắc Đirichlet có gái giữ bài, trị chơi khơng thể kết thúc Giả sử n = 1994 Gọi cô gái G1, G2, , G1994 giả
sử ban đầuG1 tất Ta định nghĩa giá trị tạm thời i nếuGi
đang giữ nó, ≤ n ≤ 1994 Gọi S tổng giá trị tạm thời Ban đầu S = 1994 Nếu cô gái khác vớiG1 G1994 chuyển S khơng thay đổi Nếu G1 G1994
(117)b) Khi cô chuyển cho cô lần thứ nhất, hai đánh dấu tên lên Lần sau đó, hai người phải chuyển bài, cô ta đưa đánh dấu cho người Nếu làm thế, đánh dấu kẹt lại hai gái kề nói Nếu n < 1994, có gái kề không chuyển cho Bây giờ, giả sử làm mà trị chơi khơng kết thúc được, phải tồn gái chuyển đến vơ hạn lần Từ suy có chuyển đến vơ hạn lần, cô kề bên cô ta chuyển hữu hạn lần mà Khi cô kề bên thực khơng chuyển ( chuyển hữu hạn lần ) đống tiếp tục tăng lên Điều rõ ràng mâu thuẫn
Bài toán 32 ( Bungary MO 1999 ) Ba đống sỏi có 51, 49 viên Ta thực hai nước sau Một nước dồn hai đống tùy ý thành đống Nước khác chọn đống có số chẵn viên sỏi để chia thành hai đống Hỏi thực dãy nước để chia ba đống sỏi thành 105 đống mà đống có viên sỏi hay khơng?
Lời giải Ban đầu số sỏi ba đống 51, 49 viên số lẻ nên bước phải dồn hai đống lại
Trường hợp : Dồn hai đống có 49 viên ta có hai đống 51 54 viên, đống bội Bước thứ hai ta chia đống có 54 viên thành dống có 27 viên Bây số sỏi ba đống 51, 27, 27 chia hết cho Vì ba đống có số lẻ viên nên bước thứ ba ta lại phải gộp hai đống 27 51 viên thành đống 78 viên Vì hai số 27 51 chia hết tổng ( = 78 ) chúng chia hết cho Tức thực nước luân phiên số sỏi đống ln bội Đây bất biến trường hợp Thật gộp hai đống sỏi có số sỏi chia hết cho đống sỏi có số sỏi chia hết cho chia đống sỏi ( gộp hai đống có số sỏi lẻ chia hết cho ) có số chẵn viên sỏi chia hết cho thành hai phần số sỏi phần chia hết cho Do số sỏi đống chia hết cho nhiều 35 đống, đống viên
Trường hợp : Bước dồn hai đống có 51 viên, ta hai đống có 49 56 viên, hai bội Khi thực bước luân phiên số sỏi đống nhận bội Do số đống với số sỏi nhỏ 15 đống đống viên Trường hợp : Bước dồn hai đống có 49 51 viên, ta hai đống 100 viên, số sỏi đống bội Khi thực hai nước đi, số sỏi đống nhận bội Do số đống với số sỏi nhỏ 21 đống, đống viên
Kết luận : Không thể chia ba đống sỏi thành 105 đống đống viên sỏi
(118)Lời giải Người chơi thứ thắng n = n chẵn Người chơi thứ hai thắng với n lẻ vàn >3 Ta kiểm tra kết luận đến n = sau chứng minh quy nạp
1) Nếu n > chẵn, người thứ tạo cột cỡ ( có đồng xu ) cột cỡ n – ( có n – đồng xu ) Vì n – lẻ nên người thứ thắng theo giả thiết quy nạp ( người chơi thứ trở thành người chơi thứ hai vị trí xuất phát có số lẻ đồng xu )
2) Nếu số lẻ, người chơi thứ tạo cột có số chẵn đồng xu cột có số lẻ đồng xu Người thứ hai lại chia cột có số chẵn đồng xu thành hai cột có số lẻ đồng xu Tiếp tục vậy, người chơi thứ hai luôn đáp lại bước người chơi thứ đưa người thứ vào tình có số cột với số lẻ đồng xu Khi cột tiến tới cỡ ( đồng xu ), chúng khơng cịn liên quan đến bước chơi Vấn đề nguy kịch cho người chơi có cỡ cột tiền: có cách để người thứ hai thua người chơi thứ hai tạo cho người chơi thứ cột cỡ nhiều cột cỡ Nhưng trường hợp người chơi thứ phải tạo cột hai đồng xu cột đồng xu, cột đồng xu Dù trường hợp nào, người chơi thứ hai thắng bước cách làm giảm đi: trường hợp thứ nhất, cột đồng xu chia thành cột đồng xu cột đồng xu trường hợp thứ hai, người thứ hai chia cột đồng xu thành hai cột cột đồng xu Tóm lại chiến thuật thắng người thứ hai luôn tạo toàn cột tiền lẻ trừ dẫn đến tình tất cột đồng xu cột có đồng xu, trường hợp chiến thuật thắng thực mô tả tả
Bài tốn 34 ( Tạp chí Kvant ) Cho dãy số 1, 0, 1, 0, 1, Từ số hạng thứ số chữ số tận tổng số hạng trước Chứng minh dãy số không chứa số hạng liên tiếp 0, 1, 0, 1, 0,
Lời giải Ta phát biểu lại toán sau :
Một số(x1, x2, x3, x4, x5, x6)được biến đổi thành (x2, x3, x4, x5, x6, x7)với x7 chữ
số tận tổngx1+x2+x3+x4+x5+x6 Hỏi nhận ( 0, 1, 0, 1, 0, ) từ
bộ ( 1, 0, 1, 0, 1, ) cách áp dụng phép biến đổi qua hữu hạn bước thực không? Ta chứng minh điều khơng thể cách thiết lập bất biến không đổi qua phép biến đổi Thật gọi s (x1, x2, x3, x4, x5, x6) chữ số tận tổng 2x1+ 4x2+ 6x3+ 8x4+ 10x5+ 12x6
Vì s(x2, x3, x4, x5, x6, x7) - s(x1, x2, x3, x4, x5, x6)
= 2x2+ 4x3+ + 10x6+ 12 (x1+x2+x3+x4+x5+x6)−2x1−4x2− −12x6
≡10 (x1+x2+x3+x4+x5+x6)≡0(mod10)
Từ cho thấy s (x1, x2, x3, x4, x5, x6) bất biến Vì s(1, 0, 1, 0, 1, ) = 18 s(0, 1, 0, 1, 0,
1 ) = 24 nên xuất ( 0, 1, 0, 1, 0, ) dãy số
5 Một số toán luyện tập vấn đề này:
(119)Bài toán 36 ( IMO Shortlist 1998, Iran ) Cho bảng kích thước m x n Một gồm mn quân bài, quân có hai mặt trắng đen Ta thực sau : đặt tất quân vào bảng, để mặt trắng ngửa lên tất ô ngoại trừ ô góc bảng để ngửa mặt đen Mỗi bước ta lấy quân có mặt đen khỏi bảng đồng thời đổi mặt tất quân nằm ô chung đỉnh với ô vừa bị loại bỏ Hãy xác định tất cặp ( m, n ) để tất quân bị loại bỏ khỏi bảng
Bài toán 37 ( IMO 1993 ) Trên bàn cờ có vơ hạn người ta quy ước trò chơi sau : Đầu tiên, mảnh xếp thành khối n x n hình vng kề nhau, mảnh đặt hình vng Một lần di chuyển ( nước ) tức lần nhảy theo chiều ngang chiều đứng băng qua hình vng chiếm chỗ kề để đến hình vng khơng bị chiếm chỗ tiếp liền theo sau Mảnh bị nhảy qua coi dời chỗ Trò chơi kết thúc cịn mảnh bàn cờ Tìm giá trị n để trò chơi kết thúc
Bài toán 38 ( Anh 2000 ) Alice chơi trị chơi bàn cờ 20 X 20 Khởi đầu Alice trải bàn cờ ô đồng xu gồm dạng sau đây: 100 penny, 100 nickel, 100 dime 100 quarter Alice chọn 59 đồng xu lấy khỏi bàn cờ Sau lần Alice lấy đồng xu theo ngun tắc sau đây:
•Một đồng penny lấy có hình vng ( trên, , phải , trái ) bỏ trống Những ngồi bàn cờ khơng tính ô trống theo nguyên tắc Ví dụ ô góc bàn cờ bên cạnh bàn cờ, chí có bên cạnh trống đồng xu khơng tính theo quy tắc
• Một đồng Nikel lấy tồn trống bên cạnh ( ngồi bàn cờ khơng tính trống )
• Một đồng Dime lấy có bên cạnh trống ( ngồi bàn cờ khơng tính trống )
• Một đồng Quarter lấy có ô trống bên cạnh ( ô ngồi bàn cờ khơng tính trống ) Alice thắng Alice lấy tất đồng xu bàn cờ Chứng minh khơng có khả thắng Alice
Tài liệu tham khảo
[1] T Andreescu, R Gelca,Mathematical Olympiad Challenges,, Birkhauser, 2000
[2] Nguyễn Hữu Điển, Giải Toán phương pháp Đại lượng Bất biến, NXB Giáo dục, 2005 [3] Hoàng Ngọc Minh,Một số phương pháp xây dựng nghiệm cho phương trình Diophante, Tài
liệu tập huấn giáo viên Chuyên Toán, Hà Nội, 2011
(120)MỘT SỐ DẠNG TỐN LIÊN QUAN ĐẾN DÃY SỐ CĨ QUY LUẬT
Lê Thị Thanh Hằng, Nhà XBGD Việt Nam
Trong chương trình số học, ngồi tập tính tốn đơn giản dựa quy tắc, tính chất phép tính mà học sinh rèn luyện thông qua tập SGK SBT, cịn có dạng tập tính tốn dãy số, dãy phân số có quy luật mà dựa vào quy luật tính tốn đó, học sinh giải tốn cách sáng tạo, lơgic, đem lại nhiều hứng thú say mê học học tập, phát triển tư duy, trí tuệ, phát huy lực sáng tạo, khiếu toán học học sinh
Trong chuyên đề này, đề cập số dạng toán tính tốn dãy số, dãy phân số có quy luật vài trải nghiệm định hướng tư phát triển tư học sinh nhằm bồi dưỡng lực học toán cho em học sinh có khả học giỏi tốn
1 Tìm số số hạng dãy số có quy luật
Với dạng tập dãy số, dãy phân số có quy luật, ta thường dùng phương pháp sau:
- Phương pháp phân tích số hạng tổng quát khử liên tiếp để tính tổng dãy số, dãy phân số có quy luật, giải tốn tìm x,và tốn có liên quan
- Phương pháp làm trội để chứng minh bất đẳng thức toán liên quan Với phương pháp ta thường dùng tính chất bất đẳng thức để đưa vế bất đẳng thức dạng tính tổng hữu hạn tích hữu hạn
Để tính tổng
Sn =a1+a2 +a3+· · ·+an
Ta biểu diễn
i=i, n, , qua hiệu hai số hạng liên tiếp dãy số khác Chẳng hạn
a1 =b1−b2;a2 =b2−b3; .;an=bn−1−bn
⇒Sn=a1+a2+a3+· · ·+an =b1−bn
Để tính tích hữu hạnPn =a1.a2.a3 an ta biến đổi ak thương hai số hạng liên tiếp
nhau :
a1 =
b1
b2
; a2 =
b2
b3
; .;an=
bn−1
bn
⇒Pn =a1.a2, a3 an=
b1
b2
.b2 b3
.bn−1 bn
= b1
bn
(121)Bài tốn Tìm n cho tổng 2n số hạng
1 1.3 +
1 2.4+
1
3.5 +· · ·+
1
(2n−1).(2n+ 1) +
1
2n(2n+ 2) =
14651 19800
Giải Đặt
A= 1.3 +
1 2.4 +
1
3.5+· · ·+
1
(2n−1).(2n+ 1) +
1 2n(2n+ 2)
Ta có
k(k+ 2) =
k −
1
k+ với k = 1;n 2A=
1.3+
3.5+· · ·+
2
(2n−1).(2n+ 1)
+ 2.4 +
2
4.6+· · ·+
2 2n(2n+ 2)
2A=1−
3 + −
1
5 +· · ·+ 2n−1 −
1 2n+
+1 2− 4+ 4−
6 +· · ·+ 2n −
1 2n+
2A=1−
2n+
+1 −
1 2n+
=
2n+ + 2n+
A= 14651 19800 ⇒
3 2−
2n+ + 2n+
= 14651 19800
⇒
2n+ + 2n+ =
199 9900
⇒ 4n+
(2n+ 1)(2n+ 2) = 199 9900
Do
(4n+ 3; 2(2n+ 1)) = (4n+ 3; 4n+ 2) =
và
(4n+ 3; 2(2n+ 2)) = (4n+ 3; 4n+ 4) =
⇒ 4n+
(2n+ 1)(2n+ 2)
là phân số tối giản vớin số tự nhiên ⇒
(
4n+ = 199
(2n+ 1)(2n+ 2) = 9900 ⇒n= 49
Bài tốn Tìm số ngun dương n thỏa mãn
2.22+ 3.23+ 4.24+ 5.25+· · ·+n.2n= 2n+10
Giải Ta có
(122)hay
1.2 + 2.22+ 3.22+ 4.24 + 5.25+· · ·+n.2n= 2n+10+
Vế trái biến đổi sau :
2 + 22+ 23+ 24+· · ·+ 2n= 2n+1−2 22+ 23+ 24+· · ·+ 2n= 2n+1−22
23+ 24+· · ·+ 2n= 2n+1−23
2n= 2n+1−2n
nên
1.2 + 2.22+ 3.23+ 4.24+ 5.25+· · ·+n.2n =n.2n+1−(2n+1−2) = 2n+1(n−1) +
⇒2n+1(n−1) + = 2n+10+
⇒n−1 =
n+10 2n+1 =
9 ⇒n = 29+ = 513
Bài tập áp dụng
Bài toán Cho
Q= √
b2 −
√
b1
−√
b3−
√
b2
+ √
b4−
√
b3
− · · ·+ (−1)n√
bn−
p
bn−1
Trong đób1;b2;b3;b4; .;bn−1;bn; số hạng cấp số cộng (dãy số cách đều)
a) TínhQ
b) Biết cơng sai (khoảng cách hai số hạng liên tiếp nhau) 17, số hạng thứ bội của17 khoảng từ 200 đến 500, tính n choQ= hoặcQ=−1
Giải
a) Ta có
Q= √
b2−
√
b1
− √
b3−
√
b2
+√
b4 −
√
b3
− · · ·+ (−1)n√
bn−
p
bn−1
⇒Q=
√
b2+
√
b1
b2−b1
− √
b3+
√
b2
b3−b2 +
√
b4+
√
b3
b4−b3
− · · ·+ (−1)n
√
bn+
p
bn−1
bn−bn−1
⇒Q=
√
b2+
√
b1−
√
b3 −
√
b2+
√
b4+
√
b3− · · ·+ (−1)n(
√
bn+
p
bn−1)
d
với d=bk−bk−1
⇒Q=
p
bn−1 +
√
b1
d Q=
√
b1−
p
bn−1
(123)tùy theo n chẵn hoặcn lẻ
b) Do d = 17, b1 bội 17 khoảng từ 200 đến 500 Tính n cho Q =
Q=−1
Bài toán Viết tất phân số sau thành dãy
1 1; 1; 2; 1; 2; 3; 1; 2; 3; 4;
a) Hãy nêu quy luật viết dãy viết tiếp năm phân số theo quy luật b) Phân số 50
31 số hạng thứ dãy
Giải
a) Quy luật dãy : Các phân số theo nhóm có tổng TS MS số tự nhiên liên tiếp
1 1; 1; 2; 1; 2; 3; 1; 2; 3; 4; 1; 2; 3; 4; 5; 5phân số dãy :
5 1; 2; 3; 4;
b) Phân số có tổng T S M S 81 Ta nhận thấy tổng T S M S k số phân số viết (k−1).Số phân số viết từ đầu đến có tổng T S M S là80 :
1 + + +· · ·+ 79 = (1 + 79).79
2 = 40.79 = 3160
Các phân số có tổng T S M S 81được bắt đầu sau :
Như đến phân số có 31 phân số viết thêm Vậy phân số 50
31 số hạng thứ (3160 + 31) = 3191
của dãy
Bài toán Cho
1 +
1 + 22
1 + 23
1 + 222009
=
1−
22n
Tìm n?
2 Các dạng tốn liên quan đến bất đẳng thức
Bài toán So sánh
A= 22 +
1 32 +
1
42 +· · ·+
(124)Giải Dựa vào
n2 <
n2−1 với (∀n≥2)
⇒A < B = 22−1 +
1
32−1 +· · ·+
n2−1
Do
B =
1.3 + 2.4+
2
3.5 +· · ·+
2 (n−1)(n+ 1)
B =
1−
3 + − + −
5+· · ·+
n−1−
n+
B =
1 + +
1
3 +· · ·+
n−1
− + +
5 +· · ·+
n−1+
n +
1
n+
B =1 + 2−
1
n −
1
n+
< =
4 <1⇒B <1
Vậy A < B <1 hay A <1
n2 <
(n−1)n ⇒A <
1 1.2 +
1
2.3+· · ·+ (n−1)n
Bài toán So sánh:
P = 22 +
1 42 +
1
62 +· · ·+ (2n)2 với
1
Giải Ta có
P = 22 +
1 42 +
1
62 +· · ·+
(2n)2 < Q= 22−1 +
1 42−1 +
1
62−1+· · ·+ (2n)2−1
Do
1 2n−1−
1 2n+ =
2
(2n−1)(2n+ 1)
⇒2Q= 1.3 +
2 3.5 +
2
5.7+· · ·+
2
(2n−1).(2n+ 1)
⇒2Q= 1−1
3 + − 5+ 5−
7 +· · ·+ 2n−1+
1 2n+
⇒2Q= 1−
2n+ <1⇒Q <
Do P < Q⇒P <
2 (đpcm)
Bài toán So sánh
1
999999 1000000
(125)Giải Ta có
A=
999997 999998
999999 1000000 < A
∗ =
999998 999999
1000000 1000000
⇒A2 < A.A∗ = 2 3 6 7 8
999997 999998 999998 999999 999999 1000000 1000000 1000000
⇒A2 =
10002 ⇒A < 1000
Từ tập ta chứng minh BĐT chặt chẽ :
1
2n−1 2n <
1
√
3n+ với n ≥1
Bài toán Chứng minh với số tự nhiênn ≥1ta có :
A= + 22 +
1 32 +
1
42 +· · ·+
n2 >
n n+
Bài toán 10 Chứng minh với số tự nhiên n≥1 ta có :
1 + 13 + 25+
41+· · ·+
1
n2+ (n+ 1)2 < 20
Bài toán 11 Chứng minh
S = +
1 22 +
1
23 +· · ·+ 27 <1
Bài toán 12 Chứng minh
A= +
2 32 +
3
33 +· · ·+ 101 3101 <
3
Giải Ta có
2A= 3A−A =
1 + +
3 32 +
4
33 +· · ·+ 100
399 + 101 3100
−1
3+ 32 +
3
33 +· · ·+ 101 3101
⇒2A <1 + 3+
1 32 +
1
33 +· · ·+
3100 = +
1−
3100
<1 + =
3
⇒A <
4
Tổng quát :
1 3+
2 32 +
3
33 +· · ·+
n
3n <
3
với số tự nhiên n≥1
Bài toán 13 Chứng minh
S = 32 +
2 33 +
3
34 +· · ·+ 100 3101 <
(126)Giải Ta có
⇒2S= 3S−S=1 3+
2 32 +
3 33 +
4
34 +· · ·+ 100 3100
−1
32 + 33 +
3
34 +· · ·+ 100 3101
⇒2S= 3+
1 32 +
1 33 +
1
34 +· · ·+ 3100 −
100 3101
⇒2S <
3+ 32 +
1 33 +
1
34 +· · ·+ 3100 =
1
1−
3100
<
2
⇒S= 32 +
2 33 +
3
34 +· · ·+ 100 3101 <
1
Tổng quát : Với mọin ∈
mathbbN∗, a∈
mathbbN∗;a 6= ta chứng minh : a) S1 =
1
a +
1
a2 +
a3 +
a4 +· · ·+
an <
1
a−1
b) S2 =
a +
2
a2 +
a3 +
a4 +· · ·+
n an <
a
(a−1)2
c) S3 =
a2 +
a3 +
a4 +
a5 +· · ·+
n an+1 <
a
(a−1)2
Bài toán 14 Cho
S1 = + 5;
S2 = + 5+
1 52;
S3 = + 5+
1 52 +
1 53; .;
Sn=Sn−1+ 5n
Chứng minh :
1 5S2
1 +
52S2
+ 53S2
3
+· · ·+ 5nS2
n
< 35
36
Bài toán 15 Biết
a21 +a22+a32+a24+a25 =
Chứng minh : giá trị nhỏ (ai−aj)2 (1≤i6=j ≤5) vượt
1 10
Bài toán 16 Chứng minh √
n+ 1−√n >
2√n+ với n∈mathbbN
Từ chứng minh
1 + √1
2 + √ + √
4 +· · ·+ +
√
1994 <2
√
(127)Giải Ta có
1 2√n+ =
1
√
n+ +√n+ <
1
√
n+ +√n =
(√n+ 1−√n)
(√n+ +√n)(√n+ 1−√n)
hay
1 2√n+ <
√
n+ 1−√n ( đpcm) Bài toán 17 Chứng minh
1 5+
1
13+· · ·+
1
20022+ 20032 <
Bài toán 18 Chứng minh
1 2√1 +
1
3√2 +· · ·+
(n+ 1)√n với n∈mathbbN;n≥1
Bài toán 19 Cho biểu thức :
A= +
8 +
15
16 +· · ·+ 9999 10000
Chứng minh rằng98< A <99
Tổng qt hóa tốn, ta có
n−2<
4 + +
15
16+· · ·+
n2−1
n2 < n−1 với n ≥2
Bài toán 20 Cho tổng
Sn= +
1 + +
1
1 + + +· · ·+
1
1 + + +· · ·+n
Tìm số hữu tỷ s nhỏ đểSn < avới n∈
mathbbN∗
Bài toán 21 Cho
A= 14+
1
29+· · ·+
1
n2+ (n+ 1)2+ (n+ 1)2 +· · ·+ 1877
Chứng minh
0,15< A <0,25
Ngồi chứng minh toán tổng quát :
1 −
1 3(k+ 2) <
1
12+ 22+ 32 +
1
22+ 32+ 42 +· · ·+
1
k2+ (k+ 1)2 + (k+ 2)2
<1
(128)Bài tốn 22 Tìm A
B biết A=
2.32+
3.33 +· · ·+
n(n+ 30) +· · ·+ 1973.2003
B =
2.1974 +
3.1975 +· · ·+
1
n(n+ 1972)+· · ·+ 31.2003
Áp dụng (??) với k= 30 ta có
30A= 30 2(2 + 30) +
30
3(3 + 30) +· · ·+
30
1973(1973 + 30) 30A=1
2 − 32 + −
33 +· · ·+ 1973 +
1 2003 30A=1
2 +
3 +· · ·+ 31
−
1974 +
1975 +· · ·+ 2003
(1) Áp dụng (??) với k = 1972 ta có
1972B = 1972 2(2 + 1972) +
1972
3(3 + 1972) +· · ·+
1972 31(31 + 1972) 1972B =1
2− 1974
+1 3−
1 1975
+· · ·+ 31−
1 2003
1972B =
1
2+
3+· · ·+ 31
−
1974 +
1975 +· · ·+ 2003
(2) Từ (1) (2) ta có :
30A = 1972B ⇒ A
B =
1972 30 =
986 15
Bài toán 23 Chứng minh r»ng
A= 32 −
1 34 +
1
36 − · · ·+ 34n+2 −
1
34n +· · · −
1
3100 <0,1
Hãy tổng quát hóa toán
Bài tập vận dụng
Bài toán 24 Chứng tỏ tổng 100 số hạng dãy sau nhỏ
4 5; 45; 117; 221; 357;
Bài toán 25 Chứng minh
A = +
8 +
15
(129)Bài toán 26 Chứng minh
a) n!>2n−1 (n ≥3)
b) +b+b2+· · ·+bn= 1−b
n−1
1−b (b6= 1)
c) + 1! +
1
2! +· · ·+
n! <3
Bài toán 27 Cho số dươnga1;a2; .;an Chứng minh
Cn2 a1a2
+
a1a3 +
a1a4
+· · ·+
a1an
+
a2a3 +
a2a4
+· · ·+
an−1an
≥
4
a1+a2
+
a1+a3
+
a1+a4
+· · ·+
a1+an
+
a2+a3
+
a2+a4
+· · ·+
an−1+an
Tìm điều kiện củaak(k = 1; 2; 3; 4; .;n) để có đẳng thức
Bài toán 28 Cho số tự nhiên a1 < a2 <· · ·< an Chứng minh tổng A:
A=
√
a2−a1
a2 +
√
a3−a2
a3
+· · ·+
√
an−an−1
an
<1 + +
1
3 +· · ·+
n2
Bài toán 29 Chứng minh
1
n+ +
n+ +
n+ +· · ·+ 2n >
1
2 với n ∈mathbbN;n >
Bài toán 30 Chứng minh
1 √ + √ + √
3 +· · ·+
√
n >
√
n với n ∈mathbbN;n >1
Bài toán 31 Chứng minh
2(√n+ 1−1)<1 + √1
2+
√
3+· · ·+
√
n >2
√
n−1 với n ∈mathbbN
Bài toán 32 Chứng minh
1 +
1
25 +· · ·+
(2n+ 1)2 <
4 với n∈mathbbN;n ≥1
Bài toán 33 Chứng minh
1 + 13+ 25+
41+· · ·+
1
(130)Bài toán 34 Chứng minh
1 65 <
1 53 +
1
63 +· · ·+
n3 +· · ·+ 20043 <
1 40
Bài toán 35 Chứng minh
1 3+
2 32 +
3 33 +
4
34 +· · ·+ +
n
3n <
3
Bài toán 36 Chứng minh
Sn=
3 12.22 +
5 22.32 +
7
32.42 +· · ·+
2n+
n2.(n+ 1)2 +
n2
4(13+ 23+· · ·+n3)
là số
Bài toán 37 Chứng minh a) − + − 16+ 32 − 64 < b) − 32 +
3 33 −
4
34 +· · ·+ 99 399 −
100 3100 <
3 16
c)
52 − 53 +
3 54 −
4
55 +· · ·+ 99 5100 −
100 5101 <
1 36
Bài toán 38 So sánh tổngA gồm 11 số hạng sau với
16
A= 52 +
2 53 +
3
54 +· · ·+
n
5n+1 +· · ·+ 11 512
Tổng qt tốn, ta có: Với a, nlà số tự nhiên khác 0, a >1
A=
a2 +
a3 +
a4 +· · ·+
n a−n+1 <
1 (a−1)2
Bài toán 39 Chứng minh
1 +a2 −
a
(1 +a2)2 +
a2 (1 +a2)3 −
a3
(1 +a2)4 +· · ·+
(−1)n+1.an−1
(1 +a2)n +· · · ≤
3
3 Các toán tổng hợp
1 Toán chia hết
Bài toán 40 Cho
A= 1.2.3.4 .1001;
B = 1002.1003.1004 .2002
(131)Giải Ta có
B = (2003−1001)(2003−1000)(2003−999) .(2003−1)
⇒B =BS(2003)−1.2.3.4 .1001 =BS(2003)−A
⇒A+B =BS(2003)
Vậy (A+B)chia hết cho 2003
Bài toán 41 Viết tổng
2 1+
22 +
23
3 +· · ·+ 2n
n
về dạng phân số P
S
Chứng minh rằngp với n >3
Bài toán 42 Cho tổng
1 + +
1
3 +· · ·+ 18 =
a
b với (a;b) =
Các mẫu số số hạng tổng số tự nhiên liên tiếp từ2 đến 18
Chứng minh :
b 11.13.17
Tổng qt tốn, ta có : Nếu plà số ngun tố, n số tự nhiên thỏa mãnp < n <2pvà tổng
1 + 2+
1
3+· · ·+
n−1+
n = a
b với (a;b) =
Thì b p
2 Tốn tìm x
Bài tốn 43 Tìm x biết
11.13+ 13.15+
2 15.17
.561−[3,6 : (x−9,52) : 1,2] = 10
Giải
11.13+ 13.15+
2 15.17
.561−[3,6 : (x−9,52) : 1,2] = 10
⇔
11− 13+ 13 − 15 + 15 − 17
.561−10 = [3,6 : (x−9,52) : 1,2]
⇔
11− 17
.561−10 =
18
5 : (x−9,52) :
⇔
187.561−10 =
18
5
6 : (x−9,52)
⇔18−10 = [3 : (x−9,52)]⇔8 = : (x−9,52)⇔(x−9,52) = :
(132)Bài tốn 44 Tìm x biết a)
11.13+ 13.15 +
1
15.17+· · ·+ 19.21
: 0,75x+
x = 231 b) 17 13 − + 52
x− 66
44
= 1.4 +
1 4.7+
1 7.10+
1 10.13
c)
5.8+ 8.11 +
1
11.14+· · ·+
x(x+ 3) = 101 1540
d) + +
1 +
1
10 +· · ·+
1
x(x+ 1) : = 1991 1993
e)
21.22+
22.23 +· · ·+ 29.30
.140 + 1,08 : [0,3.(x−1)] = 11
f)
31.33+
33.35 +· · ·+ 39.41
.2542−[2,04.(x+ 1,05)] : 0,12 = 19
Bài tốn 45 Tìm tỷ số hai số A B biết :
A=
1.1981 + 2.1982 +
1
3.1983 +· · ·+
1
n(1980 +n)+· · ·+ 25.2005
B =
1.26 + 2.27+
1
3.28 +· · ·+
1
m(25 +m) +· · ·+ 1980.2005
Trong đóA có 25số hạng, B có 1980 số hạng
Bài tốn 46 Tìm tỷ số hai số A B biết : A có(n−1)thừa số
A=1−
1 +
1−
1 + +
1−
1 + + +
.1−
1 + +· · ·+n
B = n+
n
Bài tốn 47 Tính M
N biết
M =
3.5 + 5.7+
1
7.9 +· · ·+ 29.31
N =
3.5.7+ 5.7.9+
1
7.9.11 +· · ·+ 27.29.31
Bài tốn 48 Tính A
B biết
A=
2.32 +
3.33+· · ·+
n(n+ 30) +· · ·+ 1973.2003
B =
2.1974 +
3.1975 +· · ·+
1
n(n+ 1972)+· · ·+ 31.2003
(133)Tài liệu tham khảo
[1] Phan Huy Khải(2004), "Chuyên đề bồi dưỡng học sinh giỏi tốn trung học phổ thơng ", Các toán số học, NXB Giáo dục
[2] Hà Huy Khoái, "Chuyên đề bồi dưỡng học sinh giỏi tốn trung học phổ thơng",Số học, NXB Giáo dục
[3] Nguyễn Văn Mậu (chủ biên), Một số vấn đề số học chọn lọc, NXB Giáo dục [4] Đặng Hùng Thắng (1995), Bài giảng số học, NXB Giáo dục
[5] Đặng Hùng Thắng, Nguyễn Văn Ngọc, Vũ Kim Thủy(1997), Bài giảng số học,Tuyển tập 30 năm tạp chí tốn học tuổi trẻ, NXB Giáo dục
(134)VẬN DỤNG TÍNH ĐƠN ĐIỆU TRONG CÁC BÀI TỐN TÌM GIỚI HẠN DÃY SỐ VÀ GIẢI PHƯƠNG
TRÌNH, BẤT PHƯƠNG TRÌNH, HỆ PHƯƠNG TRÌNH
Trương Văn Điềm, Trường THPT Chuyên Lê Quý Đôn - Khánh Hịa
Trong nhiều năm qua, tốn giới hạn dãy số, phương trình, hệ phương trình dạng thường gặp kỳ thi chọn học sinh giỏi quốc gia cấp tỉnh Đây dạng phần lý thuyết đơn giản, viết đề cập đến việc vận dụng tính đơn điệu để giải
1 Lý thuyết bản
Các toán dãy số, giải phương trình, bất phương trình hệ phương trình có nội dung đa dạng Ở ta quan tâm tốn tìm giới hạn dãy số (bản chất giải tích) tốn giải phương trình, bất phương trình hệ phương trình vận đụng tính đơn điệu hàm số
Với toán giới hạn dãy số , ta cần nắm vững định nghĩa giới hạn dãy số định lý giới hạn dãy số, bao gồm:
1 Định lý Weierstrass: Dãy đơn điệu bị chặn hội tụ Định lý kẹp: Nếu xn≤yn≤zn với n≥n0 lim
n→∞xn= limn→∞zn=a nlim→∞yn =a Một dạng dãy số thường gặp dãy số xác định bởix0 =a, xn+1 =f(xn)
với f hàm số Và với loại dãy số này, câu hỏi thường gặp là: Chứng minh dãy số {xn} có giới hạn hữu hạn
2 Tìm tất giá trị a cho dãy số {xn} có giới hạn hữu hạn
Để giải toán dạng này, ta có số tính chất sau Nếu f hàm số tăng dãy {xn} dãy đơn điệu
2 Nếu f hàm số giảm dãy {x2n} (dãy với số chẵn) {x2n+1} (dãy với số
lẻ) dãy đơn điệu
3 Nếu với mọix, y ta có|f(x)−f(y)| ≤q|x−y| với qlà số 0< q <1và {xn}bị chặn
thì {xn} hội tụ Đặc biệt |f0(x)| ≤q <1 ta ln có điều
(135)2 Một số toán minh họa
2.1 Các toán dãy số
Bài toán Cho dãy số {xn} xác định x0 =
√
2 xn+1 =
√
2xn với n = 0,1,2, Chứng minh dãy {xn} có giới hạn hữu hạn tìm giới hạn
Lời giải Đặt f(x) = (√2)xn thì dãy số có dạng x
0 =
√
2 xn+1 = f(xn) Ta thấy f(x)
hàm số tăng vàx1 =
√
2
√
2
>√2 =x0 Từ đó, f(x)là hàm số tăng nên ta có x2 =f(x1)>
f(x0) =x1, x3 =f(x2) > f(x1) = x2, Suy {xn} dãy số tăng Tiếp theo, ta chứng minh
bằng quy nạp xn < với n Điều với n = Giả sử ta có xk <2 rõ
ràng xk+1 =
√
2xk <√22 = Theo nguyên lý quy nạp toán học, ta có xn <2 với mọin
Vậy dãy {xn}tăng bị chặn nên dãy có giới hạn hữu hạn Gọi a giới hạn
thì chuyển đẳng thứcxn+1 =
√
2xn sang giới hạn, ta đượca=√2a Ngồi ta cóa ≤2 Xét phương trình x = √2x ⇔ lnx
x = ln(
√
2) Khảo sát hàm số lnxx ta thấy phương trình có nghiệm < evà nghiệm lớn e Vì nghiệm phương trình nên rõ ràng có nghiệm phương trình thoả mãn điều kiện≤2 Từ suy
a=
Vậy giới hạn của{xn} khin dần đến vô
Bài toán (Đề dự bị VMO 2008) Cho số thực a dãy số thực {xn} xác định bởi:
x1 =avàxn+1 = ln(3 + cosxn+sinxn)−2008 với n= 1,2,3,
Chứng minh dãy số {xn} có giới hạn hữu hạn n tiến đến dương vô
Lời giải Đặt f(x) = ln(3 + cosxn+sinxn)−2008
f0(x) = cosx−sinx + sinx+ cosx
Từ đó, sử dụng đánh giá |cosx−sinx| ≤√2,|sinx+ cosx| ≤√2 ta suy |f0(x)| ≤
√
2
3−√2 =q <1
Áp dụng định lý Lagrange chox, y thuộc R, ta có
f(x)−f(y) =f0(z)(x−y)
Từ suy ra|f(x)−f(y)| ≤q|x−y| với x, y thuộcR Áp dụng tính chất vớim > n ≥N, ta có
|xm−xn|=|f(xm−1)−f(xn−1)| ≤q|xm−1−xn−1| ≤ · · · ≤qn−1|xm−n+1−x1| ≤qN−1|xm−n+1−x1|
Do dãy {xn} bị chặn q < nên với ε >0 tồn N đủ lớn để qN−1|xm−n+1−x1| < ε
(136)Bài toán ( Đề thi chọn HSG Nghệ An 2007) Chứng minh với số n nguyên dương , tồn số thực xn cho
1
2008xn −xn+n = Xét dãy số (xn), tìm lim(xn+1−xn)
Lời giải Với n ∈N∗, xétf(x) =
2008x −x+n;x∈R
f0(x) = −ln 2008
2008x −1 ∀x∈R ⇒ f(x) nghịch biến R (1)
Ta có (
f(n) = 20081 n >0
f(xn+1) = 20081n+1 −1<0
⇒f(x) = có nghiệm xn∈(n, n+ 1) (2)
Từ (1) (2) suy đpcm
Ta có xn−n = 20081xn >0⇒xn > n⇒0< xn−n < 20081 n
Mặt khác lim
2008n = ⇒ lim(xn−n) = Khi lim(xn+1 −xn) = lim{[xn+1−(n+ 1)]− (xn−n) + 1}=
Bài toán Cho a∈R xét dãy số thực (xn), n = 0,1,2,3, xác định
(
x0 =a
xn =
p
6xn−1−6 sin(xn−1)
Chứng minh rằng∀a∈R, dãy số(xn)ln có giới hạn hữu hạn n→+∞.Hãy tínhlim(xn)
Lời giải
+) Trường hợp 1: Xéta= xn= ∀n = 0,1,2,3, Suy limxn=
+) Trường hợp 2: Xéta >0 Ta ln có
x− x
3
6 ≤sinx≤x, ∀x≥0 (Dấu = xảy khix= 0)
Vìsinx≤x⇒xn≥0,∀n (Cm qui nạp)
Vìsinx≥x− x3
6 ⇒sinxn−1 > xn−1−
x3
n−1
6 ,∀n ≥1
⇒6xn−1−6 sinxn−1 < x3n−1 ∀n≥1
⇒xn =
√
6xn−1−6 sinxn−1 < xn−1 ∀n ≥1
Suy (xn) dãy giảm bị chặn 0, nên(xn)có giới hạn hữu hạn n →+∞
Đặt limxn =α≥0⇒α=
√
6α−6 sinα⇒α=
+) Trường hợp 3: Xéta <0 Đặtb =−a >0 Xét dãy(yn) xác định sau:
(
y1 =b
yn+1 =
√
6yn−6 sinyn vớin = 0,1,2,
Theo CM thì(yn) có giới hạn hữu hạn Suy limxn=
Kết luận: ∀a∈R, dãy (xn) có giới hạn hữu hạn limxn =
Bài tốn (Đề chọn HSG Thanh Hóa 2006) Cho dãy (xn) : x0 = 1, xn = 1+xn1−1 Tính
(137)Lời giải Nhận dãy dương và:x0 = 1, x2 = 23, x4 = 85 x1 = 12, x3 = 35
Chia dãy cho thành hai dãy con(x2n)và x2n+1
Ta chứng minh quy nạp (x2n) giảm
Ta có: x2n+2 = 1+x12n+1 = 2+1+xx22nn; x2n = 2+1+xx22nn−−22
Giả thiết x2n < x2n−2, cho ta x2n+2 < x2n⇔ 1+2+xx22nn <
1+x2n−2
2+x2n−2 ⇔ x2n < x2n−2
Lúc đó:x2n+2 = 2+1+xx22nn < x2n ⇔x2n> −1+
√
5
2 Tương tự với dãy (x2n+1), ta =x0 > >
x2n> −1+
√
5
1
2 =x1 < < x2n−1 <
−1+√5
Suy dãy (x2n) (x2n+1) đơn điệu, bị chặn Theo dấu hiệu Vaiơstrat, dãy có giới
hạn Khi gọiA = lim
n→∞x2n, B = limn→∞x2n+1
Ta hệ:A= 1+1B B = 1+1A Giải hệ ta :A=B = −1+
√
5
Dãy cho gồm hợp hai dãy có giới hạn Suy lim
n→∞xn= −1+√5
2
Bài toán (đề đề nghị Olympic 30 – năm 2001) Cho dãy số {xn} thoả mãn x1 = −52
xn+1 = 12x2n+xn−2 với mọin số nguyên dương Chứng minh dãy {xn} hội tụ Tìm giới
hạn
Lời giải Từ cơng thức xác định dãy ta có: xn+2 = 12xn2+1+xn+1 −2 = 12
1
2x
n+xn−2
2
+
2x
n+xn−4 = 18x4n+12x
n−xn−2
Xét hàm số: f(x) = 18x4 + 2x
3 − x− 2 ∀x ∈ (−2;−1) Ta có: f0(x) = 2x
3 + 2x
2 −1 và
f00(x) = 2x
3+3 2x
2−1<0 ∀x∈(−2;−1)
Vậy f0(x)> f0(−1) = (do f0(x) nghịch biến (−2;−1))
Do đóf(x)đồng biến khoảng(−2;−1), nên suy raf(−2)< f(x)< f(−1),∀x∈(−2;−1), hay −2< f(x)<−11
8 <−1, ∀x∈(−2;−1)
Nhưng x2n+2 =f(x2n) , từ suy −2< x2n<−1, với n = 1,2,3,
Mặt khác dox2 > x4 ta suy raf(x2)> f(x4)hay x4 > x6,
Hồn tồn tương tự ta có: −1> x2 > x4 > x6 > > x2n> >−2
Vậy dãy {x2n}giảm bị chặn nên hội tụ
Đặta = lim
n→∞x2n, từ công thức xác định dãy, chuyển qua giới hạn ta có:
a= 8a
4+1 2a
3−a−2⇔(a−2)(a+ 2)3 = 0⇔a=−2
Hoàn toàn tương tự ta chứng minh dãy{x2n+1}tăng bị chặn – lim
n→∞x2n = −2 Như dãy {xn} hội tụ có giới hạn –
Bài tốn ( đề đề nghị Olympic 30 – năm 2002) Cho dãy số {xn} xác định bởi: x0 = 2,7;x3
n+1−3xn+1(xn+1−1) = xn+ Chứng minh dãy số {xn} hội tụ
Lời giải Ta có: x3
n+1−3xn+1(xn+1−1) =xn+ ⇔(xn+1−1)3 =xn ⇔xn+1 =
√
xn+
Xét hàm số g(x) = +√3 x, ta có x
n+1 =g(xn) Ta có:
g0(x) =
3√3x2 ⇒0< g
0
(x) = 3√3
(138)Dox0 = 2,7⇒xn>2, ∀n ∈ Ta cóx=g(x)⇔x= 1+3
√
x⇔x−1 = √3x⇔x3−3x2+3x−1 =
x⇔f(x) =x3−3x2+ 2x−1 = 0
Ta có:f0(x) = 3x2−6x+ = 3x(x−2) + 2>0;∀x >2 Mặt khác:f(2) = −1<0;f(3) = 5>0 Do phương trìnhx = g(x) có nghiệm x = r ∈ (2; 3) Ta chứng minh: |xn−r| ≤
qn, ∀n.
Với n= :|x0 −r|=|2,7−r|<1 =q0
Giả sử|xk−r| ≤qk, ta có |xk+1−r|=|g(xk)−g(r)|
Áp dụng định lí Lagrange, ta có:
|xk+1−r|=|g0(c)(xk−r)|=|g0(c)| |xk−r| ≤ |g0(c)|.qk
Do xk, r >2⇒c∈(xk;r)∨c∈(r;xk)⇒c >2⇒0< g0(c)< q
Vậy|xk+1−r| ≤qk.q=qk+1 Theo ngun lí quy nạp ta có:|xn−r| ≤qn, ∀n ∈ Vậy lim n→∞xn =
r (do < q < 1)
Bài toán (đề đề nghị Olympic 30 – năm 2002) Cho dãy số{xn} xác định bởix1 =x2 =
và xn+2 = 52πx2n+1+ 2π
5 sinxn ; ∀n ∈ N
∗ Chứng minh dãy {x
n} có giới hạn tính giới
hạn
Lời giải Trước hết ta chứng minh: xn ∈ 0;π2
,∀n∈N∗ Thật vậy: x
1 =x2 = ∈ 0;π2
Giả sử có xn ∈ 0;π2
,∀k ≤ n Khi đó: xn+1 < 52π π2
2
+ 25π = π2 với xn ∈ 0;π2
xn+1 >0⇒xn+1 ∈ 0;π2
Theo nguyên lí quy nạp: xn∈ 0;π2
,∀n ∈N∗. Xét hàmf(x) = 52πx2+2π
5 sinx−x với x∈ 0;
π
2
f0(x) = 5πx+
2π
5 cos−1 vàf
00(x) = 5π −
2π
5 sinx
Ta có f00(x) = 0⇔ 5π −
2π
5 sinx= 0⇔x=arcsin
π2 =x0
Ta có bảng biến thiên f0(x):
x x0 π2
f00(x) + −
f0(x) 25π −1% &−3
Từ bảng biến thiên suy raf0(x) = 0có nghiệm nhấtx1 ∈ x0;π2
vàf0(x)>0vớix∈(0;x1)
,f0(x)<0 với x∈ x1;π2
Khi ta có bảng biến thiên hàm f(x):
x x1 π2
f0(x) + −
(139)Từ suy f(x)>0, ∀x∈ 0;π2 f(x) = x= 0;x= π2 Vậy dãy {xn} dãy bị chặn
(1)
Mặt khác từ công thức xác định dãy quy nạp, ta dễ dàng chứng minh dãy không giảm Từ suy dãy có giới hạn
Gọi a= lim
n→∞xn⇒1≤a≤
π
2 a thỏa mãn f(a) = suy a=
π
2 Vậy nlim→∞xn =
π
2
Bài toán ( Phan Huy Khải ) Dãy số thực xác định theo quy luật: x1 = 2,9;xn+1 =
√
3 +
xn
√
x2
n−1
với n ≥ Hãy tìm số thực nằm bên trái dãy {x1, x3, } bên phải dãy
{x2, x4, } dãy số {xn}
Lời giải Quy luật dãy số suy xn ≥
√
3 với số tự nhiên n ≥ Yêu cầu tốn tìm số thực a cho x2k < a < x2k−1 với số tự nhiên k= 1,2,3,
Dự đốn a cần tìm giới hạn dãy số {xn} n → ∞ Khi a nghiệm phương
trìnhx=√3 + √ x
x2−1 với x≥
√
3 (∗)
+) Giải phương trình (*) Vìx≥√3suy 0< 1x <1nên đặt x= sin1α;α∈ 0;π2 Khi (*) trở thành sin1α =√3 +cos1α ⇔sinα−cosα+√3 sinαcosα= Giải phương trình lượng giác ta tìm sinα =
√
3 ±
√
5−1 sinα > nên chọn sinα =
√
3
√
5−1 ⇒
x=
√
3
√
5 +
+) Lấy a =
√
3
√
5 + ta chứng minh x2k < a < x2k−1 với số tự nhiên k = 1,2,3,
bằng quy nạp
Dựa vào quy luật dãy số, xét hàm sốy=f(x) = √3 + √x
x2−1 (x≥
√
3) Hàm f liên tục có đạo hàm √3; +∞
; f0(x) = −
(x2−1)√x2−1 < ; ∀x ≥
√
3, suy hàm f nghịch biến trên√3; +∞
Ta có: Với k = x1 = 2,9 Khi a =
√
3
√
5 + <2,9 =x1 x2 =f(x1) =
√
3 + √2,9 2,92−1 <
f(a) = a Vậy với k = ta có x2 < a < x1 hay mệnh đề với k =
Giả sử mệnh đề với k =n ta chứng minh với k =n+ Thật vậy, giả thiết quy nạp x2n < a < x2n−1 Khi
x2n+1 =f(x2n)> f(a) =a
x2n+2 =f(x2n+1)< f(a) = a
Vậy ta có điều phải chứng minh: x2n+2 < a < x2n+1
Theo ngun lí quy nạp ta có: x2k < a =
√
3
√
5 + < x2k−1 với k = 1,2, tức
a=
√
3
√
5 + giá trị cần tìm
Bài tốn 10 (Đề thức - Olympic 30 – năm 2002 )
Cho phương trình xn+xn−1 + +x−1 = 0. Chứng tỏ với mỗi n nguyên dương thì
phương trình có nghiệm dương xn tìm lim n→∞xn
Lời giải Đặt fn(x) = xn+xn−1+ +x−1t hì fn(x) hàm số liên tục toàn trục số
Xét:
fn(x1)−fn(x2) = (xn1 −x
n
2) + x
n−1 −x
n−1
(140)
Nếu0< x2 < x1 fn(x1)−fn(x2)>0hay fn(x1)> fn(x2)Do hàm sốfn(x) hàm đồng
biến (0; +∞)(có thể dùng đạo hàm để chứng minh)
Hơn nữafn(0) =−1<0, fn(1) =n−1>0với n≥2nên phương trìnhfn(x) = 0có nghiệm
dương nhấtxn Vỉ1 = xn+x2n+ +xnn nên khin tăng xn giảm, tức dãy{xn}giảm
và bị chặn Do tồn tạix0 = lim
n→∞xn Mặt khác, = 1−xnn
1−xnxn 0< xn <1 nên cho qua giới hạn ta có x0
1−x0 = hay x0 =
1 Vậy lim
n→∞xn=
1
Bài toán 11 (VMO -2005) Xét dãy số thực (xn), n = 1,2,3, , xác định bởi: x1 = a
xn+1 = 3x3n−7x2n+ 5xn với n = 1,2,3, ,trong a số thực Hãy xác định tất
các giá trị a để dãy số (xn) có giới hạn hữu hạn khin →+∞ Hãy tìm giới hạn dãy số
(xn) trường hợp
Lời giải Xét hàm số f(x) = 3x3 −7x2 + 5x Khi đó, viết hệ thức xác định dãy (x
n)
dưới dạngxn+1 =f(xn)với n= 1,2,3,
•Ta có f0(x) = 9x2 −14x+ 5 Từ đó, ta có bảng biến thiên sau hàm f(x) :
x −∞
9 +∞
f0(x) + − +
f(x) −∞%
275 243
&1% +∞
Vìf(x)−x= 3x3−7x2 + 4x=x(x−1)(3x−4) nên
f(x) = x⇔x= x= x= 43 (1)
f(x)<0khi x <0 (2)
f(x)>0khi x >
3 (3)
Hơn nữa, dof(0) = 0, f(43) = 43 275243 < 43 nên từ biến thiên hàm f R suy ra: Với x∈(−∞; 0) ln có f(x)∈(−∞; 0) (4) Với x∈(0;43)ln có f(x)∈(0;43) (5) Với x∈(43;−∞)ln có f(x)∈(43;−∞) (6) Xét trường hợp sau:
•Trường hợp 1: a <0 Khi đó:
(4)⇒xn ∈(−∞; 0) ∀n ≥1
(2)⇒x2 =f(x1)< x1
Từ đó, hàm số f(x)đồng biến khoảng (−∞; 0), dễ dàng chứng minh dãy
(xn)là dãy số giảm Kết hợp điều với (1), suy (xn) dãy hội tụ limxn=α
thì phải cóα∈ {0; 1;43}vàα < a Vìa <0nên khơng thể có số α thỏa mãn đồng thời hai điều kiện nêu Điều chứng tỏ dãy (xn) khơng dãy hội tụ
•Trường hợp 2: a > 43 Khi đó:
(6)⇒xn ∈(43; +∞) ∀n ≥1
(141)Từ đó, hàm sốf(x)đồng biến khoảng(43; +∞), dễ dàng chứng minh dãy
(xn) dãy số tăng Kết hợp điều với (1), suy nếu(xn) dãy hội tụ limxn=α
thì phải cóα ∈ {0; 1; 43} vàα > a
Vìa > αnên không tồn sốα thỏa mãn đồng thời hai điều kiện nêu Điều chứng tỏ dãy(xn) không dãy hội tụ
•Trường hợp 3: a= Khi đó, dãy (xn)là dãy hằng: xn= ∀n≥1 Vì vậy,(xn)là dãy hội
tụ vàlimxn =
•Trường hợp 3: a= 43 Khi đó, dãy (xn)là dãy hằng: xn = 43 ∀n≥1 Vì vậy,(xn) dãy hội
tụ vàlimxn = 43
•Trường hợp 5: 0< a < 43 Khi đó, từ (5) suy xn ∈(0;43) ∀n ≥2.Ta có:
|xn+1−1|=|3x3n−7x2n+ 5xn−1|= (xn−1)2|3xn−1| ∀n≥1 (7)
Vìxn ∈(0;43) ∀n≥1 nên |3xn−1|<1 ∀n≥1 Do đó, từ (7) suy
|xn+1−1|<(xn−1)2 ∀n ≥1
Từ đó, quy nạp theon, dễ dàng chứng minh |x−n−1|<(a−1)2n−1
∀n≥1 (8)
Vìa ∈(0; 43)nên |a−1|<1 Do lim(a−1)2n−1 = Vì thế, từ (8) suy dãy(xn)là dãy
hội tụ vàlimxn=
Vậy tóm lại, dãy (xn)là dãy hội tụ a∈[0;43] Và đó:
• limxn= a=
• limxn= 43 a= 43
• limxn= a∈(0;43)
2.2 Dạng 2: Phương trình, hệ phương trình
1> Nếu hàm số y=f(x) ln ln tăng( giảm) D phương trình f(x)=0 có nhiều nhiệm D
2>Nếu f(x)=0 có đổi dấu lần phương trình f(x)=0 có nhiều hai nghiệm D
Bài toán Giải phương trình −2x3+ 10x2−17x+ = 2x2√3
5x−x3 (1)
Lời giải x = nghiệm phương trình nên (1) ⇔ −2x3 + 10
x −
17
x2 +
8
x3 =
2q3
x2 −1
Đặtt = x1 (t6= 0)
8t3−17t2+ 10t−2 = 2√3
5t2−1
⇔ (2t−1)3+ 2(2t−1) = 5t2−1−2√3
(142)Xétf(x) =x3+ 2x ⇒ f00(x) = 3x2+ 2>0∀x∈R suy f tăng R
Từ (2), ta có:
f(2t−1) =f(√3
5t2−1)
⇔ 2t−1 = √3
5t2−1.
⇔8t3 −12t2+ 6t−1 = 5t2−1.
⇔
t = (loại)
t = 17±
√
97 16
Vậy nghiệm phương trình x= 17±
√
97 12
Bài tốn Giải phương trình 12log2(x+ 2) +x+ = log22xx+1 + + x12+ 2√x+
Lời giải Điều kiện : −2< x <−1
2; x >0
Phương trình tương đương
log2√x+ 2−2√x+ +x+ = log2
2 +
x
+
2 +
x
2 −2
2 +
x
Đặtf(t) = log2t+t2−2t (t >0)
Ta có
f0(t) = tln 21 + 2t−2
f00(t) = 2−
t2ln 2
f00(t)>0 ⇔ 2> t21ln 2 ⇔t >
1
√
ln
t √1
ln +∞
f0(x) − +
f(x) & %
Với f√1
ln
=
√
ln ln +
2
√
ln −2 =
ln 4+2 ln 2−2 ln 2√ln ln 2.√ln
f0√1
ln
= ln 4−ln ln 2.√ln =
ln 4(2−√ln 4) ln 2.√ln >0
⇒f0(t)>0, ∀t >0 ⇒ f(t)đồng biến
Theo raf √x+ 2=f + x1 ⇔ √x+ = + 1x (∗)
Đặtt =√x+ (t >0) ⇒ x=t2−2.Phương trình (*) trở thành
t = +
t2−2 ⇔t
3−2t2−2t+ = 0
⇔
t=
t= 1+
√
13
t= 1−
√
13 (loại)
Vậy nghiệm phương trình:
x=−1
x= 3+
√
(143)Bài tốn Giải phương trình: x2+x−1 =xex2−1+ (x2−1)ex(∗)
Lời giải
(∗)⇔ −(x2−1)(ex−1) =x(ex2−1−1) (1)
Nếu
x=±1
x= V T = =V P ⇒
x=±1
x= nghiệm phương trình
Khi
x6=
x6=±1 (1)⇔
ex2−1−1
x2−1 = e
x−1 x (2)
Xét hàm số f(x) = et−t1 D=R\ {0}
f0(x) = tet−te2t+1
Xét g(t) =tet−et+ 1
g0(t) = et+tet−et=tet
t −∞ +∞
g0(t) − +
g(t)
+∞
&0% +∞
⇒g0(t)>0, ∀t6= ⇒ f(t)tăng khoảng xác định
x −∞ +∞
f0(x) + +
f(x) %
1% +∞
Do đó(2)⇔f(x2−1) =f(x) ⇔ x2−x−1 = 0 ⇔ x= 1±√5
Bài tốn (Bình Định) Giải phương trình:
−2x3+ 10x2−17x+ = 2x2√35x−x3
Lời giải Dễ thấy x= 0khơng phải nghiệm phương trình Chia hai vế phương trình chox3, ta được
−2 + 10
x −
17
x2 +
x3 =
3 r
5
x2 −1⇔8t
3−17t2+ 10t−2 = 2√3
(144)với t= 1/x (t6= 0)
Ta biến đổi phương trình (1) tiếp tục sau
(1)⇔(2t−1)3+ 2(2t−1) = 5t2−1 + 2√35t2−1
Xét hàm số f(x) = x3+ 2x thì f0 = 3x2+ 2 > 0 nên f là hàm số tăng trên
R Phương
trình cuối viết lại thành
f(2t−1) =f(√35t2−1)
Do f hàm số tăng nên phương trình tương đương với
2t−1 =√35t2 −1⇔8t3−12t2+ 6t−1 = 5t2 −1
Giải ta đượct= (loại),t = 17±
√
97
16 Tương ứng ta tìm đượcx=
17±√97 12
Bài toán (Vĩnh Phúc đề xuất Olympic Đồng Bắc 2010) Giải phương trình
(6x−3x)(19x−5x)(10x−7x) + (15x−8x)(9x−4x)(5x−2x) = 231x
Lời giải Nhận xét1: Với a > b > c >1 (
ax ≥bx nếu x≥0
ax ≤bx nếu x≤0
Nhận xét 2: Hàm số f(x) = ax −bx xác định đồng biến liên tục tập D = [0; +∞) do
f0(x) = axlna−bxlnb >0 ∀a≥0 với a > b >0cho trước
Nhận xét 3: Tích hai hàm số đồng biến, nhận giá trị dương tậpD hàm đồng biến tổng hai hàm số đồng biến trênD hàm số đồng biến
Áp dụng
(+) x ≤ 0: V T = (6x −3x)(19x−5x)(10x −7x) + (15x −8x)(9x −4x)(5x −2x) ≤ 0;
V P = 231x >0 Suy phương trình khơng có nghiệm dương
(+) Vớix >0chia vế phương trình cho 231x = (3.7.11)x ta được
(2x−1)
19 11 x − 11 x 10 x −1 + 15 11 x − 11 x x − x x − x =
Từ nhận xét (2) nhận xét hàm số
g(x) = (2x−1)
19 11 x − 11 x 10 x −1 + 15 11 x − 11 x x − x x − x
là hàm số đồng biến D= (0; +∞) g(1) =
phương trình cho⇔g(x) =g(1) ⇒x= nghiệm phương trình Bài tốn ( Chọn HSG Khánh Hịa 2002) Giải hệ phương trình :
(1 + 42x−y) 51−2x+y = + 22x−y+1 (1)
(145)Lời giải Đặt t= 2x−y Khi hệ (I) :
(1 + 42x−y) 51−2x+y = + 22x−y+1 (1)
y3 + 4x+ + ln(y2+ 2x) = 0 (2)
Ta có (1)⇔(1 + 4t) 5−t+1 = + 2t+1 ⇔5h 15t+ 45ti = + 2.2t
Đặtf(t) =
h
1
t
+ 45t
i
; g(t) = + 2.2t
Ta có: f(t) hàm số giảm, g(t) hàm số tăng f(1) =g(1) Do đó(3)⇔t = 1⇔2x−y=
Vậy hệ (I)⇔
2x=y+
y3+ 2y+ + ln(y2+y+ 1) = 0
Đặth(y) =y3+ 2y+ + ln(y2 +y+ 1)
Ta có h0(y) = 3y2+ + 2y+1
y2+y+1 = 3y2+
2y2+4y+3
y2+y+1 = 3y2+
2(y+1)2+1
y2+y+1 >0
h0(y)>0⇒h(y) hàm số tăng h(−1) = Vậy (I)⇔
2x=y+
y =−1 ⇔
x=
y=−1
Bài toán (Đồng Nai) Giải hệ phương trình
x5+xy4 =y10+y6 (1)
√
4x+ +py2+ = 6 (2)
Lời giải Nếu y = từ phương trình (1) suy x = 0, phương trình (2) khơng thoả mãn Vậyy6= Chia hai vế phương trình (1) choy5, ta đượcx
y
5
+xy =y5+y (3).
Xét hàm số f(x) =x5+x, ta có f0(x) = 5x4+ 1 >0, suy ra f là hàm số tăng trên
R Phương
trình (3) viết lại thànhf(x/y) =f(y)và dof hàm tăng nên tương đương vớix/y =y, suy x = y2 Thay vào phương trình (2), ta được √4x+ +√x+ = 6 (4).) Giải ta
đượcx = nghiệm phương trình (4) Từ hệ ban đầu có nghiệm
(x;y) = (1; 1) (1;−1)
Bài toán (Nguyễn Trãi- Hải Dương đề xuất Olympic Đồng Bắc 2010) Giải hệ phương trình:
(
(x+y)4+ = (x+y)
x4−y4
64 +
9(x2−y2)
32 + 7(x−y)
8 + ln
x−3
y−3
=
Lời giải Theo bđt Cauchy ta có(x+y)4+1+1+1≥4q4
(x+y)4.1.1.1 = 4|x+y| ≥4 (x+y)
Dấu xảy ra⇔x+y= 1(∗)
Từ kết hợp với đk xy−−33 >0⇒ −2< x, y <3
Pt thứ hai hệ⇔ x4
64 + 9x2
32 + 7x
8 + ln (3−x) =
y4
64+ 9y2
32 + 7y
8 + ln (3−y)
Xét hàm số f(x) = x644 +932x2 + 78x + ln (3−x) (với x <3)
f0(x) = x163 +916x + 78 +x−33 = (x
3+9x+14)(x−3)+48
16(x−3) = x4−3x3+9x2−13x+6
16(x−3) =
(x−1)2(x2−x+6)
16(x−3) ≤ Suy hàm số nghịch biến (−2; 3), f(x) =
(146)Bài toán (THPT Chuyên Biên Hòa - Hà Nam đề xuất Olympic Đồng Bắc 2010) Giải hệ phương trình:
(x−y) (x2+xy+y2 −2) = ln
y+
√
y2+9
x+√x2+9
x5y−3xy−1 =
Lời giải Từ (x−y) (x2+xy+y2−2) = ln
y+√y2+9
x+√x2+9
⇔x3−2x+ lnx+√x2+ 9=y3−2y+ lny+p
y2 + 9 (1)
Xétf(t) =t3−2t+ ln t+√t2+ 9
t∈R
f0(t) = 3t2 −2 + √
t2+ 9 =
t2+ √
t2+ 9 −
Ta có
t2+√2
t2+9 −
2 =t
2+ + √2
t2+9 −
29 =
t2+9
27 +
√
t2+9 +
1
√
t2+9 +
26 27(t
2+ 9)− 29
≥1 + 2627(t2+ 9)− 29
3 ≥1 + 26
3 = 29
3 − 29
3 =
Suy raf0(t)≥0 ∀t ⇒hàm số đồng biến liên tục trênR.Mà (1)⇔f(x) =f(y)⇔x=y
Thay vào phương trình cịn lại hệ ta có x6−3x2 −1 = (2). Đặt x2 =u(u ≥0) suy
rau3−3u= (3)
Xét g(u) = u3−3u−1với u≥0.
g0(u) = 3u2−3, cóg0(u) = ⇔u=±1
Căn vào BBT phương trình (3) có nghiệm thuộc(0; 2)
Đặt u= cosα với α∈ 0;π
2
(3) trở thành cos3α=12 ⇔α=π9 ⇒x=±p
2 cosπ9
Vậy hệ có nghiệm p
2 cosπ9;p
2 cosπ9 ; −p
2 cosπ9;−p
2 cosπ9
Bài tốn 10 (Đề Hải phịng đề xuất Olympic Đồng Bắc 2010) Giải hệ phương trình:
x= cos
π
3√3y
y= cos π 3√3z
z = cos π 3√3x
Lời giải Xét hàm số
f(x) = cos
π
3√3x
⇒f0(x) = π 3√3sin
π
3√3x
⇒ |f0(x)|<1
(147)Tương tự ta có |x−y| ≤ |y−z| ≤ |z−x| ≤ |y−x| ⇒ |x−y|=|y−z|=|z−x| Giả sử x= max{x, y, z} ⇒x=y=z
Từ có f(x) = x Xét hàm số:
g(x) = x−cos
π
3√3x
⇒g0(x) = 1− π
3√3sin
π
3√3x
>0
Vậy g(x) đồng biến mà g
√
3
= nên hệ phương trình cho có nghiệm x= y =
z=
√
3
Bài toán 11 (Chuyên Hưng Yên đề xuất Olympic Đồng Bắc 2010) Tìm nghiệm hệ phương trình với x≥0
2x−2y+√2x+y+ 2xy+ =
3
√
3y+ = 8x3−2y−1
Lời giải
2x−2y+√2x+y+ 2xy+ = (1)
3
√
3y+ = 8x3−2y−1 (2)
(1) ⇔(2x+ 1)−2 (y+ 1) +p(2x+ 1) (y+ 1) =
ĐK:(2x+ 1)(y+ 1)≥0 Mà x≥0 nên
2x+ 1>0
y+ ≥0
(1)⇔√2x+ 1−py+ √2x+ + 2py+ 1=
⇔√2x+ 1−py+ =
⇔y= 2x
Thay vào (2): √3
6x+ = 8x3−4x−1
⇔(6x+ 1) +√3 6x+ = (2x)3+ 2x
Hàm số f(t) = t3+t đồng biến trên R
(2) ⇔√3
6x+ = 2x
⇔4x3−3x=
Nhận xét:x >1khơng nghiệm phương trình Xét0≤x≤1.Đặtx=cosα với0≤α≤ π
2
⇒cos 3α =
⇔
α= π9 +k23π α=−π
9 +k 2π
3
(k ∈Z)
Do 0≤α ≤ π
2 ⇒α=
π
9
(148)Bài tốn 12 Giải hệ phương trình sau:
√
x2− 2x + 6.log
3(6 − y) = x
p
y2− 2y + 6.log
3(6 − z) = y
√
z2− 2z + 6.log
3(6 − x) = z
(I)
Lời giải Điều kiện xác định: x, y, z <6
Viết lại hệ cho dạng tương đương
x
√
x2−2x+6 = log3(6−y)
y
√
y2−2y+6 = log3(6−z)
z
√
z2−2z+6 = log3(6−x)
• Xét hàm sốf(u) = √ u
u2−2u+6 g(u) = log 3(6−u)trên (−∞; 6)
Ta có: f0(u) = (u2−6−2uu+6)
√
u2−2u+ 6 > 0, với u ∈ (−∞; 6) và g0(u) = −1
(6−u) ln < 0, với
u∈(−∞; 6)
Suy f(u) đồng biến g(u)nghịch biến (−∞; 6) (1)
• Giả sử(x, y, z)là nghiệm hệ (I) Do tính đối xứng vịng quanh hệ (I)
x, y, z,khơng tổng qt giả sử x= maxx, y, z.Khi đó, từ (I) (1) ta có
g(y) =f(x) = maxf(x), f(y), f(z) = maxg(x), g(y), g(z)
⇒y=minx, y, z ⇒g(z) = f(y) = minf(x), f(y), f(z) = ming(x), g(y), g(z)
⇒z =maxx, y, z ⇒z =x⇒f(z) = f(x)⇒g(x) =g(y)⇒x=y⇒x=y =z
Như vậy, hệ (I) có nghiệm dạng x=y=z
• Với x=y=z, từ hệ (I) ta có hệ
f(x) = g(x)
f(y) = g(y)
f(z) = g(z)
(II)
Xét phương trình f(u) =g(u) (2)
Do (1) nên phương trình (2) có tối đa nghiệm trên(˘∞; 6) Hơn nữa, cách thử trực tiếp dễ thấyu = nghiệm (2) Suy pt (2) có nghiệm nhấtu = Do hệ (II) có nghiệm x=y=z = Vậy, hệ cho có nghiệm x=y=z =
Bài toán 13 Cho hệ
m(x2 +√3
x4+√3
x2+ 1) = yx
m(√3x8+x2+√3
x2+ 1) + (m−1)√3
x4 = 2y√3
x4
(149)Lời giải Nếu m=
yx =
2y√3 x4 =−√3
x4 ⇔(x;y) = (0, c)∀c∈R
m6= đặt t =√3 x ta có hệ
m(t6+t4 +t2 + 1) =yt3
m(t8+t6 +t4 +t2+ 1) = (2y+ 1)t4
Nhận xét t=0 nghiệm hệ Đặtu=t+
t (|u| ≥2)
m(t3+t+
t +
1
t3) =y
m(t4+t2+ +
t2 +
1
t4) = 2y+
⇔
m(u3−2u) = y
m(u4−3u2+ 1) = 2y+ 1
⇔
m(u3−2u) = y
m(u4−3u2+ 1) = 2m(u3−2u) + 1(∗)
Phương trình (*) ⇔m(u4−2u3−3u2+ 4u+ 1) =
⇔u4−2u3−3u2+ 4u+ =
m
Xét hàm số f(u) =u
4−2u3−3u2+ 4u+ 1 (|u| ≥2)
f0(u) = 4u3−6u2−6u+ 4
Vậy hệ có nghiệm⇔
m ≥ −3⇔
m ≤ −1
m >0
Bài toán 14 Giải hệ
ex =ey−x+ 1
ex =ex−y+ (I)
Lời giải
(I)⇔
ex =ey−x+ 1
ex−ey =e(y−x) + (y−x) ⇔
ex =ey−x+ 1 (1)
ex−ey = (y−x)(e+ 1) (2)
Nếux > y ex−ey >0>(y−x)(e+ 1)
Nếux < y ex−ey <0<(y−x)(e+ 1)
Nên từ (2) ⇔ x=y, thay vào (1):
ex−ex+x−1 =
Xét f(x) = ex−ex+x D =
R
f0(x) =ex−e+
f0(x) = ⇔ ex =e−1 ⇔x= ln(e−1)
x −∞ ln(e−1) +∞
f0(x) − +
(150)Do phương trìnhf(x) = có nhiều hai nghiệm, ngồi f(0) = 0, f(1) =
Vậy hệ có hai nghiệm
x=
y =
x=
y=
2.3 Dạng 3: Bất phương trình, hệ bất phương trình
Bài tốn ( Phan Huy Khải) Tìm m để hệ bất phương trình sau có nghiệm dương
x(3−y2)> m
y(3−z2)> m z(3−x2)> m
Lời giải Xét hàm số f(t) = 3t−t3 với t≥0⇒f0(t) = 3−3t2
x +∞
f0(x) + −
f(x) 0%
&−∞
Từ suy m < bpt: t(3−t2) > m có nghiệm t > 0 Do hệ cho có nghiệm
x=y=z =t0 với t0 nghiệm tùy ý bpt : 3t−t3 > m
Xét khim≥2 , giả sử hệ có nghiệm (a;b;c) :
a >0;b >0;c >0 : a(3−b2)> m;b(3−c2)> m;c(3−a2)> m (1)
suy : 3−a2 >0; 3−b2 >0; 3−c2 >0 Nên: 0< a;b;c <√3
Nhân vế theo vế bất đẳng thức dương chiều (1) Ta :a(3−a2)b(3−b2)c(3−c2)>
m3 (2)
Mặt khác vì:0< a;b;c <√3 , nên : 0< f(a)≤2; < f(b)≤2; 0< f(c)≤2
suy : 0< f(a).f(b).f(c)≤23 ≤m3 ( 3) Từ (2) (3) suy vơ lý Vậy hệ có nghiệm khim <2
Bài toán Cho α, β, γ ∈[0;π2] Chứng minh rằng: 2cosαsinα+ 2cosαcosβ+ 2sinα ≥4 .
Lời giải Xét hàm số f(x) = 2x−x−1 ∀x∈[0; 1]
Ta có: f0(x) = 2xln 2−1là hàm liên tục trên
R
f00(x) = 2x(ln 2)2 > ∀x ∈ R nên f0(x) hàm tăng R Suy tồn x0 để
f0(x0) = Ta có bảng biến thiên sau:
x x0
f0(x) + −
f(x) 0%
CĐ
(151)⇒f(x)≥0, ∀x∈[0; 1] hay 2x≥x+ 1, ∀x∈[0; 1]
Để ý khiα, β, γ ∈[0;π2] cosαsinβ,cosαcosβ,sinα∈[0; 1]
Áp dụng bổ đề suy
2cosαsinα+ 2cosαcosβ + 2sinα ≥cosαsinβ+ cosαcosβ+ sinα+
≥cos2αsin2β+ cos2αcos2β+ =
Dấu xảy chẳng hạn: cosα = 0,sinα= tức α= π2
Bài toán ( Đề chọn HSG Quốc Học Huế -2005) Giải bất phương trình: ex+ (x3−x) ln(x2+ 1)≤3√3x
(∗)
Lời giải Biểu thức ln(x2 + 1) xác định
x= 0;x= 1;x=−1là giá trị thỏa mãn bất phương trình Ta có : x3−x= (x−√3x)(x2+x√3x+√3x2).
Khi x /∈ {0; 1;−1} x6= √3x Theo định lý Lagrange tồn số cnằm giữa x và √3x sao
cho:ex−e√3x
= (x−√3x)ec
Vậy
(∗)⇒(x−√3
x)[ec+ (x2+x√3
x+
√
x2)] ln(x2+ 1)≤0
⇔x−√3
x≤0 (Vì[ec+ (x2+x√3
x+√3 x2)] ln(x2+ 1)>0)
⇔x3−x≤0
Nghiệm bất phương trình cho :x∈(−∞;−1]∪[0; 1]
Bài tốn Cho 16=a >0, chứng minh : aln−a1 ≤ 1+3 √
a a+√3a Lời giải aln−a1 ≤ 1+3
√
a
a+√3a (1) với 16=a >0
Trường hợp 1:a >1
(1)⇔(a+√3a) lna≤(1 +√3a)(a−1) (2) Đặt x=√3 a⇒x >1
(2)⇔3(x3 +x) lnx≤(1 +x)(x3−1) ∀x >1
⇔x4 +x3−x−1−3(x3+x) lnx≥0 (3) ∀x >1
Đặtf(x) =x4+x3−x−1−3(x3+x) lnx x∈[1; +∞)
Ta có f0(x) = 4x3+ 3x2−1−3[(3x2+ 1) lnx+ (x3+x)1
x] = 4x
3−4−3(3x2+ 1) lnx
f00(x) = 3(4x2−3x−6xlnx−
x); f
(3)(x) = 3(8x+
x2 −6 lnx−9)
f(4)(x) = 3(8−
x −
2
x3) =
6(4x3−3x−1)
x3 =
6(x−1)(4x2+4x+1)
x3 ≥0, ∀x≥1
Suy f(3)(x) đồng biến trên [1; +∞)
f(3)(x)≥f(3)(1) = tương tự f0(x)≥0 ∀x≥1
⇒f(x)> f(1) = ∀x >1suy (3) Trường hợp 2:0< a <1, đặt a= a1
1, a1 >1 quay trường hợp
(152)Lời giải
kn>2nk ⇔nlnk >ln +klnn
⇔nlnk−klnn >ln 2(∗)
Xét f(x) =nlnx−xlnn Trên [1; +∞)
f0(x) = nx −lnn;f0(x)>0⇔x < lnnn
x lnnn +∞
f0(x) + −
f(x) % &
Do đk2≤k≤n−1nên khif(x)xét[2;n−1]thì giá trị nhỏ đạt f(2)hay f(n−1)
Ta chứng minh
f(2)>ln
f(n−1)>ln
f(2) >ln 2⇔nln 2−2 lnn >ln 2⇔(n−1) ln 2>2 lnn
2n−1 > n2 (bằng quy nạp với n≥7 đúng)
f(n−1)>ln 2⇔nln(n−1)−(n−1) lnn > ln
⇔nln(n−1)>(n−1) lnn+ ln (n−1)n >2.nn−1(2)
Đặtn−1 =t (t ≥6)Thì (2) trở thành: t
t+1 ≥2(t+ 1)t (3)
⇔t≥2 t+1t t= + 1tt
Ta có: + 1tt<3 , ∀t
Do + 1tt
<6≤t với t ≥6Vậy (3) ⇒(2)đúng ⇒ (1) chứng minh
3 Bài tập đề nghị
Bài toán ( VMO 2007) Cho số thực a > Đặt fn(x) = a10xn+10 +xn +· · ·+x + 1,
(n = 1,2, ) Chứng minh với n phương trìnhfn(x) =a có nghiệm Chứng
minh dãy số (xn) có giới hạn hữu hạn n → ∞
Bài toán (Hà Tĩnh 2009) Cho dãy {xn} biết x1 =−12, xn+1 = x
2
n−1
2 với n = 1,2,3,
Tìm giới hạn dãy {xn} n dần tới vơ
Bài tốn (Bà Rịa Vũng Tàu 2009) Cho dãy số xác định x1 = 1, xn+1 = 2(x21
n+1) −2008
Chứng minh {xn} có giới hạn hữu hạn n dần đến vơ
Bài tốn 4.(PTNK 1999) Choa >1và dãy số{xn}được xác định sau:x1 =a, xn+1 =axn
(153)Bài toán (Bắc Ninh 2009) Cho dãy số {xn} xác định
x1 = π2
xn+1 = π+cos 24 xn ∀n∈N∗
Bài toán Chứng minh với số nguyên dươngncho trước, phương trìnhx2n+1 =x+1
có nghiệm thực Gọi nghiêm xn Tìm lim n→+∞xn Bài tốn Cho phương trình: x13−x6+ 3x4−3x2+ = 0.
1) Chứng minh phương trình có nghiệm thực 2) Đặt x1 = xn+1 =
x−n7/3+
−3/13
với số nguyên dương n Chứng minh dãy số {xn} có giới hạn đặt x0 =− lim
n→+∞xn x0 nghiệm nói Bài tốn Cho tam giác ABC khơng có góc tù thoả mãn hệ thức:
1
3(cos3A+cos3B)−
2(cos2A+cos2B) +cosA+cosB=
Hãy tính góc tam giác
Bài tốn Giải phương trình: 2x2−6x+ = log
h
2x+1 (x_1)2
i
Bài toán 10 Giải phương trình: 22x + 32x = 2x+ 3x+1+x+ 1
Bài tốn 11 Giải phương trình 2x− x
2 −1 =
Bài tốn 12 Giải phương trình 3x− 2x
3 −1 =
Bài toán 13 Giải phương trình log
√
2+√3(x
2−2x−2) = log
2+√3(x
2−2x−3)
Bài toán 14 Tìm a để phương trình có nghiệm
a
r
x−3
x +
√
x−1 +√x−3−√x−√x+ =
Bài toán 15 Giải hệ
(4x2+ 1)x+ (y−3)√5−2y= 0 4x2+y2+ 2√3−4x=
Bài toán 16 Giải hệ
2x+ ln(x2 +x+ 1) =y 2y+ ln(y2+y+ 1) =z
2z+ ln(z2 +z+ 1) =x
Bài toán 17 Giải hệ
x2+ 3x+ + ln(x2+x+ 1) =y
(154)Tài liệu tham khảo
[1] Các đề thi HSGQG (VMO)
[2] Các đề thi Toán quốc tế nước
[3] Phan Huy Khải, Dãy số giới hạn, Nhà xuất Hà Nội, (1996)
[4] GS-TSKH Nguyễn Văn Mậu, Nguyễn Thủy Thanh Dãy số giới hạn, Nhà xuất Giáo dục, (2002)
[5] GS-TSKH Nguyễn Văn Mậu,Phương pháp giải phương trình bất phương trình, Nhà xuất Giáo dục, (1996)
[6] TS Trần Nam Dũng, Tập dãy số đề thi Olympic 30/4 Các Tỉnh phía Nam
[7] Các đề thi Olympic Các Tỉnh Duyên Hải Bắc [8] Báo Toán học Tuổi trẻ
(155)ỨNG DỤNG MỘT SỐ ĐỊNH LÍ CƠ BẢN CỦA GIẢI TÍCH
Huỳnh Tấn Châu, Trường THPT Chuyên Lương Văn Chánh - Phú Yên
Trong chương trình tốn học bậc THPT định lí giải tích lớp hàm số liên tục khả vi có nhiều ứng dụng thú vị, chưa đề cập nhiều sách tham khảo cho học sinh chuyên toán Trong số trường hợp dùng định lí để giải hổ trợ cho tốn phương trình, chứng minh bất đẳng thức, toán liên quan đến dãy số, tỏ hiệu thú vị so với cách giải khác Còn nhiều ứng dụng khác định lí trên, mong trao đổi từ phía Thầy giáo em học sinh u thích mơn tốn
1 Một số định lí tính liên tục hàm số
Định lý Nếu hàm số f liên tục đoạn [a;b] hàm số f bị chặn đoạn [a;b], tức tồn sốM > cho : |f(x)| ≤M, ∀x∈[a, b]
Định lý Nếu hàm số f liên tục đoạn [a;b] f đạt giá trị lớn đạt giá trị nhỏ đoạn đó, tức tồn điểmx1, x2 ∈[a;b] cho :
f(x1)≤f(x)≤f(x2), ∀x∈[a, b]
Định lý (Định lí giá trị trung gian hàm số liên tục) Giả sử hàm số f liên tục đoạn [a;b] f(a) = A, f(b) = B Khi C số nằm A B có điểm c∈(a;b) để f(c) = C
Hệ Nếu hàm số f liên tục đoạn[a;b]thì f nhận giá trị trung gian giá trị nhỏ m giá trị lớn M đoạn
Định lý (ĐỊNH LÍ BOLZANO – CAUCHY ) Nếu hàm số f liên tục đoạn [a;b]
f(a).f(b)<0 có điểm c∈(a;b) để f(c) =
2 Một số định lí phép tính vi phân
Định lý (Định lí Lagrange ) Nếu hàm số f(x) liên tục đoạn [a;b], khả vi khoảng
(a;b) Khi tồn điểm c∈(a;b) cho f(b)−f(a) = (b−a)f0(c)
Hệ Nếu hàm số f(x) liên tục đoạn [a;b] f0(x) = 0∀x ∈[a;b] f(x) số trên[a;b]
Định lý (Định lí Rolle) Nếu hàm số f(x) liên tục đoạn [a;b], khả vi khoảng (a;b)
và f(a) = f(b) tồn điểm c∈(a;b) cho f0(c) =
(156)3 Một số toán áp dụng
3.1 Các toán phương trình
Bài tốn Cho hàm số f(x) liên tục đoạn [a;b] x1, x2, , xn n giá trị thuộc
[a;b] Chứng minh tồn số ξ ∈[a;b] cho : f(ξ) =
n[f(x1) +f(x2) + +f(xn)]
Lời giải Vì hàm sốf(x)liên tục đoạn[a;b]nên tồn GTNN m GTLN M
[a;b]
⇒m ≤
n[f(x1) +f(x2) + +f(xn)]≤M
Do theo định lí giá trị trung gian tồn số ξ∈[a;b] cho :
f(ξ) =
n[f(x1) +f(x2) + +f(xn)]
Bài toán Cho hàm số f(x) liên tục đoạn [0; 1] thoả mãn điều kiện f(0) =f(1) Chứng minh phương trình : f(x) = f x+ 20121
có nghiệm thuộc đoạn [0; 1]
Lời giải Xét hàm số g(x) = f x+ 2012
−f(x)
Hàm số xác định liên tục 0;20112012
Ta có g(0) =f 20121 −f(0)
g 2012 =f 2012 −f 2012
;· · · ;g
2011 2012
=f(1)−f
2011 2012
Vậy g(0) +g 20121 + +g 20112012 =f(1)−f(0) =
Suy tồn tạii, j ∈N i, j ≤2011 cho g 2012i ≤0 g 2012j ≥0
Vì g hàm số liên tục 0;20112012, nên theo định lí Bolzano – Cauchy phương trình
g(x) = f x+20121
−f(x) = có nghiệm
0;20112012
do phương trình :f(x) =f x+20121 có nghiệm thuộc đoạn[0; 1]
Bài tốn Tìm cặp số thực (b;c)sao cho với số thực a phương trìnhacos 2x+
bcosx+c= có nghiệm thực thuộc khoảng 0;π2
Lời giải Đặt f(x) = acos 2x+bcosx+c= 2acos2x+bcosx+c˘a
Điều kiện cần : Giả sử (b, c) cặp số thực thỏa mãn yêu cầu toán - Nếu b= 0, ta lấya= 0, suy c=
- Nếu b6= 0, lấy a=− b
2√2 ⇒f(x) = ⇔ −
√
2
cosx− √1
2
+√1
2 +
c b =
⇒ √1 +
c
b ≥0⇒ c b ≥ −
1
√
2
Lấya=−b
2 ⇒f(x) = 0⇔cosx(1−cosx) + +
c b =
⇒ +
c
b =−cosx(1−cosx)<0, ∀x∈ 0; π
2
⇒ c
b <−
1
Như ta có : hoặcb =c= 0, b 6= −√1 ≤
c b <−
1 2(1)
(157)- Nếu b=c= ta có :f π4 = 0, ∀a∈R - Nếu b6= và−√1
2 ≤
c b <−
1
2, ta có bf
π
4
=b2√1 +
c b
≥0 (2)
Với ∀x∈ 0;π2 ta có :
b
f(x) +f π
2 −x
= 2b2c
b +
1
2(sinx + cosx)
→2b2 c b +
1
khix→0, tồnx0 ∈ 0;π2
mà bf(x0)<0 (3)
Từ (2) (3), theo định lí Bolzano – Cauchy hàm số liên tục ta suy tồn nghiệm
x∈ 0;π2 mà f(x) =
Vậy cặp số (b, c) thỏa mãn (1) cặp số cần tìm
Bài toán (VMO – 2006) Cho hàm sốf(x) = −x+p(x+a) (x+b), a, b hai số thực dương cho trước Chứng minh với số thực s thuộc khoảng (0; 1) tồn số thực dươngα cho : f(α) = as+2bs
1
s.
Lời giải Ta có f(x)là hàm số liên tục [0; +∞) Ta chứng minh khẳng định sau : 1.f(x) tăng thực [0; +∞)
Với a6=b 0x≥ta có : f0(x) =−1 + 2x+a+b 2√(x+a)(x+b) =
(√x+a−√x+b)2
2√(x+a)(x+b) >0
Do đóf(x) hàm tăng [0; +∞)
2.f(0) =√ab; lim
x→+∞f(x) =
a+b
2
Mặt khác lim
x→+∞f(x) = limx→+∞
−x2+(x+a)(x+b)
x+√(x+a)(x+b) = limx→+∞
a+b+abx 1+
q
(a x+1)(
b x+1)
= a+2b
3 Với mọi0< s <1, ta có : √ab < as+2bs
1
s ≤ a+b
2
BĐT bên trái theo BĐT AM – GM Đặtm = as+2bs
1
s, x= a m, y =
b m x
5+y5 = 1.
Theo BĐT Bernoulli ta có :
x= (1 +xs−1)1s ≥1 + x s−1
s ;y= (1 +y
s−1)1s ≥1 + y s−1
s ;
(không đồng thời xảy đẳng thức)
Cộng theo vế ta x+y >2, suy BĐT bên phải Từ kết suy ra∃!α∈(0; 1) cho :
f(α) =
as+bs
2
1s
Bài toán (OLYMPIC SINH VIÊN VIỆT NAM – 1994) Cho n số nguyên dương, ak, bk∈
R(k = 1,2, , n).Chứng minh phương trình x+
n
P
k=1
(aksinkx+bkcoskx) = có nghiệm
trong khoảng(−π;π)
Lời giải Xét hàm F (x) = x22 +
n
P
k=1
−ak
k coskx+ bk
k sinkx
, x∈ mathbbR
(158)XétF0(x) =x+
n
P
k=1
(aksinkx+bkcoskx) =
Khi đóF (−π) = π22 +
n
P
k=1
−ak
k(−1) k
;F (π) = π22 +
n
P
k=1
−ak
k(−1) k
DoF (−π) =F (π).Sử dụng định lí Rolle khoảng(−π;π),ta nhận điều phải chứng minh
Bài toán Cho a0, a1, , an số thực thỏa mãn điều kiện
a0+
a1 +
a2
3 + +
an
n+ =a0+a1+
a2.22 +
a3.23
4 + +
an.2n
n+ =
Chứng minh phương trình : a1+ 2a2x+ 3a3x2+ +nanxn−1 = có nghiệm
thuộc khoảng(0; 2)
Lời giải Xét hàm số f(x) =a0x+ 12a1x2+ 13a2x3+ +n+11 anxn+1
Ta có f(1) =a0+a21 +a32 + + nan+1
f(2) = 2a0+a1+a2.2
2
3 +
a3.23
4 + +
an.2n n+1
Theo giả thiết ta có f(1) = f(2) = Hiển nhiên f(0) Theo định lí Rolle, tồn số
c1, c2 : 0< c1 <1< c2 <2 cho :
f0(c1) =f0(c2) =
Áp dụng định lí Rolle với hàm số f0(x) [c1;c2], ta thấy tồn x0 ∈ (c1;c2) cho :
f00(x0) =
Do x0 ∈ (c1;c2) ⊂ (0; 2) nghiệm phương trình f00(x) = a1 + 2a2x+ 3a3x2 + +
nanxn−1 = (đpcm)
Bài tốn Giải phương trình : 3x = +x+ log3(1 + 2x) (1)
Lời giải ĐK: + 2x >0⇔x >−1
(1) ⇔x+ 3x = (1 + 2x) + log3(1 + 2x)
⇔3x+ log33x = (1 + 2x) + log3(1 + 2x(2)
Đặtϕ(t) = t+ log3t, t >0
] Ta có ϕ(t) hàm đồng biến trên(0; +∞) (2)⇔ϕ(3x) = ϕ(1 + 2x)
⇔3x = + 2x⇔3x−2x−1 =
Xét hàm số f(x) = 3x−2x˘1, x > −1
f0(x) = 3xln 3˘2
Ta có f0(x) hàm đồng biến, nên theo định lý Rolle phương trình f(x) = có khơng q nghiệm Rõ ràngf(0) =f(1) =
(159)Bài tốn Giải phương trình : (1 + cosx) (2 + 4cosx) = 3.4cosx
Lời giải Đặt y= cosx,−1≤y≤1
Ta có phương trình:(1 +y) (2 + 4y) = 3.4y ⇔ 3.4y
2+4y −y−1 =
Đặtf(y) = 2+43.4yy −y−1 Ta có f
0(y) = ln 4.4y
(2+4y)2˘1
f0(y) = ⇔6 ln 4.4y = (2 + 4y)2
Đây phương trình bậc hai theo 4y nên có khơng q nghiệm.
Vậy theo định lí Rolle phương trìnhf(y) = có khơng q nghiệm Mặt khác ta thấyy= 0;y = 12;y= nghiệm phương trình f(y) =
Vậy phương trình cho có nghiệm tương ứngx=k.2π;x= π2 +k.π;x=±π
3 +k.2π(k ∈)
Bài toán Cho a, b, c ∈ R n ∈ N∗ thỏa mãn điều kiện c = −6(a+b)
5(n+2) Chứng minh
phương trình sau có nghiệm khoảng 0;π2:
asinnx+bcosnx+csinx+c=
Lời giải Đặt f(x) = n2+2a sinn+2x− 2b n+2cos
n+2x+ 2c
3sin
3x−ccos2x
Ta cóf(x)xác định, liên tục có đạo hàm trênR f0(x) = 2asinn+1xcosx+ 2bcosn+1xsinx+ 2csin2xcosx+ 2ccosxsincx
f0(x) = sinxcosx(asinnx+bcosnx+csinx+c)
Ta có f(0) =− 2b n+2 +
6(a+b) 5(n+2) =
6a−4b
5(n+2)
f π2= n2+2a −
6(a+b) 5(n+2) =
2a n+2 −
4(a+b) 5(n+2) =
6a−4b
5(n+2)
⇒f(0) =f π2, nên theo định lý Rolle ∃x0 ∈ 0;π2
:f0(x0) =
Hay∃x0 ∈ 0;π2
:f0(x0) = sinx0cosx0(asinnx0+bcosnx0+csinx0+c) =
∃x0 ∈ 0;π2
:asinnx0+bcosnx0+csinx0+c=
Do phương trìnhasinx+bcosnx+csinx+c= 0 có nghiệm khoảng 0;π
2
Bài toán 10 (VIỆT NAM TST – 1997) Hãy xác định tất cặp số thực dương a, b cho với n∈N∗ và với nghiệm thựcx
n phương trình 4n2x= log2(2n2x+ 1) ta ln
có:
axn+bxn ≥2 + 3xn
Lời giải 4n2x= log
2(2n2x+ 1) Đặt y= 2n2x,
PT trở thành:2y= log2(y+ 1) ⇔4y =y+⇔f(y) = 4y−y−1 = 01
Ta có f0(y) = 4yln 4−1, f(y) = 4y(ln 4)2 >0
Do phương trìnhf(y) = có khơng q nghiệm phân biệt
Thật vậy, phương trìnhf(y) = 0có nghiệm phân biệt theo định lí Rolle, phương trình
f0(y) = có nghiệm phân biệt phương trình f”(y) = có nghiệm, mâu thuẫn Ta cóf(0) =f −1
2
= 0nên
y =
y =−1
⇒xn= 0∨xn=−4n12 • Nếuxn = 0thì ∀a, b >0 ta
cóa0+b0 ≥2 + 3.0 là đúng.
•Nếu xn=−4n12 Ta cần tìm a, b >0 đểa
− 4n2 +b−
1
4n2 ≥2−
4n2,∀n ∈N
(160)Ta có : f0(x) = 3−a−xlna−b−xlnb, f(0) =
Hàmφ(x) = t−xlnt cóφ0(x) = −t−x(lnt)2 <0 nên φ(x) giảm (0; +∞)
⇒f0(x) tăng [0,+∞)
- Nếu f0(0)<0, f(x) liên tục nên∃ε >0 cho f0(x)<0,∀x∈[0, ε)
Suy f(x) giảm [0, ε)
Luôn ∃n∈N∗ để
4n2 ∈[0, ε) nên f
1 4n2
< f(0) = 2và khẳng định (*) không thỏa mãn - Xét f0(0) ≥ Khi f0(0) ≥ 0,∀x ∈ [0; +∞) Suy f(x) tăng [0; +∞) Do
f 4n12
≥f(0) = (*) thỏa mãn
Vậy f0(0) ≥0là điều kiện cần đủ để ta có (*) Do đó3≥lna+ lnb= lnab⇔ab≤e3
3.2 Chứng minh bất đẳng thức
Bài toán Cho a < b < c Chứng minh :
3a < a+b+c−√a2+b2 +c2−ab−bc−ca < a+b+c+√a2+b2+c2−ab−bc−ca
Lời giải Xét hàm số f(x) = (x−a) (x−b) (x−c)
Hàm số thỏa mãn điều kiện định lí Lagrange đoạn[a;c]
Ta cóf(a) =f(b) =f(c) = Theo định lí Lagrange tồn x1;x2 : a < x1 < b < x2 < c
cho :f(b)−f(a) = (b−a)f0(x1)và f(c)−f(b) = (c−b)f0(x2)
⇒f0(x1) = f0(x2) =
Mặt khác f0(x) = 3x2−2 (a+b+c)x+ab+bc+ca
Vìf0(x1) = f0(x2) = nên suy x1, x2(x1 < x2) hai nghiệm phương trình f0(x) =
x1 =
a+b+c−√a2+b2+c2−ab−bc−ca ;x2 =
a+b+c+√a2+b2+c2−ab−bc−ca
Từ a < x1 < b < x2 < c suy đpcm
Bài toán Chứng minh với số thực dương a, b, c, d ta có :
3 r
abc+abd+acd+bcd
4 ≤
r
ab+ac+ad+bc+bd+cd
6
Lời giải Do a, b, c, d có vai trị nhau, khơng tính tổng qt giả sử a≤b ≤c≤d
Xét hàm số f(x) = (x−a) (x−b) (x−c) (x−d)
Hàm số thỏa mãn điều kiện định lí Lagrange [a;b],[b, c],[c, d]
Ta có f(a) = f(b) =f(c) = f(d) = Theo định lí Lagrange tồn x1, x2, x3 :
a≤x1 ≤b≤x2 ≤c≤x3 ≤d cho f0(x1) = f0(x2) = f0(x3) =
Do đóf0(x) = (x−x1) (x−x2) (x−x3)
Số hạng không chứa x f0(x) −4x1x2x3
Hệ số số hạng chứa x củaf(x)là : −(abc+abd+acd+bcd)
Suy :−4x1x2x3 =−(abc+abd+acd+bcd)⇔x1x2x3 = abc+abd+4acd+bcd
(161)] Hệ số số hạng chứa x2 f(x) : ab+ac+ad+bc+bd+cd
Suy (x1x2+x2x3+x3x1) = (ab+ac+ad+bc+bd+cd)
⇒x1x2+x2x3+x3x1 = ab+ac+ad+2bc+bd+cd
Theo bất đẳng thức AM – GM : x1x2+x2x3+x3x1
3 ≥
3 q
(x1x2x3)
ab+ac+ad+bc+bd+cd
6 ≥
3 s
abc+abd+acd+bcd
4
2
Hay q
abc+abd+acd+bcd
4 ≤
q
ab+ac+ad+bc+bd+cd
6 (đpcm)
Đẳng thức xảy khia =b =c=d
Bài toán Cho t >0 Chứng minh : + t+11 t+1 > + 1tt
Lời giải Xét hàm số f(x) =xln + x1
=x[ln (x+ 1)−lnx]với x >0
Ta có f0(x) = ln (x+ 1)−lnx+x x+11 −
x
= ln (x+ 1)−lnx−
x+1(1)
Xét hàm sốg(y) = lnytrên đoạn[x;x+ 1] Theo định lí Lagrange tồn tạic:x < c < x+ 1sao cho :
g(x+ 1)−g(x) = (x+ 1−x)g0(c)⇒ln (x+ 1)−lnx= 1c > x+11
⇒ln (x+ 1)−lnx−
x+1 >0 (2)
Từ (1) (2) suy f0(x)>0, ∀x >0 Vậy f(x) hàm số đồng biến (0; +∞)
Như vớit >0 ta có f(t+ 1)> f (t)⇒(t+ 1) ln + t+11 > tln + 1t
⇒ln + t+11 t+1 >ln + 1tt⇒ + t+11 t+1 > + 1tt (đpcm)
Bài toán Giả sử S1 = 4n2 P
k=1
k12 S2 =
n
P
k=1
k13 Với n nguyên dương ta có S1 < S2 Lời giải Xét hàm số f(x) =x12 (x≥1)
Theo định lí Lagrange đoạn :[n;n+ 1] tồn số c∈(n;n+ 1) ta có :
f(n+ 1)−f(n) = f0(c) = 2c
−1 <
2n
−1
⇒n−12 >2 h
(n+ 1)12 −n12 i
Cho n nhận giá trị từ1,2, ,4n2, cộng lại ta nhận :
S1 >2
−1 +√4n2+ 1>4n−2
Xét hàm số g(x) =x23 (x≥1)
Theo định lí Lagrange đoạn :[n;n+ 1] tồn số c∈(n;n+ 1) ta có :
g(n+ 1)−g(n) = g0(c) = 23c−13 >
3(n+ 1)
−1
⇒2(n+ 1)−13 <3 h
(n+ 1)23 −n23 i
Cho n nhận giá trị từ0,2, , n˘1, cộng lại ta nhận :
2S2 <3n
2
(162)Suy 2S2 <2S1,∀n ∈N∗ Do không tồn số n nguyên dương thỏa mãn toán
3.3 Các toán dãy số
Bài tốn Xét phương trình : xn−x2−x−1 = (n >2)
1 Chứng minh với số ngun n >2 phương trình có nghiệm dương Tìm lim
n→∞n(xn−1), xn nghiệm dương phương trình
Lời giải Xét hàm f(x) =xn−x2−x−1 = (n >2).
Ta có f(1) = −2 < 0;f(2) = 2n−7 > 0, nên theo định lí Bolzano – Cauchy suy phương
trìnhf(x) = có nghiệm thuộc (1; 2)
Rõ ràng x nghiệm dương phương trình x >1 (do xn =x2 =x+ 1>1).
Với x >1 f0(x) = nxn−1−2x−1>0, nên theo định lý Rolle phương trìnhf(x) = có nhiều nghiệm Suy điều phải chứng minh Hơn ta cóxn ∈(0; 2)
2 Trước hết ta chứng minh lim
n→∞xn= Thật vậy1< xn= n
p
x2
n+xn+ 1≤ x
2
n+xn+n
n <1 +
5
n] (BĐT AM – GM)
Suy lim
n→∞xn= (1) Ta có xn
n=x2n+xn+ ⇒n=
ln(x2n+xn+1)
lnxn
⇒n(xn−1) =
(xn−1) lnxn ln (x
2
n+xn+ 1) (2)
Ta chứng minh : lim
n→∞
xn−1
lnxn = (3)
Thật đặtxn−1 =yn
Ta lim
n→∞
lnxn
xn−1 = limn→∞(1 +yn)
yn
= lne= (do lim
n→∞yn = 0) Từ (1), (2) (3) suy : lim
n→∞n(xn−1) = ln
Bài tốn (VMO – 2002) Xét phương trình : x−11 + 4x1−1 + + k2x1−1 + +
1
n2x−1 =
1 2] (1)
trong n tham số nguyên dương
1 Chứng minh với số nguyên dương n, phương trình nêu có nghiệm lớn Kí hiệu nghiệm xn Chứng minh dãy số {xn} có giới hạn
n→+∞
Lời giải (1)⇔ −1 +
1
x−1 +
4x−1 + +
n2x−1 = (2)
Đặtfn(x) = −12 +x−11 + 4x1−1 + + n2x1−1
Với mỗin∈N∗, hàm sốf
n(x)liên tục nghịch biến khoảng(1; +∞)và lim
x→1+fn(x) = +∞,
lim
x→+∞fn(x) =−
1
2, nên theo định lí Bolzano – Cauchy ∃!xn >1 :fn(xn) =
Vậy phương trình (1) có nghiệm lớn Với mỗin ∈N∗, ta có :
f(4) =−1
2 + 22−1 +
1
42−1+ + (2n)2−1 =
2
−1 + 1−
3+ 3−
1
5 + + 2n−1−
1 2n+
= −1
(163)Do hàm f(x) nghịch biến (1; +∞) nên suy raxn<4,∀n∈N∗ (3)
Do với mỗin∈N∗, hàm f
n(x)khả vi trên[xn; 4], nên theo định lí Lagrange với mỗin ∈N∗ tồn
tại t∈(xn; 4): fn(4)
−fn(xn) 4−xn =f
/
n(t) = (t−−11)2 +
−4
(4t−1)2 + +
−n2
(n2t−1)2 <−
1
9, ∀n∈N
∗
⇒ −1
2(2n+1)(4−xn) <−
9, ∀n∈N
∗ ⇒x
n>4−2(2n9+1), ∀n ∈N∗ (4)
Từ (3) (4) ta được4−
2(2n+1) < xn<4
Do theo định lí giới hạn dãy số kẹp hai dãy số, ta có lim
n→∞xn =
Bài toán Cho dãy số thực (xn) với n = 1,2, , thỏa mãn ln (1 +x2n) +nxn = 1] với
số nguyên dương n Tìm lim
n→+∞
n(1−nxn)
xn
Lời giải Với n ∈N∗ ta đặtf
n(x) = ln (1 +x2) +nx−1, x∈R
Ta có fn0(x) = 1+2xx2 +n =
(x+1)2
1+x2 +n−1≥0
fn0(x) = 0⇔n= 1, x=−1] Do hàm sốfn(x)] hàm số tăng thực
Chú ýfn(0) =−1<0;fn n1
= ln + n12
>0, nên theo định lí Bolzano – Cauchy, suy có số xn∈R thỏa mãn fn(xn) = 0< xn< 1n Bởi
lim
n→+∞
n(1−nxn)
xn
= lim
n→+∞
nln (1 +x2
n)
xn
= lim
n→+∞
nxn.ln +x2n
x12
n
=
Do lim
x→0ln (1 +x
2)x12 = nx
n= 1−ln (1 +x2n)→1khi n →+∞,
vì xn→0 khin →+∞ Chú ý
n(1+nxn)
xn =
n xn +n
2 →+∞ khi n→+∞
Kết luận : lim
n→+∞
n(1−nxn)
xn =
4 Bài tập áp dụng
Bài toán Cho hàm số f(x) liên tục đoạn [a;b] α, β hai số dương Chứng minh phương trình: f(x) = αf(aα)++βfβ (b) có nghiệm đoạn [a;b]
Bài tốn (ROMANIA – 1998) Cho số thực a, b, cthoả mãn :2a+ 10b+ 29c= Chứng minh phương trình :ax3+bx+c= 0 có nghiệm thuộc đoạn [0; 1]
Bài toán Cho hàm sốf(x)liên tục đoạn [0; 1], nhận giá trị khoảng (0; 2) Chứng minh tồn số c∈(0; 1) cho : (1−c)2+ [f(c) + 21−c]2 = 1
Bài toán Chứng minh với a, b, c số thực tuỳ ý cho trước, phương trìnhacos 3x+
acos 2x+ccosx+ sinx= ln có nghiệm khoảng (0; 2π)
Bài tốn Cho m >0, cịn a, b, c thoả mãn điều kiện m+2a + mb+1 + mc = Chứng minh phương trình ax2+bx+c= 0 có nghiệm thuộc khoảng (0; 1).
Bài tốn Chứng minh phương trình : x5−5x4+ 15x3−x2+ 3x−7 = 0 có nghiệm duy
(164)Bài tốn Xác định số nghiệm phương trình : 2e2−x2(x6−3x4 + 5x2−1)−2e−5 =
Bài toán Chứng minh với a, b phương trìnha (25 sin 5x−sinx)+b(49 sin 7x−9 sin 3x) = có nghiệm [0; 2π]
Bài toán Cho P(x) = (x−x1) (x−x2) (x−x3), với x1 < x2 < x3 Chứng minh :
P00(x1)
P0(x
1) +
P00(x2)
P0(x
2) +
P00(x3)
P0(x
3) =
Bài toán 10 (VIỆT NAM TST – 1994) Cho đa thức bậc bốnP(x)có nghiệm dương Chứng minh phương trình : 1−x42x.P(x) + 1−
1−4x x2
P0(x)−P00(x) = có nghiệm dương Bài tốn 11 Giải phương trình :
1
2sin2x
+1
2 =cos2x+ log4 4cos
32x−cos6x−1
Bài tốn 12 Giải phương trình :
64x−8.343x−1 = + 12.4x.7x−1
Bài tốn 13 Tìm nghiệm dương phương trình:
xln
1 +
x
1+1x
−x3ln
1 +
x2
1+x12
= 1−x
Bài toán 14 (OLYMPIC SINH VIÊN VIỆT NAM – 1999) Cho hàm sốf(x)khả vi [0; 1]
và thỏa mãn điều kiện f(0) = 0;f(1) = 1; ≤ f(x) ≤ 1, ∀x ∈ R Chứng minh tồn hai số a, b∈(0; 1), a6=b cho f0(a).f0(b) =
Bài toán 15 (OLYMPIC SINH VIÊN VIỆT NAM – 1994) Cho hàm số f(x) liên tục có đạo hàm cấp (0; +∞) hàm Cho a, b hai số thực thỏa mãn điều kiện0< a < b Chứng minh phương trình : xf0(x)−f(x) = af(b)b−−bfa(a) có nghiệm thuộc khoảng(a;b)
Bài tốn 16 (OLYMPIC SINH VIÊN VIỆT NAM – 2003) Cho hàm số f(x)khả vi đoạn
[a;b]và thỏa mãn điều kiện a) f(a) = 12(a−b) b)f(b) = 12(b−a)c) f a+2b6= Chứng minh tồn số đôi khác c1, c2, c3 ∈(a;b)sao cho f0(c1).f0(c2).f0(c3) =
Bài toán 17 Chứng minh 0< b < a < π2 ta có cosa−2bb <tana−tanb <
a−b cos2a
Bài toán 18 Chứng minh với ∀x∈(0; 1) ;∀n∈N∗ , ta có xn√1−x < √1 2ne
Bài toán 19 (VMO – 1996) Cho bốn số thực không âm a, b, c, d thỏa mãn điều kiện :
2(ab+ac+ad+bc+bd+cd) +abc+abd+acd+bcd= 16 Chứng minh :
a+b+c+d≥
3(ab+ac+ad+bc+bd+cd)
(165)Bài toán 20 (OLYMPIC 30 – – 2002) Cho phương trình :xn+xn−1+ +x−1 = Chứng minh với n ngun dương phương trình có nghiệm dương xn
Chứng minh dãy số (xn) có giới hạn hữu hạn khin →+∞ Tìm lim n→∞xn
Bài tốn 21 Cho phương trình với tham số n nguyên dương : x+ 2x2+ +nxn=
1 Chứng minh phương trình có nghiệm dương với n nguyên dương, kí hiệu xn Chứng minh dãy số (xn) có giới hạn hữu hạn n → +∞ Tính giới hạn
đó
Bài tốn 22 Cho phương trình 1x +x−11 + x21−1 + +
1
xn−1 = 0(n∈N
∗)
1 Chứng minh với mọin ∈N∗ , phương trình ln có nghiệm nhất x
n∈(0; 1)
2 Chứng minh dãy số (xn) , với xn xác định câu có giới hạn Tìm giới hạn
Bài tốn 23 Tìm lim
n→+∞
1 + √1
2 +
√
3 + +
√
n
Bài toán 24 (OLYMPIC SINH VIÊN VIỆT NAM – 2002) Cho dãy số thực {un} xác
định sau :u1 =a∈, un+1 = 12ln (1 +u2n)−2002, n≥1.Chứng minh dãy {un}
dãy hội tụ
Bài toán 25 Cho dãy số thực (xn), n = 1,2,3, xác định sau :
x1 =b
xn+1 = 20063 ln x2n+ 2006
2
−20062, n= 1,2,3,
Chứng minh dãy (xn) có giới hạn hữu hạn n→+∞
Tài liệu tham khảo
1 Nguyễn Văn Mậu, Phương trình bất phương trình, Nhà xuất Giáo dục 1999 Nguyễn Văn Mậu, Dãy số áp dụng, Nhà xuất Giáo dục 2008
3 Nguyễn Văn Mậu, Các đề thi Olympic Toán sinh viên, Nhà xuất Giáo dục 2006 Phan Huy Khải, Toán nâng cao giải tích, Nhà xuất Hà Nội 2002
5 Nguyễn Khắc Minh, Nguyễn Việt Hải, Các Bài Thi Olympic Toán THPT Việt Nam (1990 – 2006)
6 Tuyển tập đề thi Toán học Olympic truyền thống Tỉnh phía nam Các nguồn tài liệu Internet
8 Tạp chí Tốn học Tuổi trẻ
(166)MỘT SỐ PHƯƠNG PHÁP GIẢI HỆ PHƯƠNG TRÌNH
Lê Văn Thẩn, Trường THPT Chuyên Lê Quý Đơn - Khánh Hịa
Hệ phương trình đại số mảng kiến thức quan trọng chương trình tốn học phổ thơng, thường gặp kì thi tuyển sinh vào lớp 10; tuyển sinh đại học, cao đẳng; thi học sinh giỏi Mặc dù học sinh cọ sát phần nhiều song phần lớn em thường lúng túng trình tìm cách giải Kinh nghiệm tơi mặt hình thức khơng Cái phân loại có tính chất xun suốt chương trình bám vào kĩ thuật quen thuộc, phù hợp với tư học sinh Thêm vào đó, với tốn có phân tích lơgic, có tổng quát điều đặc biệt cho học sinh tìm gốc tốn, tốn từ đâu mà có, người ta tạo chúng cách Thông qua việc làm thường xuyên này, học sinh thích nghi cách tốt, có tư sáng tạo, có lực làm toán tạo toán Học sinh thường hiểu sâu thích nghi học phần
1 Phương pháp thế
- Cơ sở phương pháp Ta rút ẩn (hay biểu thức) từ phương trình hệ vào phương trình cịn lại
- Nhận dạng.Phương pháp thường hay sử dụng hệ có phương trình bậc ẩn
Ví dụ Giải hệ phương trình
2x+ 3y= (1)
3x2−y2+ 2y= 4 (2)
Chứng minh Từ (1) ta có x= 5−23y vào (2) ta 5−23y2−y2+ 2y−4 = 0
⇔3(25−30y+ 9y2)−4y2+ 8y−16⇔23y2−82y+ 59 = 0⇔y = 1, y = 59 23
Vậy tập nghiệm hệ phương trình (1; 1) ; −31 23;
59
23
Ví dụ Giải hệ phương trình
x4+ 2x3y+x2y2 = 2x+ (1)
x2+ 2xy= 6x+ 6 (2)
Phân tích Phương trình (2) bậc y nên ta dùng phép
Chứng minh x= không thỏa mãn (2) x6= 0, (2)⇔y = 6x+62x−x2 vào (1) ta
x4+ 2x3
6x+ 6−x2
2x
+x2
6x+ 6−x2
2x
2
(167)⇔x4+x2(6x+ 6−x2) + (6x+ 6−x 2)2
4 = 2x+ ⇔x(x+ 4)
3 = 0⇔
x=
x=−4
Do x6= nên hệ phương trình có nghiệm −4;174
Chú ý
+ Hệ phương trình theo phương pháp sau: Hệ ⇔
(x2+xy)2 = 2x+ 9
x2+xy= x2+6x+6
2
⇔ (
x2+6x+6
2
2
= 2x+
x2+xy= x2+62x+6
+ Phương pháp thường công đoạn cuối ta sử dụng phương pháp khác
2 Phương pháp cộng đại số
- Cơ sở phương pháp Kết hợp phương trình hệ phép tốn: cộng, trừ, nhân, chia ta thu phương trình hệ mà việc giải phương trình khả thi có lợi cho bước sau
-Nhận dạng Phương pháp thường dùng cho hệ đối xứng loại II, hệ phương trình có vế trái đẳng cấp bậc k
Ví dụ Giải hệ phương trình (
3y= y2x+22
3x= x2y+22 Chứng minh ĐK:xy6=
Hệ ⇔
3x2y =y2 + (1)
3y2x=x2+ 2 (2) Trừ vế hai phương trình ta 3x2y−3xy2 =y2−x2 ⇔3xy(x−y) + (x−y)(x+y) = 0⇔
x−y= 3xy+x+y=
TH x−y= ⇔y =x vào (1) ta 3x3−x2−2 = 0⇔x=
TH 3xy+x+y= Từ 3y= y2x+22 ⇒y >0, 3x=
x2+2
y2 ⇒x >0
⇒3xy+x+y >0Do TH khơng xảy
Vậy hệ phương trình có nghiệm (1; 1)
Ví dụ Giải hệ phương trình √ x + q
2−1
y = (1)
1
√
y +
q
2−
x = (2)
Chứng minh ĐK: x≥ 2, y ≥
1
2 Trừ vế hai pt ta √ x − √ y + q
2−1
y −
q
2−
x = 0⇔
√
y−√x
√
xy +
2−
y − 2−
1
x
q
2−
y +
q
2−
x
= 0⇔ √ y−x
xy √x+√y +
y−x xyq2−
y +
q
2−
x
(168)TH 1.y−x= 0⇔y=x vào (1) ta √1
x +
q
2−
x =
Đặt t= √1
x, t >0ta
√
2−t2 = 2−t⇔
2−t≥0
2−t2 = 4−4t+t2 ⇔
t≤2
t2−2t+ = ⇔t = 1⇒x= 1và y=
TH √
xy(√x+√y) +
1
xyq2−1
y+
√
2−1
x
= 0.TH vô nghiệm ĐK
Vậy hệ có nghiệm (1; 1)
Ví dụ Giải hệ phương trình
3x2+ 5xy−4y2 = 38 5x2−9xy−3y2 = 15
Phân tích.Đây hệ phương trình có vế trái đẳng cấp bậc hai nên ta cân số hạng tự thực phép trừ vế
Chứng minh Hệ ⇔
45x2+ 75xy−60y2 = 570
190x2−342xy−114y2 = 570 ⇒ −145x
2+ 417xy+ 54y2 = 0
Giải phương trình ta đượcy = 13x, y =−145
18xthế vào hai phương trình
hệ ta thu kết
Chú ý - Cách giải áp dụng cho pt có vế trái đẳng cấp bậc cao
- Cách giải chứng tỏ hệ phương trình hoàn toàn giải cách đặt
y=tx, x 6= đặt x=ty, y6=
Ví dụ Tìm m để
3x2+ 2xy+y2 = 11
x2+ 2xy+ 3y2 = 17 +m có nghiệm
Phân tích Để có kết nhanh ta đặt y =tx, x6=
Chứng minh TH x= ⇒
y2 = 11
3y2 =m+ 17 ⇔
y2 = 11
y2 = m+17
Vậy hệ có nghiệmx= ⇔ m+17
3 = 11⇔m= 16
TH x6= Đặt y=tx Hệ ⇔
3x2+ 2tx2+t2x2 = 11
x2+ 2tx2+ 3t2x2 = 17 +m
⇔
(3 + 2t+t2)x2 = 11
(1 + 2t+ 3t2)x2 = 17 +m ⇔
x2 = 3+211t+t2
(1 + 2t+ 3t2).3+211t+t2 = 17 +m
⇔
x2 = 3+211t+t2
(m−16)t2 + 2(m+ 6)t+ 3m+ 40 = (∗)
Ta có 3+211t+t2 > 0, ∀t nên hệ có nghiệm ⇔ pt (*) có nghiệm Điều xảy
m= 16 m6= 16, ∆0 = (m+ 6)2−(m−16)(3m+ 40)≥0
⇔5−√363≤m≤5 +√363
(169)Ví dụ Tìm m để
5x2+ 2xy−y2 ≥3 2x2+ 2xy+y2 ≤ m
m−1
có nghiệm
Chứng minh Nhân vế bpt thứ hai với -3 ta
5x2+ 2xy−y2 ≥3
−6x2−6xy−3y2 ≥ −3−
m−1
Cộng vế hai bpt chiều ta −x2−4xy−4y2 ≥ −
m−1 ⇔(x+ 2y) ≤
m−1
Điều kiện cầnđể hệ bpt có nghiệm m1−1 >0⇔m >1
Điều kiện đủ.Với m >1 Xét hệ pt
5x2+ 2xy−y2 = 3 2x2+ 2xy+y2 = (II)
Giả sử (x0;y0) nghiệm hệ (II) Khi
5x2
0+ 2x0y0−y02 = 2x20+ 2x0y0+y02 =
⇒
5x2
0+ 2x0y0−y02 ≥3 2x2
0+ 2x0y0+y20 ≤
m m−1
Vậy nghiệm hệ (II) nghiệm hệ (I)
(II)⇔
5x2+ 2xy−y2 = 3
−6x2−6xy−3y2 =−3 ⇒ −x −
4xy−4y2 = ⇔x+ 2y= ⇔x=−2y
Thayx=−2y vào pt thứ hệ (II) ta
8y2−4y2+y2 = 1⇔5y2 = 1⇔y =±√1
5 ⇒x=∓
√
5
Hệ (II) có nghiệm, hệ (I) có nghiệm Vậy m >1
Ví dụ Giải hệ phương trình
√
3x1 + x+1y=
√
7y1−
x+y
= 4√2
Phân tích.Các biểu thức ngoặc có dạng a+b a˘b nên ta chia hai vế pt thứ cho√3x chia hai vế pt thứ hai cho √7y
Chứng minh ĐK: x≥0, y ≥0, x+y6=
Dễ thấy x= y= không thỏa mãn hệ pt Vậy x >0, y >0
Hệ ⇔
1 +
x+y
= √2 3x
1−
x+y
= √ √ 7y ⇔ (
2 = √2
3x +
4√2
√
7y
2
x+y =
2
√
3x −
4√2
√
7y
⇔ ( 1
√
3x +
2√2
√
7y = (1)
1
√
3x −
2√2
√
7y =
1
x+y
Nhân theo vế hai pt hệ ta
1
√
3x +
2√2
√
7y
1
√
3x −
2√2
√
7y
= x+1y
⇔
3x −
8 7y =
1
x+y ⇔7y
2 −38xy−24x2 = 0⇔
y= 6x y=−4
7x
TH y= 6x vào pt (1) ta
1
√
3x +
2
√
21x = 1⇔x=
11 + 4√7
21 ⇒y=
(170)TH 2.y=−4
7xkhông xảy x >0, y >0
Vậy hệ pt có nghiệm nhất(x;y) = 11+4
√
7 21 ;
22+8√7
Chú ý Hệ phương trình có dạng
a+b=m a−b=n ⇔
m+n = 2a m−n = 2b
Trong trường hợp này, dạng thứ có vế phải chứa thức nên ta chuyển dạng thứ hai sau nhân vế để thức
Tổng quát ta có hệ sau: ( a
√
bx =m+ n px+qy c
√
dy =m+ n px+qy
Ví dụ Giải hệ phương trình
x2(y+z)2 = (3x2+x+ 1)y2z2
y2(z+x)2 = (4y2+y+ 1)z2x2
z2(x+y)2 = (5z2+z+ 1)x2y2
Phân tích Nếu chia hai vế phương trình chox2y2z2 thì ta hệ đơn giản
hơn
Chứng minh TH xyz = Nếu x = hệ ⇔ y2z2 = 0 ⇔
y=
z=t, t∈R
z =
y =t, t∈R
TH xyz 6= 0.Chia hai vế pt hệ cho x2y2z2 ta z + y
= + 1x +x12 (1)
1
x +
1
z
2
= + 1y +y12 (2) y + x
= +1z +z12 (3)
Cộng vế phương trình hệ ta z + y + x+ z + y + x
= 12 +
x+ y + z +
x2 +
y2 + z2 ⇔ x + y + z − x + y + z
−12 = 0⇔ " 1
x +
1
y +
1
z = 4(4)
1
x +
1
y +
1
z =−3(5)
Từ (4) (1) ta có 4−
x
2
= + 1x +x12 ⇔
9
x = 13⇔x=
9 13
Từ (4) (2) ta có y= 34 Từ (4) (3) ta có z = 119
Tương tự, từ (5), (1), (2), (3) ta có x=−5
6, y =−1, z =−
Vậy hệ có tập nghiệm
S =
(t; 0; 0); (0;t; 0); (0; 0;t);
13; 4; 11 ; −5
6;−1;−
, t∈R
(171)Nhận xét Qua ví dụ ta thấy: từ hệ phương trình đơn giản, cách đổi biến số (ở phép thay nghịch đảo) ta thu hệ phức tạp Vậy hệ phức tạp ta nghĩ đến phép đặt ẩn phụ để hệ trở nên đơn giản
3 Phương pháp đặt ẩn phụ
Ví dụ 10 Giải hệ phương trình
x+y+xy=−1
x2 +y2−xy = 7
Chứng minh Đây hệ đối xứng loại I đơn giản nên ta giải theo cách phổ biến Hệ ⇔
(x+y) +xy=−1 (x+y)2−3xy=
Đặt
x+y=S
xy=P (∃x, y ⇔S
2 ≥4P) ta được
S+P =−1
S2 −3P = 7 ⇔
S= 1, P =−2
S=−4, P =
TH
S =
P =−2 ⇒
x+y=
xy=−2 ⇔
x=−1, y =
x= 2, y =−1
TH
S =−4
P = ⇒
x+y=−4
xy= ⇔
x=−1, y =−3
x=−3, y =−1
Vậy tập nghiệm hệ
S ={(−1; 2); (2;−1); (−1;−3); (−3;−1)}
Chú ý - Nếu hệ pt có nghiệm làx;y tính đối xứng, hệ có nghiệm lày, x Do vậy, để hệ có nghiệm điều kiện cần làx=y
- Khơng phải lúc hệ đối xứng loại I giải theo cách Đơi việc thay đổi cách nhìn nhận phát cách giải tốt
Ví dụ 11 Giải hệ phương trình
x2 +y2+x+y= 18
xy(x+ 1)(y+ 1) = 72
Phân tích Đây hệ đối xứng loại I
Hướng Biểu diễn pt theo tổng x+y tích xy
Hướng Biểu diễn pt theo x2+x y2+y Rõ ràng hướng tốt
Chứng minh Hệ ⇔
(x2+x) + (y2+y) = 18 (x2+x)(y2+y) = 72 Đặt
x2+x=a, a≥ −1
y2+y=b, b≥ −1
ta
a+b= 18
ab= 72 ⇔
a = 6, b= 12
a = 12, b=
TH
a=
b= 12 ⇒
x2+x= 6
y2+y= 12 ⇔
x= 2, x=−3
y= 3, y =−4
TH 2.Đổi vai trò a b ta
x= 3, x=−4
(172)S ={(2; 3); (2;−4); (−3; 3); (−3;−4); (3; 2); (−4; 2); (3;−3); (−4;−3)}
Nhận xét Bài tốn hình thành theo cách sau Xuất phát từ hệ phương trình đơn giản
a+b = 18
ab= 72 (I)
1 Thay a=x2+x, b=y2+y vào hệ (I) ta hệ
(1)
x2+y2+x+y = 18
xy(x+ 1)(y+ 1) = 72 ví dụ 11
2 Thay a=x2+xy, b=y2−xy vào hệ (I) ta hệ
(2)
x2+y2 = 18
xy(x2−y2) = 72
3 Thay a=x2+ 2x, b= 2x+y vào hệ (I) ta hệ (3)
x2+ 4x+y= 18
x(x+ 2)(2x+y) = 72
4 Thay a=x+x1, b =y+1y vào hệ (I) ta hệ
(4)
(x+y)xy+x+y= 18xy
(x2+ 1)(y2+ 1) = 72xy
5 Thay a=x2+ 2xy, b=y2−xy vào hệ (I) ta hệ (5)
x2+y2+xy= 18
xy(x+ 2y)(y−x) = 72
- Như vậy, với hệ xuất (I), cách thay biến ta thu nhiều hệ pt - Thay hệ xuất phát (I) hệ xuất phát (II)
a+b =
a2−b2 = 21 làm tương tự
ta lại thu hệ khác Chẳng hạn
6 Thay a=x2+y2, b=xy vào hệ (II) ta hệ (6)
x2+y2+xy =
x4+y4+x2y2 = 21
7 Thay a=x+x1, b =y+1y vào hệ (II) ta hệ
(7)
(
x+y+x1 + 1y =
x2−y2+
x2 −
1
y2 = 21
8 Thay a=x+y1, b= xy vào hệ (II) ta hệ
(8)
xy+x+ = 7y
(173)9 Thay a=x+y, b= 1y vào hệ (II) ta hệ
(9)
(x+y)y+ = 9y
(x+y−2)2y2−21y2 = 1
10 Thay a=x2+ 2x, b =y2 + 2x vào hệ (II) ta hệ (10)
x2+y2+ 4x= 7
x4−y4+ 4x(x2−y2) = 21
Như vậy, biết cách tạo toán nghĩ cách giải tốn khác
Ví dụ 12 Giải hệ pt sau
a)
x(x+y+ 1)−3 = (x+y)2−
x2 + =
b)
x2 +y+x3y+xy2+xy=−5
x4 +y2 +xy(1 + 2x) = −5
c)
x+y−√xy=
√
x+ +√y+ = d)
x2+y2+ 2(x+y) = 7
y(y−2x)−2x= 10
Chứng minh a) ĐK x6= Hệ⇔
x+y+ 1−3.1x = (x+y)2−5 x12+ =
Đặt x+y =a, 1x =b ta hệ
a+ 1−3b=
a2 −5b2+ = ⇔
a= 3b−1
(3b−1)2−5b2+ = ⇔
a= 2, b=
a= 12, b = 12 ⇒
x=y=
x= 2, y =−3
b) Hệ ⇔
(x2 +y) +xy(x2+y+ 1) =−5 (x2 +y)2+xy=−5
4
Đặtx2 +y=a, xy=b ta được
a+b(a+ 1) =−5
a2+b=−5
⇔
a2 −a−ab= 0
b =−5 −a
2 ⇔
a= 0, b =−5
a=−1
2, b=−
TH1
a =
b =−5
⇒
x2+y=
xy =−5
⇔
x= q
5
y=−3 q
25 16
TH2
a=−1
b =−3
⇒
x2+y=−1
xy=−3
⇔
x2 − 2x =−
1
y =− 2x
⇔
x=
y=−3
Vậy tập nghiệm hệ pt S =n 1;−3
; q
5 4;−
3 q
25 16
o
c) ĐK: x≥ −1, y ≥ −1, xy ≥0
Hệ ⇔
x+y−√xy=
x+y+ + 2p(x+ 1)(y+ 1) = 16 ⇔
x+y−√xy =
x+y+ 2√x+y+xy+ = 14
Đặt x+y =a, √xy=b a≥ −2, b≥0, a2 ≥4b2 ta hệ pt
a−b =
a+ 2√a+b2+ = 14 ⇔
a= +b
2√b2+b+ = 11−b ⇔
a = +b
(174)⇔
b =
a = ⇒
x=
y= (thỏa điều kiện)
d) Hệ ⇔
(x+ 1)2+ (y+ 1)2 = 9 (y−x)2−(x+ 1)2 = 9
Đặt a=x+ 1, b=y+ ⇒b−a=y−x ta hệ
a2+b2 = 9 (b−a)2−a2 =
⇒a2+b2 = (b−a)2−a2 ⇔a2 =−2ab⇔a= hoặca=−2b a= 0⇒b =±3⇒x=−1, y = x=−1, y =−4
a=−2b ⇒5b2 = 9 ⇔b =±√3
5 ⇒a=∓
√
5
⇒x=−1− √6
5, y =−1 +
√
5 x=−1 +
√
5, y =−1−
√
5
Vậy hệ có nghiệm
4 Phương pháp đưa dạng tích
-Cơ sở phương pháp Phân tích hai phương trình hệ thành tích nhân tử Đơi cần tổ hợp hai phương trình thành phương trình hệ đưa dạng tích
- Cách thành lập hệ dạng
(ax+by+c)f(x;y) =
g(x;y) = f(x, y) chọn
sao cho
f(x;y) =
g(x;y) = vô nghiệm
f(x;y) =
g(x;y) = giải được; g(x, y) chọn cho
ax+by+c=
g(x;y) = giải thỏa mãn kết hợp vớif(x, y)
Ví dụ 13 Giải hệ phương trình sau
xy+x+y=x2−2y2 (1)
x√2y−y√x−1 = 2x−2y (2)
Phân tích Rõ ràng, việc giải phương trình (2) hay kết hợp (1) với (2) không thu kết khả quan nên tập trung để giải (1)
Chứng minh ĐK: x≥1, y ≥0
(1)⇔y(x+y) + (x+y) = x2−y2 ⇔(x+y)(y+ 1−x+y) =
TH1 x+y= (loại dox≥1, y ≥0 )
TH2 2y+ 1−x= 0⇔x= 2y+ vào pt (2) ta
(2y+ 1)√2y−y√2y= 4y+ 2−2y⇔(y+ 1)√2y= 2(y+ 1)
⇔
y+ =
√
2y = ⇔
y=−1
y= Do y≥0⇒y = Vậy hệ có nghiệm(x;y) = (5; 2)
Chú ý Do phân tích thành tích hai nhân tử bậc đối y (hay x) nên giải pt (1) cách coi (1) pt bậc hai ẩn y (hoặc x)
Ví dụ 14 Giải hệ phương trình sau
x−
x =y−
1
y (1)
(175)Phân tích Từ cấu trúc pt (1) ta thấy đưa (1) dạng tích
Chứng minh ĐK:xy6= (1)⇔x−y−1
x+
1
y = ⇔x−y+ x−y
xy = ⇔(x−y)
1 + xy1 =
TH1 x=y vào (2) ta x3−2x+ = 0⇔x= x= −1±
√
5 (TM)
TH2 + xy1 = 0⇔y =−1
x vào pt (2) ta
x4+x+ = 0⇔(x2− 2)
2+ (x+ 2)
2+3
2 = Pt vô nghiệm
Vậy hệ có nghiệmS =n(1; 1); −1+
√
5 ;
−1+√5
; −1−
√
5 ;
−1−√5
o
Ví dụ 15 Giải hệ phương trình sau
x−
x3 =y−
1
y3 (1)
(x−4y)(2x−y+ 4) =−36 (2)
Chứng minh
x−
x3 =y−
1
y3 ⇔(x−y) =
(y−x)(y2+xy+x2)
x3y3 ⇔ "
x=y
y2+xy+x2
x3y3 =−1
TH1 x=y vào (2) ta x2+ 4x−12 = 0⇔
x=−6
x=
TH2 y2+xxy3y+3x2 =−1⇒xy <0
thế vào pt (2) ta
(2)⇔2x2+ 4y2−9xy+ 4x−16y=−36⇔2(x+ 1)2+ 4(y−2)2−9xy =−18
Trường hợp không xảy xy <0⇒2(x+ 1)2+ 4(y−2)2−9xy >0
Vậy hệ có nghiệmS ={(2; 2); (−6;−6)}
Ví dụ 16 Giải hệ phương trình sau
x2+y2+ 8xy
x+y = 16 (1)
√
x+y=x2−y (2)
Phân tích Rõ ràng, việc giải phương trình (2) hay kết hợp (1) với (2) không thu kết khả quan nên tập trung để giải (1)
Chứng minh ĐK: x+y >0 (1)⇔(x2+y2)(x+y) + 8xy= 16(x+y)
⇔
(x+y)2−2xy(x+y) + 8xy= 16(x+y)
⇔(x+y)(x+y)2−16−2xy(x+y−4) =
⇔(x+y−4) [(x+y)(x+y+ 4)−2xy] =
TH1 x+y−4 = vào (2) ta x2+x−6 = 0⇔
x=−3⇒y=
x= 2⇒y=
TH2.(x+y)(x+y+ 4)−2xy= 0⇔x2+y2+ 4(x+y) = Pt vô nghiệm điều kiện
Vậy hệ có nghiệmS ={(−3; 7); (2; 2)}
5 Phương pháp sử dụng tính đơn điệu hàm số
* Cơ sở phương pháp Nếu f(x) đơn điệu khoảng (a;b) x, y ∈ (a;b) f(x) =
(176)* Cách xây dựng hệ theo phương pháp
- Lấy hàm số f(t) đơn điệu khoảng (a;b), u(x;y), v(x;y)∈(a;b)
- Lấy g(x;y) cho hệ
u(x;y) =v(x;y)
g(x;y) = giải tập xác định chúng
- Lập hệ phương trình
f(u) =f(v)
g(x;y) =
Ví dụ 17 Giải hệ phương trình sau
2x−2y = (y−x)(xy+ 2)
x2+y2 = 2
Phân tích Nếu thay =x2+y2 vào phương trình thứ ta hđt
Chứng minh Thay =x2+y2 vào phương trình thứ ta được
2x−2y = (y−x)(xy+x2+y2)⇔2x−2y =y3−x3 ⇔2x+x3 = 2y+y3(1)
Xét hàm số f(t) = 2t+t3, t∈
R có f0(t) = 2tln + 3t2 > 0, ∀t ∈R suy f(t) đồng biến
trên R (1) ⇔f(x) = f(y) ⇔x =y vào pt thứ hai ta x=y =±1 Vậy tập nghiệm
của hệ S = {(1; 1); (−1;−1)}
Ví dụ 18 Giải hệ phương trình sau
(4x2+ 1)x+ (y−3)√5−2y= (1) 4x2+y2+ 2√3−4x= (2)
Chứng minh ĐK:
3−4x≥0 5−2y≥0 ⇔
x≤
y≤ (1)⇔(4x2+ 1)2x+ (2y−6)√5−2y=
⇔
(2x)2+
(2x) = h √5−2y2
+ 1i√5−2y⇔(2x)3+ 2x= √5−2y3
+√5−2y
⇔ f(2x) = f(√5−2y) với f(t) = t3+t f0(t) = 3t2 + 1> 0, ∀t ∈
R, f(t) đồng biến R.Vậy f(2x) = f(
√
5−2y)⇔2x=√5−2y ⇔y= 5−24x2, x≥0
Thế vào phương trình thứ hai ta được4x2+5−4x2
2
2
+ 2√3−4x−7 = 0⇔g(x) =
Với g(x) = 4x2+
5−4x2
2
2
+ 2√3−4x−7, x∈
0;34
Ví dụ 19 Giải hệ phương trình sau
x3−3x=y3−3y (1)
x2+y2 = (2)
Phân tích Ta giải hệ phương pháp đưa dạng tích Tuy nhiên ta muốn giải hệ phương pháp sử dụng tính đơn điệu hàm số Hàm sốf(t) =t3−3t không
đơn điệu tồn trục số, nhờ có (2) ta giới hạn x y đoạn[−1; 1]
Chứng minh Từ (2) ta có x2 ≤1, y2 ≤1⇔x, y ∈[−1; 1]
Hàm số f(t) = t3 −3t có f0(t) = 3t2−3 < 0, ∀t ∈ (−1; 1) ⇒ f(t) đồng biến đoạn [−1; 1].x, y ∈ [−1; 1] nên (1) ⇔ f(x) = f(y) ⇔ x = y vào pt (2) ta x = y = ±
√
2
Vậy tập nghiệm hệ S = n √
2 ;
√
2
; − √
2 ;−
√
2
o
(177)Ví dụ 20 Tìm giá trị m để hệ phương trình sau có nghiệm
x3−y3+ 3y2−3x−2 =
x2+√1−x2−3p2y−y2+m= 0
Chứng minh ĐK: −1≤x≤1, 0≤y≤2 (1)⇔x3−3x= (y−1)3−3(y−1)
Hàm số f(t) =t3−3t nghịch biến đoạn [−1; 1]
x, y−1∈[−1; 1] nên f(x) = f(y−1)⇔x=y−1⇔y=x+
Thế vào pt(2) ta đượcx2−2√1−x2 =−m (3)
Hệ có nghiệm ⇔Pt (3) có nghiệm x∈[−1; 1]
Xét g(x) = x2−2√1−x2, x∈[−1; 1], g0(x) = 2x1 + √ 1−x2
g0(x) = 0⇔x= 0.g(0) =−2, g(±1) = g(0) =−2, g(±1) =
Pt (3) có nghiệm x∈[−1; 1] ⇔ −2≤ −m ≤1⇔ −1≤m≤2
Ví dụ 21 Giải hệ phương trình
x+√x2+ = 3y
y+py2 + = 3x
Chứng minh Trừ vế hai pt ta
x+√x2 + 1−y+py2+ 1= 3y −3x ⇔x+√x2+ + 3x=y+p
y2+ + 3y
f(x) = f(y) nên f(t) =t+√t2+ + 3t.f(t) = + √ t t2+1 +
tln 3 >0, ∀ ∈
R
⇒f(t)đồng biến R Bởi f(x) =f(y)⇔x=y vào pt thứ ta
x+√x2 + = 3x ⇔1 = 3x √x2+ 1−x
⇔g(0) =g(x)
Với g(x) = 3x √x2+ 1−x
.g0(x) = 3xln √x2+ 1−x
+ 3x
x
√
x2+1 −1
= 3x √x2+ 1−x
ln 3− √
x2+1
>0, ∀x∈R do√x2+ 1−x >0 và √x2+ 1≥1
Suy g(x) đồng biến R Bởi g(x) =g(0) ⇔x=
Vậy hệ phương trình có nghiệm x=y=
Ví dụ 22 Chứng minh hệ (
ex = 2007− √y y2−1
ey = 2007− √x x2−1
có nghiệm x >0, y >0
Chứng minh.ĐK:
x2 −1>0
y2−1>0 ⇔
x∈(−∞;−1)∪(1; +∞)
y∈(−∞;−1)∪(1; +∞) Do
x >0
y >0 nên
x >1
y >1
Trừ vế hai pt ta ex−ey = √x
x2−1 −
y
√
y2−1 ⇔e
x− √x x2−1 =e
y − √y y2−1
Hayf(x) =f(y) với f(t) =et− √ t
t2−1, t∈(1; +∞)
f0(t) =et+
(t2−1)√t2−1 >0, t∈(1; +∞)⇒f(t)đồng biến (1; +∞)
Bởi f(x) =f(y)⇔x=y vào pt thứ ta
ex = 2007−√ x
x2−1 ⇔e
x+ √x
x2−1 −2007 = 0⇔g(x) =
Với g(x) =ex+ √x
x2−1 −2007, x∈(1; +∞).Ta có
g0(x) =ex−
(x2−1)√x2−1; g
00(x) =ex+ 3x(x2−1)
(178)Suy g0(x) đồng biến (1; +∞) g0(x) liên tục (1; +∞) có
lim
x→1+g
0
(x) = −∞, lim
x→+∞g
(x) = +∞
nên g0(x) = có nghiệm trênx0 ∈(1; +∞)
g0(x)>0⇔g0(x)> g0(x0)⇔x > x0 g0(x)<0⇔1< x < x0
Từ BBT g(x) ta suy pt g(x) = có nghiệm x∈ (1; +∞) Vậy hệ phương trình
đã cho có nghiệm dương
Ví dụ 23 Giải hệ phương trình
ln(1 +x)−ln(1 +y) = x−y (1)
x2 −12xy+ 20y2 = (2)
Chứng minh ĐK: x >−1, y >−1
(1)⇔ln(1 +x)−x= ln(1 +y)−y⇔f(x) =f(y)với
f(t) = ln(1 +t)−t, t∈(−1; +∞)
f0(t) = 1+1t −1 = 1+−tt = ⇔ t = ∈ (−1; +∞) ⇒ f(t) đồng biến (−1; 0) nghịch biến khoảng
TH x, y ∈(−1; 0) x, y ∈(0; +∞) f(x) = f(y)⇔x=y
Thế vào pt (2) ta đượcx=y= (không thỏa mãn)
TH x∈(−1; 0),y ∈(0; +∞) ngược lại xy <0⇒x2−12xy+ 20y2 >0
TH xy= hệ có nghiệmx=y= Vậy hệ có nghiệm x=y =
Một số đề xuất
Mỗi tốn thường có gốc nó, việc học sinh phát toán gốc thấy toán học thực tế, tự nhiên khơng khó em nghĩ đồng thời tạo niềm tin hứng thú học tập với em Với tinh thần theo hướng thày cô giáo em học sinh tìm nhiều kinh nghiệm hay với nhiều đề tài khác Chẳng hạn, tốn tích phân, tốn tổ hợp – xác suất, toán phương pháp tọa độ mặt phẳng, không gian
Tài liệu tham khảo
[1] Nguyễn Văn Mậu,Phương trình hàm, Nhà xuất Giáo dục, 1997
[2] Pl Kannappan, Functional Equations and Inequalities with Applications, Springer, 2009, 295-323
(179)MỘT SỐ BÀI TOÁN VỀ ĐA THỨC TRONG CÁC KÌ THI HỌC SINH GIỎI
Huỳnh Kim Linh - Tô Hùng Khanh, Trường THPT Chuyên Lê Q Đơn - Khánh Hịa
1 Các tốn có lời giải
Bài tốn Tìm tất đa thức f(x) = x3 +ax2+bx +c, với a, b, c là số thực thỏa
phương trình f(x) = có nghiệm a, b, c
Giải Đa thức f(x) = x3 + ax2 +bx +c có nghiệm là a, b, c được viết dạng :
f(x) = (x−a)(x−b)(x−c)
Đồng hệ số ta có :
−(a+b+c) =a ab+bc+ca=b
−abc =c
Với c= 0, ta có nghiệma =b =c= a= 1, b=−2, c =
Với c6= ta có
−(a+b+c) = a ab+bc+ca=b
−abc=c
⇔
a =−1
b
c= 2b −b
b4+b3−2b2+ =
b4+b3−2b2 + = 0⇔(b+ 1)(b3−2b+ 2) = 0⇔b=−1 b3˘2b+ =
Vớib =−1thì a= 1, c=−1Vớib3−2b+ = 0,bằng cách đặtb= 2q2
3xta đưa phương trình
về dạng 4x3−3x+ 3√3
2√2 = 0, phương trình có nghiệm :
x=
3 r
−3√3 2√2 +
q
27 −1 +
3 r
−3√3 2√2 −
q
27 −1
2 =
3 p√
19−√27−p3 √
19 +√27 2√2
Từ suy rab =
q
2 ·
3
√√
19−√27−3 √√
19+√27 2√2 =
3
√√
19−√27−3 √√
19+√27
√
3
Đặtb0 =
3
√√
19−√27−3 √√
19+√27
√
3 ta suy raa0, c0 tương ứng
Vậy có tất đa thức thỏa mãn đề làf1(x) =x3;f2(x) =x3−2x;f3(x) =x3+x2−x− 1;f4(x) =x3+a0x2+b0x+c0 với
b0 =
3
√√
19−√27−3 √√
19+√27
√
3
a0 =−b1
0
c0 = b20 −b0
(180)Giải Giả sửα nghiệm chung củaP(x) Q(x),
Ta có : α5 =α+ 1 và α2 =−aα−b.
α+1 =α5=α(α2)2 =α(−aα−b)2 =α[a2(−aα−b) + (2abα+b2)]
= (2ab−a3)(−aα−b) + (b2−a2b)α=· · ·= (a4−3a2b+b2)α+a3b−2ab2
⇒
a4−3a2b+b2 = (1)
a3b−2ab2 = 1 (2)
Từ (1)⇒b2 = 1−a4+ 3a2b thay vào (2) rút gọn b= 2a5−2a−1
5a3
Thay vào (1) ta a10+ 3a6 −11a5 −4a2−4a−1 = điều trái vớia ∈ Q Vì phương trìnha10+ 3a6−11a5−4a2−4a−1 = 0 khơng có nghiệm hữu tỉ.
Bài tốn Giả sử a, b nghiệm phương trình : x4 +x3 −1 = (1) Chứng minh ab nghiệm phương trình x6+x4+x3−x2−1 = (2)
Giải Giả sửa, b, c, d nghiệm phương trình (1) Suy P(x) = x4+x3−1 = (x−a)(x−b)(x−c)(x−d) = 0
Ta chứng minh :(ab)3+ (cd)3+ab+cd+ = 0
Khi ta suy : (ab)6+ (ab)4+ (ab)3−(ab)2−1 = ⇔ab3−
(ab)3 +ab−
ab + =
Vìabcd =−1 hay ab=−1
cd.Thật P(a) = P(b) =
⇒a3 =
a+ 1, b
3 =
b+ ⇒(ab)
3 =
(a+ 1)(b+ 1) =
(1 +c)(1 +d)
P(−1)
Hay(ab)3 =−(1 +c)(1 +d) Tương tự (cd)3 =−(1 +a)(1 +b)Suy
(ab)3+ (cd)3+ab+cd+ =−(1 +c)(1 +d)−(1 +a)(1 +b) + ab + cd + =−1−a−b−c−d= (Đúng a+b+c+d= 1)
Bài tốn Giả sử an dãy số Fibonaxi xác định
a1 =a2 =
an+2 =an+1+an(n∈N)
Chứng minh đa thức P(x) bậc 1005 thỏa điều kiện P(k) = ak với k = 1007, ,2012 Thì
P(2013) = a2013−1
Giải Ta chứng minh quy nạp theo n∈N Khẳng định tổng quát : NếuP(x)có bậc n thỏa P(k) =ak với k =n+ 2, ,2n+
Thì P(2n+ 3) =a2n+3−1
•n = ta có P(3) = 2, P(4) = 3⇒P(x)≡x−1 P(5) = = a5−1
•Giả sử khẳng định với n−1.Ta chứng minh với n Giả sử đa thức P(x) bậc n thỏa P(k) = ak với k=n+ 2, ,2n+
Xét đa thức Q(x) = P(x+ 2)−P(x+ 1) có bậc khơng vượt q n −1 thỏa Q(k) = ak với
k=n+ 1, ,2n
VìQ(k) =P(k+ 2)−P(k+ 1) =ak+2−ak+1 =ak
Nghĩa làQ(2n+ 1) =a2n+1−1 ( theo giả thiết quy nạp)
Nhưng Q(2n+ 1) =P(2n+ 3)−P(2n+ 2)
(181)Bài tốn Tìm số ngun a cho đa thức f(x) = x13 +x + 90 chia hết cho đa thức
g(x) = x2−x+a.
Giải Giả sử :f(x) =g(x).Q(x)⇔x13+x+ 90 = (x2−x+a).Q(x)
Vìa ∈Z nên ta xét :
•a≤0 đóf(x) = x13+x+ 90và g(x) =x2−x+a có nghiệm khơng âm Vơ lía >0, cho
x=−1, x= 0, x= ta
(a+ 2).Q(−1) = 88
a.Q(0) = 90
a.Q(1) = 92
⇒2 a⇒
a=
a=
Nếu a = từ (1) suy ra88 3vơ lí Nếu a = ta có
f(x) = x2−x+a x11+x10−x9−3x8−x7+ 5x6+ 7x5−3x4−7x3+ 11x2+ 23x+ 45
Vậy a = f(x) chia hết chog(x)
Bài toán Lập đa thức bậc có nghiệm x1, x2, x3 thỏa :
x1 +
1
x2 +
1
x3 =−2 (1)
1
x2
+x12
+ x12
= (2)
x4
+x14
+ x14
= (3)
Giải Giả sử đa thức cần tìm có dạngP(x) = x3 +ax2+bx+c.
Theo định lí VIET :
x1+x2+x3 =−a
x1x2+x2x3+x3x1 =b
x1x2x3 =−c
(1)⇔ −2 = x1x2+x2x3+x3x1
x1x2x3
=−b
c ⇒b = 2c
(2) ⇔1 = x
1x22+x22x23+x23x21
x2 1x22x23
= (x1x2+x2x3+x3x1)
−2x1x2x3(x1+x2+x3)
x2 1x22x23 = b
2 −2ac
c2 ⇔b 2−
2ac=c2
(3)⇔1 = x
1x42 +x42x43+x43x41
x4 1x42x43
=· · ·= (b
2−2ac)2−2c2(a2−2b)
c4
⇔(b2−2ac)2−2c2(a2−2b) =c4
Vậy a, b, cthỏa
b= 2c
b2−2ac =c2
(b2−2ac)2−2c2(a2−2b) = c4
⇔
a= 83
b= 329
c= 169
Bài tốn Tìm tất giá trị tham số m để nghiệm x1, x2, x3, x4 đa thức :
(182)Giải Ta có
x1+x2+x3+x4 =−3
x1x2+x1x3+x1x4+x2x3+x2x4 +x3x4 =
x1x2x3 +x1x2x4+x1x3x4+x2x3x4 =−m
x1x2x3x4 =
x2x3+x2x4+x3x4 =x1x2x3x4
Suy ra4 =x1x2x3x4 =x2x3+x2x4+x3x4 = 6−x1(x2+x3+x4) = 6−x1(−3−x1) = 6+3x1+x21
Hayx2
1 =−3x1−2(*)
⇒x3
1 =x1x21 =x1(−3x1−2) =−3x12 −2x1 =−3 (−3x1−2)−2x1 = 7x1+
⇒x4
1 =x1x31 = 7x21+ 6x1 = (−3x1−2) + 6x1 =−15x1˘14
Hay(m−12)x1−4 = 0⇒x1 = m−412(m6= 12)
Thayx1 = m−412vào (*) ta : (m−1612)2 +
4
m−12+ = 0(**)
Giải phương trình (**) tìm m= m = 10
Bài tốn Cho a, b∈R Tìm đa thức P(x) thoả : xP(x−a) = (x−b)P(x), ∀x∈R
Giải
• Xéta =b = 0⇒P(x)tuỳ ý •a = 0, b6= P(x) = 0,∀x∈R
•a 6= 0, b= P(x) = const
•a 6= b6= xét trường hợp ba ∈/ N ba ∈N
Nếu ab ∈/N thay x=b x=b−a nghiệm
Tương tự thayx=b−a x=b−2a nghiệm, Do đóP(x) =x,∀x∈R
Nếu ab ∈N P(x) cóx=a, x= 2a, , x= (n−1)a nghiệm Do đóP(x) = (x−a)(x−2a) .(x−(n−1)a).Q(x)
Thế vào điều kiện ta Q(x−a) = Q(x),∀x∈R hay Q(x) = const Vậy P(x) = (x−a)(x−2a) .(x−(n−1)a)
Bài tốn Hỏi có tồn hay không đa thức f(x) bậc 2012 cho f(x2−2011) chia hết
cho f(x)
Giải Xét đa thức f(x) = (x+a)2012
Taf(x2−2011) = (x2−2011 +a)2012 =(x+a)2−2a(x+a) +a2+a−20112012
Nếu ta chọn a cho a2 +a−2011 = 0 ⇔ a = −1±√8045
2 f(x
2−2011) = (x2−a2)2012 = (x−a)2012(x+a)2012 chia hết chof(x)
Vậy đa thứcf(x) = x+−1±
√
8045
2012
thỏa điều kiện toán Cách : Xét đa thứcf(x) =
2012
Q
k=1
(x−ak), ak∈R
Ta có : f(x2−2011) =2012Q
k=1
(x2−2011−a
k)
Thì f(x2−2011) =2012Q
k=1
(x2−a2
k) =f(x)
2012
Q
k=1
(183)Vậy đa thứcf(x) = x−1±√8045
2012
thỏa điều kiện toán
Bài toán 10.Cho 0≤α≤1 Chứng minh : với số phức a phương trình :z3−az+a= 0
có nghiệm z thỏa : |z−α| ≤2−α
Giải Gọi z1, z2, z3 nghiệm pt : z3−az+a=
Ta có :
z1+z2+z3 =
z1z2+z2z3+z3z1 =−a
z1z2z3 =−a
Suy = 1−(z1+z2+z3) + (z1z2+z2z3+z3z1)−z1z2z3 = (1−z1) (1−z2) (1−z3)
Hay|1−z1| |1−z2| |1−z3|= 1⇒ ∃zi :|zi−1| ≤1
Khi đó2−α= + 1−α≥ |zi−1|+|1−α| ≥ |zi −α|
Tức có nghiệm z phương trình thoả :|z−α| ≤2−α
Bài toán 11 Cho a, b, c, d, e∈R Chứng minh phương trình ax2+ (b+c)x+d+e=
có nghiệm x0 ∈[1; +∞) phương trình ax4+bx3+cx2+dx+e = có nghiệm thực
Giải Ta có ax2
0+cx0+e=−(bx0+d)
Xétf(x) = ax4+bx3+cx2+dx+e suy ra f √x0
= (ax2
0+cx0+e) +
√
x0(bx0+d); f −√x0
= (ax20+cx0+e)−
√
x0(bx0+d)
Hay
f √x0
f −√x0
=(ax20+cx0 +e)
−x0(bx0 +d)2 = (ax2
0+cx0+e)
−x0(ax20+cx0+e) = (ax2
0+cx0+e)
(1−x0)≤0
Vậy phương trìnhf(x) = có nghiệm x∈
−√x0;
√
x0
Bài toán 12 Cho đa thức P(x) với hệ số thực bậc n (n≤ 1) có m nghiệm thực Chứng minh : đa thức Q(x) = (x2+ 1)P(x) +P,(x) có m nghiệm thực
Giải Xét f(x) =ex
3
3 +xP(x)
Dễ thấy f(x) = có tập hợp tất nghiệm thực trùng với tập nghiệm thực P(x)
Theo định lí Rolle phương trình f,(x) =ex33+x[P,(x) + (x2+ 1)P(x)] = 0 có nhất m−1
nghiệm thực
Nếu m chẵn, n lẻ thìP(x) có m+ nghiệm thực, vơ lí !
Do n chẵn Khi P,(x) + (x2+ 1)P(x) có bậc làn+ 2là số chẵn có m−1số lẻ nghiệm
thực
Suy đa thứcQ(x) = (x2+ 1)P(x) +P,(x) có m nghiệm thực.
Bài tốn 13 Cho f(x) khả vi đoạn [0; 1] thoả điều kiện :
f(0) = 0, f(1) = 0≤f(x)≤1, ∀x∈R
(184)Giải Xét hàm g(x) =f(x) +x−1 g(x) khả vi đoạn [0; 1] f(x) khả vi đoạn
[0; 1]
Do g(0) =−1và g(1) = nên theo định lí Lagrange ∃c∈(0; 1) cho g(c) =
Suy f(c) +c−1 = hay f(c) = 1−c
Mặt khác theo định lí Lagrange hàmf(x)trên đoạn[0;c] và[c; 1]ta có : f(c)c−−f0(0) =
f,(a), a∈(0;c)⊕và f(1)−f(c)
1−c =f
,(b), b ∈(c; 1)
Suy f,(a).f,(b) = f(c)
c
1−f(c) 1−c =
(1−c)c c(1−c) =
Bài toán 14 (IMO 2006) Cho P(x) đa thức bậc n > với hệ số nguyên, gọi k số nguyên dương Xét đa thức Q(x) = P(P( P(x)) .)), P xuất k lần Chứng minh có tối đa n số nguyên t cho Q(t) = t
Giải Đầu tiên , ta chứng minh với điểm cố địnhxcủa Qtrong thực tế điểm cố định P ◦P
Xét dãy cho x0 =x xi+1 =P (xi)với i≥0
Giả sửxk =x0, ta biếtP(u)−P(v)chia hết chou˘v với số nguyênu, v phân biệt Hơn
nữa
di =xi+1−xiP (xi+1)−P (xi) =xi+2−xi+1 =di+1
với mọii , kết hợp vớidk =d0 ta suy ra|d0|=|d1|= =|dk|
Giả sửd1 =d0 =d6= d2 =d (nếu khơng x3 =x1 x0 khơng thể xuất dãy
một lần ) Tương tự d3 =d, , vậyxi =x0+id6=x0 với i, điều mâu thuẫn
Màd1 =−d0, suy x2 =x0 Vậy giả thiết Q=P ◦P
Nếu số nguyên t với P(P (t)) thoả mãn P (t) = t dễ dàng nhận thấy số nghiệm toán tối đa làdegP =n
Giả sử P(t1) = t2, P (t2) = t1, P (t3) = t4, P(t4) = t3, với t1 6= t2,3,4 khơng thiết t3 6= t4)
,vìt2−t4 (t1−t3)và điều ngược lại nên t1−t3 =±(t2−t4).Giả sử t1−t3 =t2−t4
hay nói cách kháct1−t2 =t3−t4 =u6= Vì ta có t1−t4 =±(t2−t3),
nên t1−t3+u=±(t1−t3−u), điều vơ lý !
Vậy phải cót1 −t3 =t4−t2 ,hay p(t1) +t1 =p(t3) +t3 =c.Suy nghiệm nguyên t
phương trình P (P (t)) = t thoả mãn P(t)+, số số nguyên không vượt q n
Bài tốn 15 A1 Tìm tất đa thức hệ số nguyên bậc hai cho tồn đa thức q(x) hệ số nguyên thỏa đa thức có tất hệ số ±1
Lời giải 1:Chúng ta chứng minh có đa thức thỏa mãn yêu cầu đa thức x2±x±1, x2±1 vàx2 ±2x+ 1.
Gọi f(x) đa thức bậc n có tất hệ số ±1 Giả sử z nghiệm phương trình với |z|>1
|z|n=±zn−1±zn−2± ±1 ≤ |z|
n−1
+|z|n−2+ + = |z|
n
−1
(185)Điều kéo theo |z|n(|z| −2) ≤ −1; |z| < Như tất nghiệm f(x) = có giá trị tuyệt đối bé
Rõ ràng, đa thứcp(x) thỏa yêu cầu phải có dạngp(x) =x2+ax±1 với a∈Z Gọi x1
x2 nghiệm nó( khơng bắt buộc phải khác nhau) Vì x1x2 =±1 nên ta giả sử
rằng|x1| ≥1 và|x2| ≤1 Vìx1 x2 cịn nghiệm ngun đa thức có hệ số±1, nên ta
có|x1|<2 |a|=|x1+x2| ≤ |x1|+|x2|<2 + a∈ {±2,±1,0}
Nếua=±1 g(x) = cho ta lời giải Nếua= g(x) =x+ cho ta lời giải
Nếu a = ±2, hai đa thức x2 ±2x−1 có nghiệm với giá trị tuyệt đối lớn nên chúng
không thỏa yêu cầu Cuối cùng, đa thứcp(x) =x2±2x+ thỏa yêu cầu với q(x) = ∓1
Bình luận: Từ ngữ “ nghiệm” hiểu là” nghiệm ảo” khơng cần phải nói rõ Vì số phức khơng cần phải rõ, p(x) = x2+ax±1 có nghiệm thực nếu |a| ≥2 và trường hợp
|a| ≤1phải xử lí riêng
Chú ý cần quan tâm cho dù có hệ số kết luận|z|<2vẫn Tuy vậy, có lời giải đặc biệt x2 x2±x
Lời giải 2: Giả sử đa thức p(x) = a0 +a1x+x2 g(x) = b0 +b1x+ +bnxn cho
p(x)g(x) =c0+c1x+ cn+2xn+2 với mọick =±1
Khi đó|a0|=|b0|=|bn|= a0b1 =c1−a1b0;a0bk =ck−a1bk−1−bk−2; với k = 2, , n
Và |b1|≥ |a1| −1; |bk|≥ |a1bk−1| − |bk−2| −1với k= 2, , n
Giả sử Thì rõ ràng khơng thể hằng, vớin ≥1, ta có|b1| ≥2;|bk| ≥3|bk−1| − |bk−2| −1;với
k= 2, , n
Từ bất đẳng thức cuối( viết lại) ta có
|bk−1| − |bk−2| ≥2|bk−1| − |bk−2| −1≥2(|bk−1| − |bk−2|)−1
Ta thấy dãy dk = |bk| − |bk−1| , (k = 1, , n), thỏa dk ≥ 2dk−1 −1vớik ≥ 2( qui nạp)
Vì d1 = |b1| − ≥ kéo theo giả thiết qui nạp dk ≥ với k = 1, , n Tương đương
|bk| ≥ |bk−1|+ 1với k= 3, , nnên |bn| ≥ |b0|+n trái với giả thiết|b0|=|bn|= 1, n≥1
Điều kéo theo phải có dạnga0 +a1x+x2 với |a0|= 1;|a1| ≤2
Nếu|a1| ≤1 hay|a1|= a0 = đa thức g(x) tương ứng tồn (xem ví dụ lời giải
1)
Còn lại trường hợpa0 = 1, a0 =−1 Giả sử g(x) tồn khơng tính tổng qt ta giả
sửb0 = 1vàa1 = 1( nếub0 =−1thì thayq(x)bởi−q(x)và vớia1 =−2thì thayq(x)bởiq(−x)
Vớib0 = 1,a0 =−1,a1 = 2công thức quy nạp thànhb1 = 2−c1, bk = 2bk−1+bk−2−ck
với k = 2, , n Vậy b1 ≥1, b2 ≥2b1+ 1−c2 ≥2, quy nạp cho thấy bk ≥2 với k = 2, , n
lại trái với giả thiết|b0|= 1.Như khơng có đa thức p(x) thỏa tốn ngoại trừ đa thức
đã trình bày lời giải
2 MỘT SỐ BÀI TẬP TỰ LUYỆN
(186)Hướng dẫn Nếu f(x) đa thức khơng tồn n cho |f(n)| > Gọi p ước số nguyên tố f(n) Ta có p|f(n)|2n − 1. Mặt khác p|f(n+p)|2n+p − 1. Suy ra
p|2n+p−2n = 2n(2p−1) Do (2n−1,2n) = nên từ suy p|2p−1 Nhưng theo định lý Fermat thìp|2p−2.Như từ suy rap|1 Mâu thuẫn Vậy f(x) phải đa thức Đáp
sốf(x)≡1, f(x)≡ −1
Bài Chứng minh đa thức P(x) =xn+ 29xn−1+ 2009 với n số nguyên dương lớn
hơn hay phân tích thành tích đa thức với hệ số nguyên có bậc lớn hay
Hướng dẫn.Sử dụng tiêu chuẩn Eisenstein mở rộng sau
Cho đa thức P(x) = anxn+an−1xn−1 + +a1x+a0 ∈ Z[x].Giả sử tồn số nguyên tố p
số nguyên dương k thoả mãn đồng thời điều kiện sau 1)an không chia hết cho p
2)a0 chia hết cho p không chia hết cho p2
3)a1, a2, , an−k chia hết cho p
Khi đó, P(x) = Q(x).S(x) với Q(x), S(x) đa thức với hệ số ngun hai đa thức Q(x), S(x) có bậc nhỏ k
Bài Tìm tất đa thức P(x) thoả mãn điều kiện P2(x)−P(x2) = 2x4.
Hướng dẫn Đặt P(x) = anxn+R(x) với R(x) đa thức bậc r < n Khi
P2(x)−P(x2) = (a2
n−an)x2n+ 2anxnR(x) +R2(x)−R(x2).Từ suy P2(x)−P(x2) có
bậc 2n an6= có bậc n+r nếuan= 1.Từ suy 2≤n≤4 Hơn nữa,
n= an= vàr =
n= an= vàr =
Từ đây, dùng phương pháp hệ số bất định, dễ dàng tìm nghiệm là:x4+ 1, x3+x,2x2
và−x2.
Bài Tìm tất đa thức P(x) với hệ số thực thoả mãn điều kiệnP2(x) = P(x2)−2P(x).
Hướng dẫn.Đặt Q(x) =P(x) + Q2(x) = Q(x2). Chứng minhQ(x) = xn là đa thức bậc
n thoả mãn phương trình Từ suy nghiệm tốn làxn−1 đa thức đồng sốP(x)≡0và P(x)≡ −1
Tài liệu tham khảo
[1] Nguyễn Văn Mậu, ĐA THỨC ĐẠI SỐ VÀ PHÂN THỨC HỮU TỈ, Nhà Xuất Bản Giáo Dục, 2007
[2] Nguyễn Văn Mậu(Chủ biên), Trịnh Đào Chiến, Trần Nam Dũng, Nguyễn Đăng Phất, Nhà Xuất Bản Giáo Dục, 2008
(187)MỘT SỐ BÀI TOÁN VỀ CHIA HẾT ĐỐI VỚI CÁC ĐA THỨC ĐỐI XỨNG
Nguyễn Văn Ngọc, Viện Toán học
Trong tài liệu giới thiệu số toán tính chia hết đa thức đối xứng phản đối xứng Một đa thức gọi đối xứng, giá trị khơng thay đổi ta đổi chỗ hai biến gọi phản đối xứng, đổi dấu ta đổi chỗ hai biến Để giải tốn tính chia hết đa thức ta thường sử dụng Định lý Bézout, hệ kỹ phân tích thành nhân tử
Định lý Bézout Số dư phép chia đa thức f(x) cho x−a f(a)
Hệ quả.Đa thức f(x) chia hết cho x−a f(a) = 0, tức là, x=a nghiệm
f(x)
Xét số toán sau
Bài toán Chứng minh rằngx2n+xnyn+y2n chia hết chox2+xy+y2 khin bội
Lời giải Sử dụng công thức
x3−y3 = (x−y)(x2+xy+y2),
xk−yk = (x−y)(xk−1+xk−2y+ +xyk−2+yk−1)
dễ dàng thấy x3k−y3k chia hết chox2+xy+y2 Xét trường hợp sau 1)n= 3k.Ta có
x2n+xnyn+y2n =x6k+x3ky3k+y6k = (x6k−y6k) + (x3k−y3k) + 3y6k
Từ suy rax2n+xnyn+y2n khơng chia hét cho x2+xy+y2.
2)n= 3k+ Ta có
x2n+xnyn+y2n=x6k+2+x3k+1y3k+1+y6k+2 = =x2(x6k−y6k) +xy3k+1(x3k−y3k) +y6k(x2+xy+y2)
Suy trường hợp nàyx2n+xnyn+y2n chia hết cho x2+xy+y2
3)n= 3k+ Ta có
x2n+xnyn+y2n=x6k+4+x3k+2y3k+2+y6k+4=x4(x6k−y6k)+ +x2y3k+2(x3k−y3k) +y6k(x4 +x2y2 +y4) =
=x4(x6k−y6k) +x2y3k+2(x3k−y3k) +y6k(x2+x2y2y4)(x2−xy+y2)
Suy rax2n+xnyn+y2n chia hết cho x2+xy+y2 Vậy điều kiện cần đủ đểx2n+xnyn+y2n
(188)Bài toán Chứng minh với n ∈ Z+, đa thức x2n−xnyn+y2n không chia hết cho
x2+xy+y2.
Lời giải Giả sử x2n−xnyn+y2n chia hết cho x2+xy+y2, tức
x2n−xnyn+y2n = (x2+xy+y2)q(x, y),
trong đóq(x, y)là đa thức đối xứng với hệ số nguyên (do hệ số đa thức chia 1, cịn hệ số đa thức bị chia đa thức chia số nguyên) Trong đẳng thức cho
x= y = 1, ta = 3q(1,1), vơ lý q(1,1) số ngun Điều chứng tỏ đa thức
x2n−xnyn+y2n không chia hết cho x2 +xy+y2
Bài toán Với n∈Z+ nào, thì x2n+xnyn+y2n chia hết cho x2−xy+y2 ?
Lời giải Giả sử
x2n+xnyn+y2n= (x2−xy+y2)q(x, y), (1) q(x, y) đa thức đối xứng với hệ số nguyên
Ta xét hai trường hợp:
1)n số lẻ Trong đẳng thức (1) thay x −x ta
x2n−xnyn+y2n = (x2+xy+y2)q(−x, y)
Theo Bài toán đẳng thức xảy 2)n số chẵn Trong (7) thay x −x, ta
x2n+xnyn+y2n= (x2+xy+y2)q(−x, y)
Theo Bài tốn đẳng thức khin = 3m+ n= 3m+
Nếun = 3m+ , don số chẵn, nên m phải số lẻ, hay m= 2k+ 1,do n = 6k+
Nếun = 3k+ 2, n số chẵn, nênm phải số chẵn, haym = 2k,do n = 6k+
Vậy x2n+xnyn+y2n chia hết cho x2−xy+y2 khi khin = 6k+ 2 hoặc n= 6k+ 4,
với k ∈Z, n ∈Z+.
Bài tốn Với n ∈Z+ nào thì x2n−xnyn+y2n chia hết cho x2−xy+y2?
Lời giải Giả sử
x2n−xnyn+y2n= (x2−xy+y2)q(x, y), (2) q(x, y) đa thức đối xứng với hệ số nguyên Xét hai trường hợp n
1)n số chẵn Trong (2) thay x −x, ta
x2n−xnyn+y2n = (x2+xy+y2)q(−x, y)
Theo Bài tốn đẳng thức khơng thể xảy 2)n số lẻ Trong (2) thay xbởi −x, ta
x2n+xnyn+y2n= (x2+xy+y2)q(−x, y)
Theo Bài tốn đẳng thức khin = 3m+ n= 3m+
Nếun = 3m+ 1,thì n số lẻ, nên m phải số chẵn, tức m= 2k n = 6k+
Nếun = 3m+ 2,thì n số lẻ, nênm phải số lẻ, tức m= 2k−1.Khi n= 6k−1
Vậy x2n−xnyn+y2n chia hết cho x2−xy+y2 khi khi n= 6k±1, k ∈
(189)Bài toán Xác định n để (x+y)n+xn+yn chia hết cho x2+xy+y2
Lời giải Giả sử (x+y)n+xn+yn chia hết chox2+xy+y2 Khi ta có
(x+y)n+xn+yn= (x2+xy+y2)q(x, y), (3) đóq(x, y) đa thức đối xứng với hệ số nguyên Trong (3) thayx, y tương ứng bởix2, y2,
ta có
(x2+y2)n+x2n+y2n = (x4+x2y2+y4)q(x2, y2),
= (x2+xy+y2)(x2−xy+y2)q(x2, y2) (4) Đẳng thức (4) chứng tỏ(x2+y2)n+x2n+y2n phải chia hết cho x2−xy+y2.Ta có
(x2+y2)n−(xy)n = (x2+y2−xy)[(x2+y2)n−1+ (x2+y2)n−2+
+(x2+y2)(xy)n−2+ (xy)n−1] (5) Tiếp theo ta có
(x2+y2)n+x2n+y2n= [(x2+y2)n−(xy)n] + (x2n+xnyn+y2n) (6) Từ (5) (6) suy ra(x2+y2)n+x2n+y2nchia hết cho x2−xy+y2 khi khix2n+xnyn+y2n
chia hết cho x2 −xy+y2. Theo Bài tốn điều có khi n = 6k+ 2 hoặc
n= 6k+ với k∈Z, n ∈Z+.
Ngược lại, giả thiết n= 2m, với m = 3k+ m= 3k+ Thế
(x+y)n+xn+yn = (x+y)2m+x2m+y2m = [(x+y)2m−(xy)m] + (x2m+xmym+y2m)
Để ý
(x+y)2m−(xy)m = [(x+y)2]m−(xy)m] = [(x+y)2−xy]p(x, y) = (x2+xy+y2)p(x, y),
trong p(x, y) đa thức đối xứng với hệ số nguyên Do (x+y)2m−(xy)m chia hết cho
x2+xy+y2.
Mặt khác, m= 3k+ 1, m= 3k+ nên theo Bài toán 1, đa thứcx2m+xmym+y2m chia hết chox2 +xy+y2.
Kết luận: Đa thức (x+y)n+xn+yn chia hết cho x2+xy+y2 n = 6k+
hoặc n= 6k+ 4,với k∈Z, n∈Z+.
Bài toán Chứng minh x2n+xnyn+y2n chia hết cho x2 +xy+y2 khi khi m
(190)Lời giải Sử dụng công thức
x3−y3 = (x−y)(x2+xy+y2),
xk−yk = (x−y)(xk−1+xk−2y+ +xyk−2+yk−1)
dễ dàng thấy x3k−y3k chia hết chox2+xy+y2 Xét trường hợp sau 1)n= 3k.Ta có
x2n+xnyn+y2n =x6k+x3ky3k+y6k = (x6k−y6k) + (x3k−y3k) + 3y6k
Từ suy rax2n+xnyn+y2n không chia hét cho x2+xy+y2
2)n= 3k+ Ta có
x2n+xnyn+y2n=x6k+2+x3k+1y3k+1+y6k+2 = =x2(x6k−y6k) +xy3k+1(x3k−y3k) +y6k(x2+xy+y2)
Suy trường hợp nàyx2n+xnyn+y2n chia hết cho x2+xy+y2.
3)n= 3k+ Ta có
x2n+xnyn+y2n=x6k+4+x3k+2y3k+2+y6k+4=x4(x6k−y6k)+ +x2y3k+2(x3k−y3k) +y6k(x4 +x2y2 +y4) =
=x4(x6k−y6k) +x2y3k+2(x3k−y3k) +y6k(x2+x2y2y4)(x2−xy+y2)
Suy rax2n+xnyn+y2n chia hết cho x2+xy+y2. Vậy điều kiện cần đủ đểx2n+xnyn+y2n
chia hết chox2+xy+y2 bội
Bài tốn Chứng minh vơíi n ∈Z+, đa thức x2n−xnyn+y2n không chia hết cho
x2+xy+y2
Lời giải Giả sử x2n−xnyn+y2n chia hết cho x2+xy+y2, tức là
x2n−xnyn+y2n = (x2+xy+y2)q(x, y),
trong đóq(x, y)là đa thức đối xứng với hệ số nguyên (do hệ số đa thức chia 1, hệ số đa thức bị chia đa thức chia số nguyên) Trong đẳng thức cho
x= y = 1, ta = 3q(1,1), vơ lý ví q(1,1) số nguyên Điều chứng tỏ đa thức
x2n−xnyn+y2n không chia hết cho x2 +xy+y2
Bài toán Với n ∈Z+ nào thì x2n+xnyn+y2n chia hết cho x2−xy+y2?
Lời giải Giả sử
(191)Ta xét hai trường hợp:
1)n số lẻ Trong đẳng thức (7) thay x −x ta
x2n−xnyn+y2n = (x2+xy+y2)q(−x, y)
Theo Bài toán đẳng thức xảy 2)n số chẵn Trong (7) thay x −x, ta
x2n+xnyn+y2n= (x2+xy+y2)q(−x, y)
Theo Bài tốn đẳng thức khin = 3m+ n= 3m+
Nếun = 3m+ , don số chẵn, nên m phải số lẻ, hay m= 2k+ 1,do n = 6k+
Nếun = 3k+ 2, n số chẵn, nênm phải số chẵn, haym = 2k,do n = 6k+
Vậy x2n+xnyn+y2n chia hết cho x2−xy+y2 khi khin = 6k+ 2 hoặc n= 6k+ 4,
với k ∈Z, n ∈Z+.
Bài toán Với n ∈Z+ nào thì x2n−xnyn+y2n chia hết cho x2−xy+y2?
Lời giải Giả sử
x2n−xnyn+y2n= (x2−xy+y2)q(x, y), (8) q(x, y) đa thức đối xứng với hệ số nguyên Xét hai trường hợp n
1)n số chẵn Trong (8) thay x −x, ta
x2n−xnyn+y2n = (x2+xy+y2)q(−x, y)
Theo Bài tốn đẳng thức khơng thể xảy 2)n số lẻ Trong (8) thay xbởi −x, ta
x2n+xnyn+y2n= (x2+xy+y2)q(−x, y)
Theo Bài tốn đẳng thức khin = 3m+ n= 3m+
Nếun = 3m+ 1,thì n số lẻ, nên m phải số chẵn, tức m= 2k n = 6k+
Nếun = 3m+ 2, don số lẻ, nênm phải số lẻ, tức m= 2k−1.Khi n= 6k−1
Vậy x2n−xnyn+y2n chia hết cho x2−xy+y2 khi khi n= 6k±1, k ∈
Z, n∈Z+
Bài toán 10 Xác định n để (x+y)n+xn+yn chia hết cho x2+xy+y2
Lời giải Giả sử (x+y)n+xn+yn chia hết chox2+xy+y2 Khi ta có
(x+y)n+xn+yn= (x2+xy+y2)q(x, y), (9) đóq(x, y) đa thức đối xứng với hệ số nguyên Trong (9) thayx, y tương ứng bởix2, y2,
ta có
(x2+y2)n+x2n+y2n = (x4+x2y2+y4)q(x2, y2),
(192)Đẳng thức (10) chứng tỏ (x2+y2)n+x2n+y2n phải chia hết chox2−xy+y2 Ta có
(x2+y2)n−(xy)n = (x2+y2−xy)[(x2+y2)n−1+ (x2+y2)n−2+
+(x2+y2)(xy)n−2+ (xy)n−1] (11) Tiếp theo ta có
(x2+y2)n+x2n+y2n= [(x2+y2)n−(xy)n] + (x2n+xnyn+y2n) (12) Từ (11) (12) suy ra(x2+y2)n+x2n+y2nchia hết chox2−xy+y2khi khix2n+xnyn+y2n
chia hết cho x2 −xy+y2. Theo Bài toán điều có khi n = 6k+ 2 hoặc
n= 6k+ với k∈Z, n ∈Z+.
Ngược lại, giả thiết n= 2m, với m = 3k+ m= 3k+ Thế
(x+y)n+xn+yn= (x+y)2m+x2m+y2m= [(x+y)2m−(xy)m]+ +(x2m+xmym+y2m)
Để ý
(x+y)2m−(xy)m = [(x+y)2]m−(xy)m] = [(x+y)2−xy]p(x, y) = = (x2+xy+y2)p(x, y),
trong p(x, y) đa thức đối xứng với hệ số nguyên Do (x+y)2m−(xy)m chia hết cho
x2+xy+y2.
Mặt khác, vìm = 3k+ 1, m= 3k+ nên theo ví dụ 1, đa thứcx2m+xmym+y2m chia hết
chox2+xy+y2.
Kết luận: Đa thức (x+y)n+xn+yn chia hết cho x2+xy+y2 n = 6k+
hoặc n= 6k+ 4,với k∈Z, n∈Z+.
Bài toán 11 Chứng minh với số nguyên dương n, đa thức
f(x, y, z) = (x+y+z)2n+1−x2n+1−y2n+1−z2n+1
chia hết cho đa thức
g(x, y, z) = (x+y+z)3−x3−y3−z3
Lời giải Trước hết ta phân tích g(x, y, z) thành nhân tử Vì khix=−y, x=−z, y =−z
g = 0, nên theo định lý Bezout đa thức g(x, y, z) chia hết cho (x+y)(x+z)(y+z).Mặt khác, bậc củag 3, nên có dạng
g(x, y, z) =a(x+y)(x+z)(y+z)
Cho x=y=z = ta tìm a = Vậy ta có
g(x, y, z) = (x+y+z)3−x3−y3−z3 = 3(x+y)(x+z)(y+z)
(193)Bài toán 12 Chứng minh với số nguyên dương n ≥2, đa thức
f(x, y, z) = (x+y+z)2n+x2n+y2n+z2n−(x+y)2n−(y+z)2n−(z+x)2n
chia hết cho đa thức
g(x, y, z) = (x+y+z)4+x4 +y4+z4−(x+y)4−(y+z)4−(z+x)4
Lời giải Đặt
σ1 =x+y+z, σ2 =xy+yz+zx, σ3 =xyz, sk =xk+yk+zk
Khi ta có:
g(x, y, z) =σ4−(σ14−4σ12σ2+ 2σ22+ 4σ1σ3)−
−4(σ1σ22−σ1σ3−2σ22)−6(σ
2 −2σ1σ3) = 12σ1σ3 = 12(x+y+z)xyz
Tiếp theo biến đổif(x, y, z)như sau
f(x, y, z) = (x+y+z)4+ [x2n−(y+z)2n] + [y2n−(x+z)2n] + [z2n−(x+y)2n]
Ta cóx2n−(y+z)2n chia hết cho x2−(y+z)2 = (x+y+z)(x−y−z),do đó x2n−(y+z)2n
chia hết chox+y+z Tương tự, y2n−(x+z)2n z2n−(x+y)2n chia hết chox+y+z
Mặt khác
f(0, y, z) = f(x,0, z) =f(x, y,0) =
Do theo Định lý Bezout,f(x, y, z) chia hết cho xyz Do đó,f(x, y, z) chia hết cho (x+y+
z)xyz, suy f(x, y, z)chia hết cho g(x, y, z)
Bài toán 13 Chứng minh rằng, đa thức đối xứng f(x, y, z) chia hết chox−y, chia hết cho (x−y)2(x−z)2(y−z)2
Lời giải Giả sử
f(x, y, z) = (x−y)g(x, y, z)
Vì
f(x, y, z) =f(y, x, z) = (y−x)g(y, x, z) =−(x−y)g(y, x, z),
nên
g(y, x, z) =−g(x, y, z),
suy rag(x, y, z) đa thức phản đối xứng theo hai biến x, y Vậy g(x, y, z) chia hết cho x−y
Do đóf(x, y, z) chia hết cho (x−y)2. Vì f(x, y, z) là đa thức đối xứng, nên vai trò của x, y, z
là nhau, f(x, y, z) chia hết cho (x−z)2 (y−z)2 Vậy f(x, y, z) chia hết cho(x−y)2(x−z)2(y−z)2.
(194)Lời giải Xét đa thức theo biến x
f(x) = x3+ (kyz)x+ (y3+z3)
Theo Định lý Bezout,f(x)chia hết cho x+y+z =x−(−y−z) khif(−y−z) =
Ta có
f(−y−z) =−(y+z)3−kyz(y+z) + (y3+z3) = (k+ 3)yz(y+z) = 0, ∀y, z
Từ suy rak =−3
Vậy, điều kiện cần đủ để x3+y3+z3 +kxyz chia hết chox+y+z làk =−3
Bài toán 15 Cho a, b, c số nguyên Chứng minh rằng, x+y+z chia hết cho 6,
a3+b3+c3 cũng chai hết cho 6.
Lời giải Ta có
x3+y3+z3 = (x+y+z)(x2 +y2+z2−xy−yz−zx) + 3xyz
Theo giả thiết x+y+z chia hết cho 6, nên ba sốx, y, z phải có số chia hết cho Từ suy ra3xyz chia hết cho Vậy, theo đẳng thức trên, thìx3+y3+x3 chia hết cho
6
Bài tập
1.Chứng minh (x+y)n−xn−yn chia hết cho x2+xy+y2 khi khi n= 6k±1.
2.Chứng minh (x+y)2n+1+xn+2yn+2 chia hết cho x2+xy+y2 với n∈Z+.
3 Chứng minh rằng, đa thức đối xứng f(x, y) chia hết cho x2 −y2, thì chia hết cho
x3+y3 −(x+y)xy
4.Chứng minh với số tự nhiên p, q đa thức
xpyq+ypzq+zpxq−xqyp−yqzp−zqxp
chia hết cho(x−y)(x−z)(y−z)
5.Chứng minh rằng, với số tự nhiên k, m, n, đa thức
xkymzn+ykzmxn+zkxmyn−xnymzk−ynzmxk−zkxmyk
chia hết cho(x−y)(x−z)(y−z)
6.Cho đa thức
f(x, y, z) =xmyn+ymzn+zmxn−xnym−ynzm−znxm
Chứng minh rằng, f(x, y, z) chia hết cho (x+y)(x+z)(y+x), chia hết cho (x2−
y2)(x2 −z2)(y2 −z2)
7.Chứng minh rằng, đa thức
a4(b2+c2−a2)3+b4(c2 +a2−b2)3+c4(a2+b2 −c2)3
(195)NÉT ĐẸP HÀM SỐ TIỀM ẨN TRONG BÀI TOÁN BẤT ĐẲNG THỨC – BÀI TỐN TÌM GIÁ TRỊ LỚN
NHẤT, GIÁ TRỊ NHỎ NHẤT
Huỳnh Duy Thủy, Trường THPT Tăng Bạt Hổ - Bình Định
“Chứng minh bất đẳng thức ”“Tìm giá trị nhỏ nhất,giá trị lớn biểu thức ” Những cụm từ hàm chứa mảng kiến thức trọng tâm, “hóc búa” chương trình tốn học phổ thơng, mà phần nhiều thí sinh “ngại” “va chạm” Cịn phần kiến thức ln “thời sự”, “cuốn hút”, “quyến rũ” người học với đối tượng khá, giỏi
“ ví dịng sơng bắt nguồn từ suối nhỏ, tốn dù khó đến đâu có nguồn gốc từ tốn đơn giản ”
Tác giả viết mong góp chút “suy nghĩ” việc tìm “con đường” đến với “dòng suối nhỏ kia”
Bài viết trình bày theo hướng giải câu hỏi “kinh điển” - Bắt đầu từ đâu? - Khai thác, khám phá, phát kiến tạo vấn đề sao? - Giải pháp khả thi?
Điểm mấu chốt phương pháp vận dụng tính chất hàm số, xây dựng hàm số “tương thích” với tốn
1 Loại 1.Chọn trực tiếp tham số biến thiên làm biến
số.
Bài toán (USAMO) Cho a, b, c số thực thuộc đoạn [0; 1]
Chứng minh rằng: b+ac+1 + c+ab+1 +a+cb+1 + (1−a)(1−b)(1−c)≤1(1)
Lời giải - Vai trò số a, b, c khơng tính to˚ang quaùt, ta chọn a biến số Như ta hình thành hàm số f biến số a
y=f(a) với a∈[0; 1]
Xét hàm số f(a) = a b+c+1 +
b c+a+1 +
c
a+b+1 + (1−a)(1−b)(1−c)trên tập [0,1]
f0(a) = b+1c+1 − b
(c+a+1)2 −
c
(a+b+1)2 −(1−b)(1−c)
f00(a) = 2b (c+a+1)3 +
2c
(a+b+1)3 ≥0, ∀a∈[0; 1]
Suy hàm sốf0(a) đồng biến đoạn [0; 1]
* Trường hợp 1: f0(a)≥0, ∀a∈[0; 1]
Suy hàm sốf(a)đồng biến đoạn [0; 1]
Do ˜noù f(a)≤f(1) = b+c1+1 + c+1+1b +1+cb+1 ≤
b+c+1 +
b c+b+1 +
c c+b+1 = +b+c
(196)* Trường hợp 2: f0(a)<0, ∀a∈[0; 1]
Suy hàm sốf(a)nghịch biến đoạn [0; 1] Do đó:
f(a)≤f(0) = b
c+ +
c
b+ + (1−b)(1−c)
= b(b+ 1) +c(c+ 1) + (1−b)(1−c)(c+ 1)(b+ 1) (c+ 1)(b+ 1)
= +b+c+b 2c2 +b+c+bc ≤
1 +b+c+bc
1 +b+c+bc =
* Trường hợp 3: f0(a)đổi dấu đoạn [0; 1]
Mặt khác f0(a) hàm số liên tục đồng biến đoạn [0; 1]
Từ suy phương trình f0(a) = có nghiệm nhấta=α, α∈(0; 1)
Lập bảng biến thiên, ta nhận đượcf(a)≤1
Dấu đẳng thức xảy (a, b, c)là hoán vị (1,1,0),(1,0,0),(1,1,1) (0,0,0)
Bài toán Cho a, b, c độ dài cạnh tam giác Chứng minh rằng:
a(b−c)2 +b(c−a)2+c(a−b)2+ 4abc > a3+b3+c3(1)
(1)
Lời giải - Khơng tính tổng qt, giả sử 0< a≤b ≤c < a+b
Ta có: (1)⇔c3−(a+b)c2−(a2+b2−2ab)c+ (a3+b3−a2b−ab2)<0
Xét hàm số f(c) =c3−(a+b)c2−(a2+b2−2ab)c+ (a3+b3−a2b−ab2), ∀c∈[b, a+b)
f0(c) = 3c2 −2(a+b)c−(a−b)2
f0(c) = 0⇔3c2−2(a+b)c−(a−b)2 = 0⇔
c1 =
(a+b)−√(a+b)2+3(a−b)2
c2 = (a+b)+
√
(a+b)2+3(a−b)2
Nhận xét c1 ≤0, b < c2 < a+b
Bảng biến thiên
Vì0< a≤b nên f(b) = a2(a−2b)<0.Suy f(c)<0,∀c∈[b;a+b)
2 Loại 2: Biến đổi quy tham số biến
thiên cho trước.
Bài toán : [IM025] Cho số thực dươngx, y, z thay đổi thỏa mãn hệ thức x+y+z =
Tìm giá trị lớn biểu thức P =xy+yz+zx−2xyz
Lời giải Khơng tính tổng quát, ta giả sử x≥y≥z Từ giả thiết , suy 0< z ≤
Ta có:
P =xy(1˘2z) +z(1˘z)≤
x+y
2
2
(1−2z) +z(1−z)
=
1−z
2
2
(1−2z) +z(1−z) = −1
2z 3+
4z 2+
(197)- Xét hàm sốf(z) =−1 2z
3+ 4z
2+1
4, ∀z ∈ 0;
f0(z) = −3 2z
2+ 2z =−
z
2(3z−1), f
0(z) = 0⇔z =
3 (vì0< z ≤ 3)
Lập bảng biến thiên, ta nhận đượcP ≤f(z)≤f(13) = 277
Kết luận: maxP = 277 đạt khix=y=z= 13
Bài toán a, b, c độ dài cạnh tam giác có chu vi Tìm giá trị nhỏ biểu thức:
T = 3(a2+b2+c2) + 4abc
Lời giải Khơng tính tổng quát, ta giả sử: 0< a≤b≤c
Mặt khác a+b+c= c < a+b nên 1≤c <
Ta biến đổi:
T = 3(a+b)2−6ab+ 3c2+ 4abc= 3(3−c)2+ 3c2+ 2ab(2c−3) = 3(3−c)2+ 3c2−2(3−2c)ab≥3(3−c)2+ 3c2 −2(3−2c)
3−c
2
2
⇔T ≥c3−
2c 2+ 27
2
Xét hàm số: f(c) =c3−3 2c
2+ 27
2 với 1≤c < 2, f
0(c) = 3c2−3c
f0(c) = 0⇔3c(c−1) = 0⇔c= (Vì 1≤c < 32.)
Lập bảng biến thiên, ta nhận được:T ≥f(c)≥f(1) = 13
Kết luận: minT = 13 đạt tạia=b=c=
3 Loại 3: Hình thành biến số mới.
Bài toán Cho số thực dương x, y thỏa mãn điều kiện x2+y2 =xp
1−y2+y√1−x2.
Tìm giá trị nhỏ biểu thức P =x2 +x12 +y
2+
y2
Lời giải - Áp dụng bất đẳng thức cơsi, ta có:
P ≥(x2+y2)
1 + (x2 +y2)2
= (x2+y2) +
x2+y2
- Theo giả thiết, ta có:x2+y2 =xp1−y2 +y√1−x2
Áp dụng bất đẳng thức cơsi, ta có:
xp1−y2+y√1−x2 ≤ 2(x
2+ 1−y2) + 2(y
2+ 1−x2) = 1
Từ đó, ta có0< x2+y2 ≤1
Khi đó, ta có: P ≥t+4t, t=x2+y2, 0< t≤1.
Xét hàm số: f(t) = t+ 4t, tập (0; 1]
f0(t) = 1−
(198)Lập bảng biến thiên ta nhận đượcP ≥f(1) =
Kết luận: minP = đạt đượcx=y=
√
2
Bài toán Cho số thựca, b, c không âm thỏa mãn a+b+c= Tìm giá trị lớn biểu thức: P = (a2−ab+b2) (b2−bc+c2) (c2−ca+a2)
Lời giải Dựa theo giả thiết ta giả sử rằng: 0≤a≤b ≤c≤3
Suy ra:
a2−ab≤0
a2−ca≤0 ⇔
a2 −ab+b2 ≤b2
a2 −ca+c2 ≤c2
Từ ta nhận được:
P ≤b2c2 b2 −bc+c2=b2c2(b+c)2−3bc
Ta có: b+c≤3⇔2√bc≤b+c≤3⇔0≤bc≤9/4
Do đó:P ≤b2c2(9−3bc) = 9b2c2−3b3c3 Đặt t=bc,0≤t≤
4 Khi đó:P ≤9t
2−3t3 Xét hàm
số:f(t) = 9t2−3t3, ∀t∈
0,94 f0(t) = 18t−9t2
f0(t) = ⇔t1 = 0, t2 =
Lập bảng biến thiên , ta nhận đượcP ≤f(t)≤12
Kết luận: maxP = 12 đạt a= 0, b= 1, c= hoán vị (a, b, c)
4 Loại 4: Chọn hàm số đặc trưng, trường hợp biến
đã “phân li”
Bài toán (Olimpic Hồng Kông) Cho số thực dương a, b, c, d thỏa mãn điều kiện a+b+
c+d= Chứng minh rằng:
6 a3+b3+c3+d3≥a2 +b2+c2+d2+1 8(1)
Lời giải Ta có: (1) ⇔6a3−a2+ 6b3−b2+ 6c3−c2+ 6d3−d2 ≥
Xét hàm số f(x) = 6x3−x2−
8x, ∀x∈(0,1), f
0(x) = 18x2−2x−
f0(x) = 0⇔x1 = 1/4∈(0,1), x2 =−5/36 (loại)
Lập bảng biến thiên , ta nhận được6x3−x2− 8x≥ −
1
8(∗), ∀x∈(0,1)
Dựa theo kết (*) ta có:6a3−a2+ 6b3−b2+ 6c3−c2+ 6d3−d2 ≥4 −1
+58(a+b+c+d)
⇔6 (a3+b3 +c3+d3)≥a2+b2+c2+d2+
Đẳng thức xảy khia=b=c=d= 14
Bài toán Cho số thực x, y, z thay đổi thỏa mãn hệ thức x+y+z = Tìm giá trị nhỏ biểu thức P =√x2+ +py2 + +√z2+ 1
Lời giải Xét hàm số f(t) = √t+3
t2+1.Tập xác định D=R
Ta có: f0(t) = √1−3t (t2+1)3, f
0(t) = 0⇔1−3t= 0 ⇔t= 1/3
Lập bảng biến thiên , ta nhận đượcf(t) ≤√10, ∀t ∈ R⇔ √t+3
t2+1 ≤
√
(199)√
10√t2+ 1, ∀t ∈ R(1)
- Sử dụng kết (1), ta có
x+ 3≤√10√x2+ 1
y+ 3≤√10py2+ 1
z+ 3≤√10√z2+ 1
Do
(x+y+z) + 9≤√10√x2+ +py2+ +√z2+ 1
⇔1 + 9≤√10√x2+ +py2+ +√z2+ 1
⇔√10≤√x2+ +py2+ +√z2+ 1
Đẳng thức xảy khix=y=z = 13
Kết luận: minP =√10, đạt x=y =z = 13
5 Loại 5: Chọn hàm số đặc trưng, trường hợp biến
chưa “phân li”
Bài toán (Darij Grinberg – Old and new lnequalityl) Cho số thực dương a, b, c Chứng minh :
a
(b+c)2 +
b
(c+a)2 +
c
(a+b)2 ≥
9 (a+b+c)
Lời giải Do tính đẳng cấp, nên ta giả sử a+b+c= suy a, b, c∈(0; 3)
Khi bất đẳng thức cần chứng minh trở thành a
(3−a)2 +
b
(3−b)2 +
c
(3−c)2 ≥
Xét hàm số f(x) = x (3−x)2 −
1
2x, ∀x∈(0; 3), f
0(x) = 0⇔x= 1 ∈(0; 3).Lập bảng biến thiên , ta có
f(x)≥ −1
4, ∀x∈(0; 3)⇔
x
(3−x)2 ≥ 2x−
1
4(∗), ∀x∈(0; 3)
Vận dụng bất đẳng thức (*) ta nhận
a
(3−a)2 +
b
(3−b)2 +
c
(3−c)2 ≥
2(a+b+c)−
⇔ a
(3−a)2 +
b
(3−b)2 +
c
(3−c)2 ≥
Bất đẳng thức chứng minh dấu đẳng thức xảy khia=b=c=
Bài toán 10 (USAMO) Cho số thực dương a, b, c Chứng minh rằng:
(2a+b+c)2 2a2+ (b+c)2 +
(2b+c+a)2 2b2+ (c+a)2 +
(2c+a+b)2 2c2+ (a+b)2 ≤8
Lời giải - Do tính đẳng cấp nên giả sử a+b+c= 3, suy a, b, c∈(0; 3)
Khi bất đẳng thức cần chứng minh viết thành:
a2+ 6a+ 9 3a2−6a+ 9 +
b2+ 6b+ 9 3b2 −6b+ 9 +
(200)Xét hàm số f(x) = 3xx22+6−6xx+9+9 −
4
3x, ∀x∈(0; 3), f
0(x) = 0⇔x= 1 ∈(0,3) Lập bảng biến thiên , ta có 3xx22+6−6xx+9+9 −
4 3x≤
4
3, ∀x∈(0; 3) (∗)
Vận dụng bất đẳng thức (*) ta nhận
a2+ 6a+ 9 3a2−6a+ 9 +
b2+ 6b+ 9 3b2−6b+ 9 +
c2+ 6c+ 9 3c2 −6c+ 9 ≤
4
3(a+b+c) + =
Bất đẳng chứng minh, dấu đẳng thức xảy khia=b=c=
6 Loại 6:Chọn hàm đặc trưng, sử dụng hệ thức phụ trợ.
Bài toán 11 Cho số thực x, y, z thuộc đoạn [0; 3] Tìm giá trị nhỏ biểu thức
P =
4 + ln(1 +x)−y +
1
4 + ln(1 +y)−z +
1
4 + ln(1 +z)−x
Lời giải - Thật với lần vận dụng bất đẳng thức cô si, ta đánh giá “phân li” được:
P ≥
12 + [2 ln(1 +x)−x] + [2 ln(1 +y)−y] + [2 ln(1 +z)−z]
- Lúc số hạng vế phải xếp theo quy luật, điều làm “hé lộ” hàm số đặc trưng:f(t) = ln(1 +t)˘t,∀t∈[0; 3]
Từ giả thiết ta có:
4 + ln(1 +x)−y >0 + ln(1 +y)−z >0 + ln(1 +z)−x >0
Sử dụng bất đẳng thức cô si, ta có:
[4 + ln(1 +x)−y] + [4 + ln(1 +y)−z] + [4 + ln(1 +z)−x]
≥3p3
[4 + ln(1 +x)−y] [4 + ln(1 +y)−z] [4 + ln(1 +z)−x]
- Lại áp dụng bất đẳng thức cô si, ta có
P ≥33 s
1
[4 + ln(1 +x)−y] [4 + ln(1 +y)−z] [4 + ln(1 +z)−x]
Nhân vế theo vế bất đẳng thức trên, ta
[4 + ln(1 +x)−y+ + ln(1 +y)−z+ + ln(1 +z)−x].P ≥9
⇔P ≥
12 + [2 ln(1 +x)−x] + [2 ln(1 +y)−y] + [2 ln(1 +z)−z]
Xét hàm số:
f(t) = ln(1 +t)˘t,∀t∈[0; 3], f0(t) =
1 +t −1 =
1−t
1 +t f
0